You are on page 1of 151

Aban - 1

Laurel v. Abrogar

G.R. No. 155076, January 13, 2009

Facts:

Philippine Long Distance Telephone Company (PLDT) filed a complaint for theft under Article 308
of the Revised Penal Code against Baynet Co., Ltd. (Baynet) for stealing its business. PLDT
alleged that Baynet offered phone cards to people in Japan to call their friends and relatives in
the Philippines using PLDT's facilities and equipment.

Issue:

Whether or not the PLDT's business of providing telecommunication services is a personal


property under Article 308 of the Revised Penal Code.

Held:

No, PLDT's business of providing telecommunication services is not a personal property under
Article 308 of the Revised Penal Code.

Term personal property as used in Art. 308 of RPC should be interpreted in the context of the
Civil Code's definition of real and personal property. Consequently, any personal property,
tangible or intangible, corporeal or incorporeal, capable of appropriation may be the subject of
theft so long as the same is not included in the enumeration of Real Properties under the Civil
Code. The only requirement for personal property to capable of theft, is that it be subject to
appropriation.

PLDT's business is intangible and cannot be taken by another and not the proper subjects of
theft because they are without form or substance.
Abragan 2

BICERRA VS. TENEZA

FACTS:

The Bicerras were the owners of the house, worth P200.00, built on and owned by them and situated in
the said municipality Lagangilang. In January 1957, the Tenezas forcibly demolished the house, claiming
to be the owners thereof. The materials of the house, after it was dismantled, were placed in the custody
of the barrio lieutenant of the place.The Bicerras filed a complaint claiming actual damages of P200,
moral and consequential damages amounting to P600, and the costs. The CFI Abra dismissed the
complaint claiming that the action was within the exclusive (original) jurisdiction of the Justice of the
Peace Court of Lagangilang, Abra.

ISSUE: W/N The action involves title to real property, as appellants contend, and therefore is cognizable
by the Court of First Instance.

HELD: Yes, the action involves a real property.

A house is classified as immovable property by reason of its adherence to the soil on which it is built (Art.
415, par. 1, Civil Code). This classification holds true regardless of the fact that the house may be
situated on land belonging to a different owner. But once the house is demolished, as in this case, it
ceases to exist as such and hence its character as an immovable likewise ceases. It should be noted that
the complaint here is for recovery of damages. This is the only positive relief prayed for by appellants. To
be sure, they also asked that they be declared owners of the dismantled house and/or of the materials.
However, such declaration in no wise constitutes the relief itself which if granted by final judgment could
be enforceable by execution, but is only incidental to the real cause of action to recover damages.

The order appealed from is affirmed. The appeal having been admitted in forma pauperis, no costs are
adjudged.
Absuelo 3
Abudante 4
Tsai vs. CA 366 SCRA 324

FACTS:

EVERTEX secured a loan from PBC, guaranteed by a real estate and chattel mortgage over a
parcel of land where the factory stands, and the chattels
located therein, as included in a schedule attached to the mortgage
contract. Another loan was obtained secured by a chattel mortgage over
properties with similar descriptions listed in the first schedule. During the
date of execution of the second mortgage, EVERTEX purchased machineries and
equipment.

Due to business reverses, EVERTEX filed for insolvency proceedings. It failed to pay its
obligation and thus, PBC initiated extrajudicial foreclosure
of the mortgages. PBC was the highest bidder in the public auctions, making it the
owner of the properties. It then leased the factory premises
to Tsai. Afterwards, EVERTEX sought the annulment of the sale and conveyance of the
properties to PBC as it was allegedly a violation of the INSOLVENCY LAW.

The RTC held that the lease and sale were irregular as it involved properties not
included in the schedule of the mortgage contract.
ISSUES:

1. Whether or not the contested properties are personal or movable properties


2. Whether or not the sale of these properties to a third person (Tsai) by the bank through an
irregular foreclosure sale is valid.

HELD:

While it is true that the controverted properties appear to be immobile, a perusal of the contract
of REM and CM executed by the parties gives a contrary indication. In the case at bar, both the
trial and appellate courts show that the intention was to treat the machineries as movables or
personal property.

Assuming that the properties were considered immovables, nothing


detracts the parties from treating it as chattels to secure an obligation under the principle
of estoppel.
Agcopra -5

G.R. No. L-50466 May 31, 1982

CALTEX (PHILIPPINES) INC., petitioner,


vs.
CENTRAL BOARD OF ASSESSMENT APPEALS and CITY ASSESSOR OF
PASAY, respondents.

FACTS:
This case is about the realty tax on machinery and equipment installed by Caltex (Philippines) Inc.
in its gas stations located on leased land.
The said machines and equipment are loaned by Caltex to gas station operators under an
appropriate lease agreement or receipt. Under the lease contract, the operators are bound to return
the equipment in good condition upon demand and that Caltex retains ownership thereof during
the term of the lease.
The Central Board of Assessment Appeals held that the machines and equipment are real property
tax within the meaning of sections 3(k) & (m) and 38 of the Real Property Tax Code and that the
definitions of real property and personal property in articles 415 and 416 of the Civil Code are not
applicable to this case.
Caltex filed a petition for certiorari praying to set aside the Boards decision and for a declaration
that the said machines and equipment are personal property not subject to realty tax.
ISSUE:
Whether or not the machines and equipment are real property subject to realty tax.
HELD:
Yes, the machines and equipment are real property subject to realty tax. Said equipment and
machinery, as appurtenances to the gas station building or shed owned by Caltex (as to which it is
subject to realty tax) and which fixtures are necessary to the operation of the gas station, for
without them the gas station would be useless, and which have been attached or affixed
permanently to the gas station site or embedded therein, are taxable improvements and machinery
within the meaning of the Assessment Law and the Real Property Tax Code.
Improvements on land are commonly taxed as realty even though for some purposes they might be
considered personalty.
Petition for certiorari is dismissed.
Alagao 6
CASE No. 06
MERALCO vs. CBAA, G.R. No. L-46245, May 31, 1982

Facts:
Pursuant to a pipeline concession issued under the Petroleum Act of 1949, Republic Act No. 387,
Meralco Securities installed from Batangas to Manila, a pipeline system consisting of cylindrical
steel pipes joined together and buried not less than one meter below the surface along the shoulder
of the public highway.

The pipes are embedded in the soil while the valves are welded to the pipes so as to make the
pipeline system one single piece of property from end to end.

Pursuant to the Assessment Law, Commonwealth Act No. 470, the provincial assessor of Laguna
treated the pipeline as real property and issued Tax Declarations.

Issue:
Whether or not the Meralco Securities Pipeline System in Laguna is a subject to a realty tax.

Held:
The Court ordered that CBAA did not act with grave abuse and discretion and acted within its
jurisdiction in sustaining the holding of the provincial assessor that Meralco Securities Pipeline
System in Laguna is subject to a realty tax. CBAA reasoned out that the pipes are machinery or
improvements and regarded as realty because they are constructions adhered to the soil. It is
attached to the land in such a way that it cannot be separated therefrom without dismantling the
steel pipes which are welded to the pipeline. In so far as the pipeline uses valves, pumps and
control devices to maintain the flow of the oil, it is in a sense a machinery within the meaning of the
Real Property Tax Code.

Thus, the Court dismiss the petition and the questioned decision and resolution of the lower court
is affirmed.

Original Case Link:

http://www.lawphil.net/judjuris/juri1982/may1982/gr_l_46245_1982.html
Alapag 7

METROPOLITAN BANK, & TRUST COMPANY vs. Hon. FLORO T. ALEJO, Presiding
Judge of Branch 172 RTC Valenzuela;

FACTS:

As security for the payment of the loans obtained from Metropolitan Bank and Trust
Company by the spouses Raul and Cristina Acampado, a Real Estate Mortgage over a parcel of
Nullity of land was executed by same spouses. Subsequently a Complaint for Declaration of the
TCT of the spouses was filed by Sy Tan Se in the RTC of Valenzuela. The bank was not made a
party to the said civil case(complaint for declaration of nullity of TCT.) They werent notified as
well. The spouses defaulted in the payment of their loan and extrajudicial foreclosure
proceedings were initiated. The bank submitted the highest and winning bid. A certificate of
sale was issued in their favor.

When they were about to get their TCT from the Register of Deeds, petitioner was
informed of the existence of the decision in the aforementioned civil case (complaint for
declaration of nullity of TCT) declaring the Spouses Acampadoss TCT null and void.

The bank filed with the CA a petition for the annulment of the RTC Decision. The CA
dismissed their petition and ruled that the bank should have filed a petition for relief from
judgment or an action for quieting of title.

ISSUES:

1. Whether or not a petition for annulment of judgment is the proper remedy available to
the bank.
2. Whether or not the judgment of the trial court (declaring the Spouses Acampados TCT
null and void) should be declared null and void

HELD :

1. Yes. Petition for annulment of judgment was the proper remedy available to the bank. Such
bank was deprived of its duly registered property right without due process of the law by
not including the petitioner as defendant to the civil case by Sy Tan Se even if he was an
indispensable party.

2. Yes. The judgment of the trial court should also be declared null and void because the bank,
which is an indispensable party, was not impleaded in the civil case. The absence of an
indispensable party renders all subsequent actuations of the court null and void, for want of
authority to act, not only as to the absent parties but even as to those present.
Aliman 8

Case Number 8:

Chavez v. Public Estates Authority


415 SCRA 403 (2003)

FACTS:
The Register of Deeds of Paranaque issued TCTs in the name of PEA, as successor of CDCP,
covering the 3 reclaimed islands known as the Freedom Islands located in the southern portion
of the Manila-Cavite Coastal Road. On April 25, 1995 , PEA entered into a Joint Venture
Agreement with AMARI, a private corporation, to develop the Freedom Islands and to reclaim
an additional 250 hectares of submerged areas surrounding the islands. With this, PEA obliged
itself to transfer to AMARI 70% of PEAs proportionate share in the reclaimed areas

ISSUE:
Is the Joint Venture Agreement, which will transfer the reclaimed lands, valid?

RULING:
No. Under Sec.2, Article XII of the Constitution, the foreshore and submerged areas of Manila
Bay are part of lands of the public domain, and consequently owned by the State. As such, these
areas shall not be alienated, unless they are classified as agricultural lands of the public domain.
The mere reclamation of these areas by PEA does not convert these into alienable or disposable
lands of the public domain. There must be a law or presidential proclamation officially or
classifying these reclaimed lands as alienable or disposable and open to disposition or
concession. Moreover, these reclaimed lands cannot be classified as alienable or disposable if the
law has reserved them for some public or quasi-public use. Absent these two official acts
classification as alienable and a declaration that these lands are not needed for public service,
lands reclaimed by PEA remain inalienable lands of public domain that cannot be transferred
even by Joint Venture Agreement. Hence, The Joint Venture Agreement is unconstitutional.
Angeles 9

9) Republic v. Santos

FACTS:

The respondents purchased three (3) parcels of unregistered land. After the said purchase, the
respondents caused the survey and consolidation of the parcels of land into a single lotLot 3. They
then filed an Application for Original Registration of Lot 3. They alleged that their predecessors-in-interest
i.e., the previous owners of the parcels of land making up Lot 3, have been in continuous, uninterrupted,
open, public and adverse possession of the said parcels since time immemorial.

The government, through the Office of the Solicitor General, filed the lone opposition to the respondents
application. It insists that Lot 3 still forms part of the public domain and, hence, not subject to private
acquisition and registration. The government, however, presented no further evidence to controvert the
claim of the respondents.

The RTC rendered a ruling granting the respondents Application for Original Registration of Lot 3. The
Court of Appeals affirmed the RTCs decision on appeal.

ISSUE:

WON the Court of Appeals erred in affirming the RTC ruling granting original registration of Lot 3 in favor
of the respondents.

RULING:

We grant the petition.

Regalia simply means that the State is the original proprietor of all lands and, as such, is the general
source of all private titles. Thus, pursuant to this principle, all claims of private title to land, save those
acquired from native title, must be traced from some grant, whether express or implied, from the State.
Absent a clear showing that land had been let into private ownership through the States imprimatur, such
land is presumed to belong to the State.

Being an unregistered land, Lot 3 is therefore presumed as land belonging to the State. It is basic that
those who seek the entry of such land into the Torrens system of registration must first establish that it
has acquired valid title thereto as against the State, in accordance with law.
Arellano 10

Villarico v. Sarmiento
Case No. 10

Facts:

Villarico here is an owner of a lot that is separated from the Ninoy Aquino Avenue
highway by strip of land belonging to the government. Vivencio Sarmiento had a building
constructed on a portion of the said government land and a part thereof was occupied
by Andoks LitsonCorp. In 1993, by means of a Deed of Exchange of Real
Property, Villarico acquired a portion of the same area owned by the government. He
then filed an accion publiciana alleging that respondents (Vivencio) on the government
land closed his right of way to the Ninoy Aquino Avenue and encroached on a portion of
his lot.

Issue:

Whether or not VIllarico has a right of way to the NAA.

Held:

No. It is not disputed in this case that the alleged right of way to the lot belongs to the
state or property of public dominion.

It is intended for public use meaning that it is not confined to privileged individuals but is
open to the indefinite public. Records show that the lot on which the stairways were built
is for the use of the people as passageway hence, it is a property for public dominion.
Public dominion property is outside the commerce of man and hence, it cannot be
alienated or leased or otherwise is the subject matter of contracts, cannot be acquired
by prescription against the state. Cannot be the subject of attachment and execution
and be burdened by any voluntary easement.

It cannot be burdened by a voluntary easement of right of way in favor of the petitioner


and petitioner cannot appropriate it for himself and he cannot claim any right of
possession over it.
Avenido 11

Menchavez vs. Teves


Facts:
On February 28, 1986 a Contract of Lease of a Fishpond was executed between the plaintiffs, as
lessor and the respondent as lessee. On June 2, 1988 the fishpond dikes constructed by respondent
was demolished as ordered by Cebu RTC. As a result, he filed a Complaint for damages alleging that
petitioners violated their contract of lease specially the peaceful and adequate enjoyment of
property for the entire duration of the contract. The RTC declared the Contract as void and declared
both parties in pari delicto. The CA however reversed the decision. Thus the present appeal.
Issue:
Are the parties of the Contract of Lease in Pari delicto?
Ruling:
Under the 1987 Constitution specifically declares that all lands of the public domain, waters,
fisheries and other natural resources belong to the State.18Included here are fishponds, which may
not be alienated but only leased, thus making the Contract of Lease void. We apply the maxim
NEMO DAT QUOD NON HABET - meaning ONE CANNOT GIVE WHAT HE DOES NOT HAVE.
Petitioners are applicants of the fishponds. Even if the State grants them the lease, the law also
prohibits sublease of the fishponds.
The parties are in pari delicto or in equal fault and the law deny them aid for any claims against the
other and courts leave them as they are. However Article 1412 of the Civil Code provides the
exception.
"Art. 1412. If the act in which the unlawful or forbidden cause consists does not constitute a
criminal offense, the following rules shall be observed:
"(1) When the fault is on the part of both contracting parties, neither may recover what he has
given by virtue of the contract, or demand the performance of the others undertaking;
"(2) When only one of the contracting parties is at fault, he cannot recover what he has given by
reason of the contract, or ask for the fulfillment of what has been promised him. The other, who is
not at fault, may demand the return of what he has given without any obligation to comply with his
promise."
The petition is GRANTED.
Bantuas 12

DACANAY V ASISTIO, JR
[G.R. No. 93654 May 6, 1992]

Facts:

A municipal ordinance was passed designating certain city and municipal streets, roads and open
spaces as sites for flea markets, one of which is Heroes del 96 where herein petitioner resides. A
change in the administration of the City of Caloocan caused the demolition of the market stalls. Stall
owners filed before the RTC an action for prohibition against city officials. The court issued the writ
prayed for, but subsequently dismissed the petition and lifted the writ of preliminary injunction which
it had earlier issued after it was found that subject streets are of public dominion, hence, outside the
commerce of man. However, shortly after the decision came out, the city administration in Caloocan
City changed hands and the policy of clearing and cleaning up the city streets did not push through.
As the stall holders continued to occupy the land through the tolerance of the public respondents,
and in clear violation of the court decision previously issued, Dacanay filed the present petition
for mandamus praying that the public respondents be ordered to enforce the final decision which
upheld the city mayor's authority to order the demolition of market stalls and other pertinent laws.

Issue:
May public streets or thoroughfares be leased or licensed to market stallholders by virtue of a city
ordinance or resolution?

Held:

No. A public street is property for public use, hence, outside the commerce of man and may not be
the subject of lease or of any other contract. The right of the public to use the city streets may not be
bargained away thru a contract. As the stallholders pay fees to the City Government for the right to
occupy portions of the public street, the City Government, contrary to law, has been leasing portions
of the streets to them. Such leases or licenses are null and void for being contrary to law. Thus, City
Mayor may not infringe upon the vested right of the public to use city streets for the purpose they
were intended to serve, i. e. as areteries of travel for vehicles and pedestrians.

Petition granted.
Banas - 13
MANILA LODGE 761
VS
COURT OF APPEALS
73 SCRA 162
______________________________________________________________________________
_______
FACTS:

The Philippine Commission enacted Act No. 1306 which authorized the City of Manila to
reclaim a portion of Manila Bay. The reclaimed area was to form part of the Luneta extension.
The act provided that the reclaimed area shall be the property of the City of Manila, and the city
is authorized to set aside a tract of the reclaimed land for a hotel site and to lease or to sell the
same. Later, the City of Manila conveyed a portion of the reclaimed area to Petitioner. Then
Petitioner sold the land, together with all the improvements, to the Tarlac Development
Corporation (TDC).

ISSUE:

W/the subject property was patrimonial property of the City of Manila.

HELD:

No, the petitions were denied for lack of merit. The court found it necessary to analyze all the
provisions of Act No. 1360, as amended, in order to unravel the legislative intent. The grant
made by Act No. 1360 of the reclaimed land to the City of Manila is a grant of a public nature.
Such grants have always been strictly construed against the grantee because it is a gratuitous
donation of public money or resources, which resulted in an unfair advantage to the grantee. In
the case at bar, the area reclaimed would be filled at the expense of the Insular Government and
without cost to the City of Manila. Hence, the letter of the statute should be narrowed to exclude
matters which, if included, would defeat the policy of legislation.
Biruar 14
Republic VS. CA
132 SCRA 514

Alluvium must be the exclusive work of nature. It has 3 requirements: 1) that the deposit be gradual
and imperceptible; 2) through the current of the river; and 3) the land where the accretion takes place
is adjacent to the river bank. Deposits made by human intervention are excluded.

FACTS:
The respondents (Tancincos) were registered owners of a parcel of land in Bulacan, bordering on
the Maycauayan and Bocaue Rivers. They filed an application for the registration of three lots
adjacent to their fishpond, but because of the recommendation of the Commissioner, they only
pushed for the registration of two. The RTC and CA granted the petition despite the opposition of
the Bureau of Lands.

The respondents based their claim on accretions to their fishponds. They presented a lone witness
(their overseer). The Bureau of Lands argue that the lands in dispute are not accretions. They assert
that what actually happened was that the respondents simply transferred their dikes simply further
down the river bed of the Meycauayan River. Thus, if there was any accretion to speak of, it was
man-made.

Respondents counter that the their evidence shows that accretion happened without human
intervention and that the transfer of the dikes occurred only after.

ISSUE:
Whether accretion took place

RULING:
No. Alluvion must be the exclusive work of nature. There is not evidence that the addition to said
property was made gradually thro
Bucay 15

MANILA INTERNATIONAL AIRPORT AUTHORITY VS CA


GR No. 155650, July 20, 2006, 495 SCRA 591
Facts: The Officers of Paranaque City sent notices to MIAA due to real estate tax delinquency.
When MIAA failed to settle the entire amount, the Officers levied and auctioned the land and
buildings of MIAA. MIAA sought for a TRO from the CA but failed to do so within the 60 days
reglementary period, so the petition was dismissed. MIAA then sought for the TRO with the SC
a day before the public auction, MIAA was granted with the TRO but unfortunately the TRO was
received by the Paranaque City officers 3 hours after the public auction. MIAA claims that
although the charter provides that the title of the land and building are with MIAA still the
ownership is with the Republic of the Philippines. MIAA also contends that it is an
instrumentality of the government and as such exempted from real estate tax. That the land
and buildings of MIAA are of public dominion therefore cannot be subjected to levy and auction
sale.
Issues: (1) Whether or not MIAA is an instrumentality of the government and not a government
owned and controlled corporation and as such exempted from tax. (2) Whether or not the land
and buildings of MIAA are part of the public dominion and thus cannot be the subject of
levy and auction sale.
Ruling:
(1) Under the LGC, government owned and controlled corporations are not exempted from
real estate tax. MIAA is not a government owned and controlled corporation, for
to become one MIAA should either be a stock or non-stock corporation. MIAA is not a
stock corporation for its capital is not divided into shares. It is not a non-stock
corporation since it has no members. MIAA is an instrumentality of the government
vested with corporate powers and government functions.

(2) The court held that the land and buildings of MIAA are part of the public dominion.
Since the airport is devoted for public use, for the domestic and international travel and
transportation. Even if MIAA charge fees, this is for support of its operation and
for regulation and does not change the character of the land and buildings of MIAA as
part of the public dominion. As part of the public dominion the land and buildings of
MIAA are outside the commerce of man. To subject them to levy and public auction is
contrary to public policy. Unless the President issues a proclamation withdrawing the
airport land and buildings from public use, these properties remain to be of public
dominion and are inalienable. As long as the land and buildings are for public use the
ownership is with the Republic of the Philippines
Cadag 16
Tufexis vs Olaguera
32 Phil 654

Facts: During the Spanish regime, Ricardo Pardo y Cabaas was allowed by the Spanish government
to have the usufruct of a public market for 40 years. Ricardo died, and the usufruct was inherited by
Ricardo Pardo y Pujol, his son. When Pujol became indebted, hi properties were sold at an auction
sale, and the usufruct was bought by Tufexis. Then a fire destroyed the market. The Council granted
Pujol the right to reconstruct the building and continue the usufruct. Tufexis complained on the
ground that he had bought at the auction sale Pujols usufruct.
Issue: Whether Tufexis can be given the usufruct and administration of the market
Held: Tufexis cannot be given the right because the right is of public character and could not be
bought at an auction sale. For Tufexis to take Pujols place would be contrary to law, for this would
be allowing a stranger who had not been selected by the government, to take over a public function.
On the other hand, in terms of the concession given to Cabaas, were personal and transferable
only by inheritance. Tufexis, not being an heir of Cabaas, cannot therefore exercise the right.
Cagampang - 17. Roman R. Santos v Hon. Florencio Moreno
21 SCRA 1141
December 4, 1967
Facts:
The Zobel famiy owned the Hacienda San Esteban. It was devoted to the planting and cultivation of
nipa palms. Accessibility through the nipa palms deep into the hacienda posed as a problem so its
administrator dug canals. By the gradual process of erosion, these canals acquired the
characteristics and dimensions of rivers.
Later, they converted the forest of nipa groves into a web of fishponds for bangus culture. They
constructed dikes and closed the canals criss-crossing the hacienda.
Sometime in 1925 or 1926, a portion of the hacienda was sold to Roman Santos who also
transformed the swamp land into a fishpond. Santos closed and built dikes across Sapang Malauling
Maragul, Quiorang Silab, Pepangebunan, Bulacus, Nigui and Nasi.
Residents complained that the closing of the canals caused floods during the rainy season and it
deprived them of their means of transportation and fishing grounds. They demanded the re-
opening of the canals. Subsequently, the mayor, accompanied by the residents, went to the hacienda
and opened the closed dikes.
Santos filed Civil Case No 448 in the CFI of Pampanga which preliminarily enjoined the mayor and
others from demolishing the dikes across the canals. The municipal officials of Macabebe countered
by filing a complaint in the same court. The CFI rendered judgment in both cases against Santos
who immediately elevated the case to the Supreme Court.
Issue:
Do the streams involved in this case belong to the public domain or to the owner of Hacienda
Esteban?
Held:
Pursuant to Art 71 of the Spanish Law of Waters of August 3, 1866 and Art 408 (5) of the Spanish
Civil Code, channels of creeks and brooks belong to the owners of the estates over which they flow.
The channels, therefore, of the streams in question, which may be classified as creeks, belong to the
owners of Hacienda San Esteban. The streams, considered as canals, of which they originally were,
are of private ownership in contemplation of Art 339 (1) of the Spanish Civil Code. Under Art 339,
canals constructed by the State and devoted to public use are of public ownership. Conversely,
canals constructed by private persons within private lands and devoted exclusively for private use
must be of private ownership.
There being a showing that the canals were constructed for the use and benefit of its owner to the
exclusion of the public, the canals in question are adjudged to be of private ownership.
Capuyan 18
Almagro v Kwan
G.R. No. 175806, October 20, 2010

Facts:
This case involves a 17,181 sqm parcel of land located at Negros Oriental and is registered in the
name of spouses Kwan Chin and Zosima Sarana. Respondents are the legitimate children of spouses
Kwan Chin and Zosima Sarana, who both died intestate.

Respondents filed with the MTC an action for recovery of possession and damages. However, the
MTC dismissed the complaint on the ground that the remaining dry portion of the subject lot has become
foreshore land and should be returned to the public domain and that plaintiff cannot use the doctrine of
indefeasibility of their Torrens title, as property in question is clearly foreshore land.

Respondents appealed to the RTC which then conducted ocular inspections on two separate dates
and observed that the small portion actually remained dry even during high tide. Thus, the RTC
concluded that the disputed remaining portion of the subject lot is not foreshore land. The CA affirmed
the decision of the RTC with a modification that petitioners are illegally occupying a portion of the
subject lot. Thus, they were ordered to vacate the premises and/or remove the houses and/or cottages
constructed therein.

Issue: WON the disputed portion of the subject lot is still private land or has become foreshore land
which forms part of the public domain.

Held:
The Supreme Court denied the petition and affirmed the Resolution of the CA that the disputed
land is not foreshore land and ruled that the land's proximity to the waters alone does not automatically
make it a foreshore land.
Castillo 19

BINALAY VS. MANALO


G.R. NO. 92161, MARCH 18, 1991
(ACCRETION; RIVER: PROPERTY OF PUBLIC DOMINION)

FACTS: Guillermo Manalo acquired two (2) parcels of land in Isabela which were
consolidated into one [Lot No. 307 (10.45 hectares)]. A 22 hectare land was formed
between the west and east branches of the Cagayan river which looked like an island (Lots
821 and 822). Lot 821 was directly opposite to Lot 307, separated by the eastern branch
during rainy season and exposed bed during dry season. Respondent claims that Lot 821
belongs to him by accretion to the submerged portion of his property. Petitioners who are
in possession and in cultivation lot 821 insist their ownership. Respondent filed action for
quieting of title.
ISSUE: Whether Lot 821 belongs to respondent by way of accretion.
HELD: No. The word old bed meant that it had existed even before sale to Manalo. Art.
420 provides that rivers are part of the public dominion, therefore, the vendors could not
have validly sold land which is property and respondent could not have acquired
ownership of the eastern part of the bed even if included in the deed of sale. Rivers
include (1) running waters, (2) beds, (3) banks. The regularly submerged portion of the
eastern bed is declared property of public dominion.
Ceniza 20

Hilario vs. The City of Manila


G.R. No. L-19570, April 27, 1967

Facts:
Dr. Jose Hilario was the registered owner of a large tract of land which was inherited by Jose
Hilario Jr., plaintiff. During the lifetime of plaintiff's father, the Hilario estate was bounded on
the western side by the San Mateo River. To prevent its entry into the land, a bamboo and lumber
post dike or ditch was constructed on the northwestern side. This was further fortified by a
stonewall built on the northern side. However, in 1937, a great and extraordinary flood occurred
which inundated the entire place. The river destroyed the dike on the northwest, left its original
bed and meandered into the Hilario estate, segregating from the rest thereof a lenticular place of
land. In 1945 the U.S. Army opened a sand and gravel plant within the premises and started
scraping, excavating and extracting soil, gravel and sand from the nearby areas the River. The
operations eventually extended northward into this strip of land. In 1947, the plant was turned
over to defendants who took over its operations.

Issue:
Whether the new riverbanks lining the said course would be of public ownership when a river,
leaving its old bed, changes its original course and opens a new one through private property

Ruling:
All riverbanks are of public ownership including those formed when a river leaves its old bed
and opens a new course through a private estate.

Art. 339 of the old Civil Code provides:


Property of public ownership is
1. That devoted to public use, such as rivers; Since undeniably all beds of rivers are
of public ownership, it follows that the banks, which form part of them, are also of public
ownership.
Art. 372 of the old Civil Code which provides that:
Whenever a navigable or floatable river changes its course from natural causes and opens
a new bed through a private estate, the new bed shall be of public ownership

Defendants were extracting from the public domain under proper authorization and did not
extract materials from plaintiff's property.
CHUN 21

21.REPUBLIC VS. PARAAQUE,


FACTS:
This is a petition for review on certiorari under Rule 45 of the
1997 Rules of Civil Procedure, on pure questions of law, assailing
the January 8, 2010 Order of the Regional Trial Court, Branch
195, Paranaque City (RTC), which ruled that petitioner Philippine
Reclamation Authority (PRA) is a government owned and
controlled corporation (GOCC), a taxable entity, and, therefore,
not exempt from payment of real property taxes.
ISSUE:W/N Reclaimed Lands are exempt from Real Estate
Tax?
RULING:The Court agrees with PRA that the subject reclaimed
lands are still part of the public domain, owned by the State and,
therefore, exempt from payment of real estate taxes. Here, the
subject lands are reclaimed lands, specifically portions of the
foreshore and offshore areas of Manila Bay. As such, these lands
remain public lands and form part of the public domain.
CHUN 22
22. Yu Chang v. Republic, G.R. No. 171726, Feb. 23, 2011

Facts: Petitioners inherited some parcel of lots from their deceased father and built on it buildings
for commercial and residential use. They then proceeded to have the land registered under the
Property Registration Decree. The state opposed the registration as it claimed that said lots being
forest land are part of the inalienable property of the public domain. Petitioners contended that it
should no longer be considered and forest land as they have already built buildings for residential
or commercial use on the lots.

Issue: Whether or not said parcels of lots are inalienable property of the public domain.

Held: The classification of land is descriptive of its legal nature or status and does not have to be
descriptive of what the land actually looks like. The fact that the area within which the subject
parcels of land are located is being used for residential and commercial purposes does not serve to
convert the subject parcels of land into agricultural land. It is fundamental that before any land may
be declassified from the forest group and converted into alienable or disposable land for
agricultural purposes, there must be a positive act from the government.

Here, petitioners failed to present well-nigh incontrovertible evidence necessary to prove their
compliance of the requirements under Section 48(b) of C.A. No. 141.
Constantino -23

23. Laurel vs Garcia

187 SCRA 797

Facts:

The subject Roppongi property is one of the properties acquired by the


Philippines from Japan pursuant to a Reparations Agreement. The property is where the
Philippine Embassy was once located, before it transferred to the Nampeidai
property. It was decided that the properties would be available for sale or
disposition. One of the first properties opened up for public auction was
the Roppongi property, despite numerous oppositions from different sectors.

Held:

The Roppongi property were assigned to the government


sector and that it was specifically designated under the agreement to house the
Philippine embassy. The fact that the Roppongi site has not been used for a long time
foractual Embassy service doesnt automatically convert it to patrimonial property. Any
such conversion happens only if the property is withdrawn from public use. A property
continues to be part of the public domain, not available for
private appropriation or ownership until there is a formal declaration on the part of the
government to withdraw it from being such.
Custodio 24

DepEd V. MARIANO TULIAO


G.R. No. 205664 June 9, 2014

FACTS: Tuliao filed an action for recovery of possession and removal of structure with damages against
DepEd with the MTCC, Tuguegarao City. He alleged that he was the registered owner of the subject land
and that a portion of such was allowed by his predecessors-in-interest to be used by the Atulayan
Elementary School (AES) as an access road for the schoolchildren in going to and from the school. In
March 2000, he demanded that the DepED vacate the property. The respondent, however, refused. Tuliao
likewise demanded payment for reasonable rent, but his demand was also ignored.

In its defense, the DepEd denied the material allegations of the complaint and averred that it did
not state a cause of action. Even if there was, the same was already barred by prescription and/or laches.

MTCC ruled in favor of Tuliao and thus had a right of action against the holder and possessor of
the said property. Further, it found that respondents possession of the subject property was merely
tolerated by Tuliao. For said reason, his right to recover it was never barred by laches. On appeal to the
RTC, said court dismissed the appeal and affirmed the MTCC decision. DepEd elevated the case to the
CA via a petition. Finding no merit, the CA affirmed the RTC decision.

ISSUES: WON DepEd had sufficient evidence to prove that it had the right to possession of the land in
question

RULING/DOCTRINE:

No it did not. The Court declared that in actions for recovery of possession, the plaintiff must
show proof to support his claim of his right to possession of the property. The defendant in turn must
show proof to controvert the plaintiffs claim; otherwise the court will rule for the plaintiff. Thus, when a
landowner filed an action for recovery of possession against a public school which built a gymnasium on
a parcel of land which the owner allowed the school to use as an access road for the schoolchildren, and
the plaintiff showed as evidence tax declarations and a certificate of title over the property, the lone
testimonial evidence the DepEd presented is not sufficient to controvert the landowners case.
DABA 25
Heirs of Yabao v. Paz Lentejas Van Der Kolk

GR No. 207266, June 25, 2014

Facts: The heirs of Yabao alleged in their complaint that they are the sole surviving heirs of the late spouses
Yabao, and that they are the absolute co-owners of the parcel of land declared in the name of the late
Paciano Yabao. The Heirs of Yabao prayed that they be declared the co-owners and possessors of the
subject property occupied and possessed by Paz Lentejas Van Der Kolk (Van Der Kolk), who also asserted
claim of ownership over the same.

Van Der Kolk started possessing the land in 1996 and refused to vacate the same despite demands
from the said heirs. Also, she filed a petition for original patent regarding such land. This was timely oppsed
by the heirs, according to them.

Aside from these material averments in the complaint, nothing else was presented to prove the
heirs' right over the subject property.

Issue: Whether or not the Heirs of Yabao are the rightful owners of the subject property.

SC: No.

Ownership by the heirs cannot be established by mere lip service and bare allegations in the
complaint. As in all matters, a party must establish his/her averments in the complaint by sufficient
evidence necessary to prove such claim.

In the case at bench, the heirs of Yabao merely alleged that they are the heirs of Paciano Yabao
without presenting any proof why they are the latters heirs and in what degree or capacity.

The basis of the heirs claim of ownership was a mere tax declaration that was supposedly in the
name of their putative ancestor Paciano Yabao. However, a tax declaration is not a proof of ownership; it
is not a conclusive evidence of ownership of real property. In the absence of actual, public, and adverse
possession, the declaration of the land for tax purposes does not prove ownership. It can only be a strong
indication of ownership if coupled with possession.

In the case at bench, it was Paz Lentejas who was in possession of the property and not the heirs of
Yabao. Consequently, the tax declaration, standing alone, is not an acceptable proof of ownership.
DADANG 26

Midway Maritime and Technological Foundation, petitioner v. Marissa Castro


GR. No. 189061 August 6,2014

Facts: In a mortgage over a parcel of land a contract of a 15 year lease was


entered in favor of Castro who later built a residential building. The mortgage
was later foreclosed and the parcel of land was then sold in a public auction.
After the lapsed of the redemption period, the bank then sold it to Cloma.
Cloma leased the property with herein petitioner which also leased the
residential building owned by the Castro with payment of monthly rentals.
Cloma then sold the property to her daughter which is the wife of the
President of herein petitioner. Petitioner now files an action for ownership
and recovery of possession.
Issue: Whether or not there was a lease agreement as regards to the
residential building?
Held: Yes, it is settled that once a contract of lease is shown to exist between
the parties, the lessee cannot by any proof, however strong, overturn the
conclusive presumption that the lessor has a better right of possession to the
subject premises. The payment of rentals confirms the existence of the lease
agreement.
DARIMBANG 27
27. Subic Bay Legend Resorts and Casinos, Inc. vs Bernard Fernandez
September 29, 2014
G.R. No. 193426

Facts:
On July 1, 1994, Bernard Fernandez, brother of Ludwig and Deoven, filed a complaint for the
recovery of sum of money and damages against Subic Bay Legend Resorts and Casinos, Inc (SBL).
According to him, he went to the casino on June 13, 1997 and handed to his brothers $6,000.00
worth of chips belonging to him, for use at the casino. Thereat, the company personnel accosted his
brothers and confiscated his casino chips worth $5,900.00. The company refused to return the
same despite their demand. Ludwig and Deoven were accused of stealing casino chips from SBL.
They were made to confess that the chips were supplied by a casino employee, Michael Cabrera.

Issue: Whether or not Bernard is the lawful possessor of the casino chips, entitling him to collect
from the casino and award damages.

Ruling:
No. There is no basis to suppose that the casino chips found in Ludwigs and Deovens
possession were stolen. Unless the independent fact that Cabrera stole the chips can be proved, it
cannot be said that the chips must be confiscated when found to be in the brothers possession.
Though casino chips do not constitute legal tender, there is no law which prohibits their use
or trade outside of the casino which issues them. Since casino chips are considered to have been
exchanged with their corresponding representative value, it is with more reason that the Court
should require SBL to prove that the chips confiscated were indeed stolen from them. If SBL cannot
prove its loss, then Art. 559 cannot apply, and the presumption that the chips were exchanged for
value remains.
DATUDACULA 28

Case 28: G.R. No. 199448, 12 November 2014


ROLANDO S. ABADILLA, JR., Petitioner, v. SPOUSES BONIFACIO P. OBRERO AND
BERNABELA N. OBRERO, Respondents.

FACTS: Respondents initiated a case for forcible entry against petitioner. Respondents
claimed that they are the registered owners of the land in question based on a TCT issued
to them. They claimed that they were in possession thereof based on improvements
erected therein utilized for residential and business purposes prior to the alleged acts of
petitioner who forcibly fenced the perimeter of the land with barbed wire.
In his answer, petitioner denied the acts imputed upon him and claimed that the land was
sold by respondents to his late father as evidenced by a Deed of Absolute Sale. Being one of
the heirs, he is one of the owners thereof. In fact, they left a caretaker to oversee the land.
Despite the sale, respondents supposedly attempted to remove the fence and even built
concrete structures on the land using it for dwelling purposes.
The MTCC dismissed the complaint and held that respondent Bonifacio's admission
confirmed that he and his wife indeed sold the land to Abadilla, Sr. Thus, ownership and
possession of the land was transferred to him and then to the petitioner and his co-heirs.
The respondents appealed the decision to the RTC and it reversed and set aside the
decision of MTCC. The RTC found that the Deed of Absolute Sale between the respondents
and Abadilla, Sr. was of no force and effect for lack of consideration.
The CA affirmed RTCs decision and dismissed the petitioners appeal.

ISSUE/s: 1.) Whether or not an ejectment proceeding is proper in the case at bar.
2.) Whether or not respondents are entitled to lands possession.

HELD: 1.) Yes. The Supreme Court ruled that ejectment proceedings are summary
proceedings intended to provide an expeditious means of protecting actual possession or
right to possession of property. Title is not involved. Issues as to ownership are not
involved in the action; evidence thereon is not admissible, except only for the purpose of
determining the issue of possession. Thus, where the parties to an ejectment case raise the
issue of ownership, the courts may pass upon that issue but only to determine who
between the parties has the better right to possess the property.
2.) As between the petitioner's Deed of Absolute Sale and the respondents' TCT, the
latter must prevail. A certificate of title is evidence of indefeasible and incontrovertible title
to the property in favor of the person whose name appears therein. A title issued under the
Torrens system is entitled to all the attributes of property ownership, which necessarily
includes possession. Hence, as holders of the Torrens title over the subject land, the
respondents are entitled to its possession.
DEL CASTILLO -29
HEIRS OF EUGENIO LOPEZ VS QUERUBIN

Facts:

Alfonso Sandoval and Roman Ozaeta, Jr. filed an Application for Registration of Title for two parcels of land.
The CFI granted the application for registration and declared the applicants and their respective wives as the true and
absolute owners in equal pro-indiviso shares of the said parcels of land. After the Court adjudicated the subject
properties in favor of the applicants and before the decrees of registration were actually issued, said applicants sold
the subject properties to Eugenio Lopez. In the Deed of Absolute Sale, the vendors-applicants obligated themselves
to file in the land registration case the necessary motion or manifestation in order that the certificates of title will be
issued in the name of Eugenio Lopez or his successors-in-interest. Unfortunately, this obligation was not complied
with for so many years. Upon learning of this fact, the Lopez heirs filed their Motion in the land registration case. Said
motion contained the Deed of Absolute Sale and prayed that the decrees of registration and original certificate of
titles be declared null and void, and the subject properties be issued in their names.

Issues:

1. W/n petitioners are properly parties-litigants in the land registration proceedings

2. W/n the motion dated Nov. 25, 1998 is proper for purposes of impugning the questioned decrees and the
corresponding original certificates of title

Ruling:

On the preliminary issue of standing, the Court ruled that Lopez heirs did not acquire legal standing as
parties-litigants in the land registration proceedings. However, petitioners undeniably have a stake in the outcome of
the pending litigation over the subject properties. However, petitioners undeniably have a stake in the outcome of the
pending litigation over the subject properties.

Section 22 of Presidential Decree No. 1529 expressly allows the disposition of lands subject matter of a
registration proceeding and the subsequent registration thereof in the name of the person to whom the land was
conveyed. As required by the law, the pertinent instruments of conveyance must be presented to the court and that
prior notice is given to the parties in the land registration case. After said requisites are complied with, the court shall
either order the land registered subject to the conveyance or encumbrance, or order that the decree of registration be
issued in the name of the person to whom the property was conveyed. Contrary to the Lopez heirs' position, nowhere
can it be inferred in Section 22 of Presidential Decree No. 1529 that the buyer of the property automatically becomes
a party to the land registration case after complying with the requirements of the aforesaid provision of law.
Nonetheless, the same provision of law authorizes the land registration court to accord certain reliefs in favor of the
person to whom the property has been conveyed.

With respect to the second issue invoked by the Lopez heirs, the Court held that the Court of Appeals was
not entirely incorrect in ruling that the Lopez heirs availed themselves of the wrong remedy in impugning Decree of
Registrations and Original Certificate of Titles. Court of Appeals adjudged that the Lopez heirs' Motion was a
collateral attack on the certificates of title covering the subject properties. The Court of Appeals, however, overlooked
the fact that the Lopez heirs never attacked the Decision the then CFI of Rizal in the Land Registration Case. When
the decrees of registration were still issued in the names of said original applicants, due to peculiar circumstances
that occurred outside the proceedings in the land registration court, petitioners were unjustly deprived of the
opportunity to enforce the remedy accorded to them under Section 22 of Presidential Decree No. 1529. Under the
highly exceptional circumstances of this case, we hold that the land registration court did not necessarily lose
jurisdiction over the case notwithstanding the improvident issuance of the decrees of registration by the LRA. To
rectify their situation, the Lopez heirs could have resorted to Section 108 of Presidential Decree No. 1529 in order to
correct the palpable mistakes in the certificates of title in this case. Accordingly, petitioners may avail of the remedy
provided under Section 108 of PD 1529. Thus, this case is remanded to the land registration court for further
proceedings, subject to the Lopez heirs' compliance with the requisites of Section 108 of Presidential Decree No.
1529.
DURIAS 30
GR No. 152141 August 8, 2011
DEL FIERRO vs SEGUIRAN
FACTS:
The Complaint alleged that plaintiffs (petitioners) were the owners and possessors of a parcel of
land identified as Lot Nos. 1625 and 1626, formerly part of Lot No. 1197, situated at Barangay
Locloc, Palauig,Zambales. On July 26, 1964, Lodelfo and Narciso Marcial unlawfully entered the land
occupied by plaintiffs. Plaintiffs sued them for forcible entry before the Municipal Court of Palauig.
The municipal court ruled in favor of plaintiffs, which decision was affirmed on appeal by the CFI of
Iba, Zambales, Branch II on August 1, 1973.Consequently, Lodelfo and Narciso Marcial were ejected
from the premises.
Meanwhile, on June 29, 1964, Marcial had mortgaged the lots to the Rural Bank of San Marcelino,
Inc., which foreclosed the real estate mortgage on December 26, 1972, and consolidated ownership
over the lots on April 22, 1982. On October 28, 1981, defendant Rene S. Seguiran purchased from
Lodelfo Marcial (deceased) the subject lots. On November 9, 1981, defendant purchased the subject
lots again from the Rural Bank of San Marcelino, Inc. Moreover, defendant, with evident bad faith,
fraudulently applied with the Bureau of Lands for a free patent over the said lots, alleging that he
was the actual possessor thereof, which constitutes a false statement, since the plaintiffs were the
ones in actual possession.
Defendant prayed that judgment be rendered by the trial court dismissing the complaint and
ordering plaintiffs to pay him actual, moral and exemplary damages as well as attorneys fees and
the expenses of litigation. The trial court held that plaintiffs (petitioners) failed to prove the identity
of the property sought to be recovered.
ISSUE: Whether petitioners are entitled to reconveyance of Lot Nos. 1625 and 1626

HELD: NO. Article 434 of the Civil Code provides that to successfully maintain an action to recover
the ownership of a real property, the person who claims a better right to it must prove two (2)
things: first, the identity of the land claimed; and second, his title thereto. In regard to the first
requisite, in an accion reinvindicatoria, the person who claims that he has a better right to the
property must first fix the identity of the land he is claiming by describing the location, area and
boundaries thereof. Anent the second requisite, i.e., the claimant's title over the disputed area, the
rule is that a party can claim a right of ownership only over the parcel of land that was the object of
the deed. In this case, petitioners failed to prove the identity of the parcels of land sought to be
recovered and their title thereto.
EBDAO 31
Del Rosario v. Gerry Roxas Foundation, Inc. G.R. No. 170575.
June 8, 2011. First Division; Del Castillo, J.

Facts: Manuel del Rosario appears to be the registered owner of Lot 3-A of Psd-301974
located in Roxas City which is described in and covered by a Transfer Certificate of Title (TCT).
Sometime in 1991, the Gerry Roxas Foundation, Inc., (GRFI) as a legitimate foundation, took
possession and occupancy of said land by virtue of a memorandum of agreement it entered with
Roxas City. Its possession and occupancy of said land is in the character of being a lessee thereof.
In February and March 2003, the Spouses Manuel and Florentina del Rosario (Spouses) served
notices upon the GRFI to vacate the premises of said land. GRFI, however, did not heed such
notices because it still has the legal right to continue its possession and occupancy of said land. On
July 2003, the spouses filed a Complaint for Unlawful Detainer against GRFI before the Municipal
Trial Court in Cities. Said complaint contains, among others, the following significant allegations: 3.
Plaintiffs are the true, absolute and registered owners of a parcel of land, situated at Dayao, Roxas
City and covered by and described in TCT No. 18397 issued to the plaintiffs by the Register of
Deeds for Roxas City. 4. Sometime in 1991, without the consent and authority of the plaintiffs,
defendant took full control and possession of the subject property, developed the same and used it
for commercial purposes. 7. Plaintiffs have allowed the defendant for several years, to make use of
the land without any contractual or legal basis. Hence, defendants possession of the subject
property is only by tolerance. 8. But plaintiffs patience has come to its limits. Hence, sometime in the
last quarter of 2002, plaintiffs made several demands upon said defendant to settle and/or pay
rentals for the use of the property. 10. Notwithstanding receipt of the demand letters, defendant
failed and refused, as it continues to fail and refuse to pay reasonable monthly rentals for the use
and occupancy of the land, and to vacate the subject premises despite the lapse of the fifteen-day
period specified in the said demand letters. Consequently, defendant is unlawfully withholding
possession of the subject property from the plaintiffs, who are the owners thereof.
Issue: Whether or not the allegations in the Complaint establish a cause of action for forcible
entry and not an unlawful detainer?
Held: Yes. In forcible entry, one is deprived of physical possession of any land or building by
means of force, intimidation, threat, strategy, or stealth. Where the defendants possession of the
property is illegal ab initio, the summary action for forcible entry (detentacion) is the remedy to
recover possession. In their Complaint, the spouses maintained that the GRFI took possession and
control of the subject property without any contractual or legal basis. Assuming that these allegations
are true, it hence follows that GRFIs possession was illegal from the very beginning. Therefore, the
foundation of the spouses Complaint is one for forcible entry that is, the forcible exclusion of the
original possessor by a person who has entered without right. Thus, there can be no tolerance as
the spouses alleged that GRFIs possession was illegal at the inception. Corollarily, since the
deprivation of physical possession, as alleged in the spouses Complaint was attended by strategy
and force, this Court finds that the proper remedy for the spouses was to file a Complaint for Forcible
Entry and not the instant suit for unlawful detainer
FELICIANO 32
GACUS 33

Bienvenido Barrientos, petitioner


-versus-
Mario Rapal, respondent

G.R. No. 169594 20 July 2011


FACTS:
Mario Rapal acquired a parcel of land from Antonio Natavio via a notarized Deed of Transfer of
Possessory Right and thereafter built a semi-concrete house. Respondent allowed Bienvenido
Barrientos and his family to stay on the subject land as caretakers on the condition that the latter
shall vacate the premises when the former needs the property. However, when respondent
eventually demanded on several occasions that the petitioner leave the property, the latter refused.
Rapal was prompted to file a case for Unlawful Detainer against Barrientos. The MeTC decided in
favor of petitioner but the RTC reversed the decision reasoning that respondent has not shown any
prior lawful possession of the property in question. Upon appeal, CA reversed the RTCs decision
and reinstated the MeTCs decision. The CA determined who between the parties was first in
possession and concluded that respondent was, indeed, first in possession of the lot.

ISSUE: Whether petitioner has the better right over the subject property.

HELD:
No. The respondent clearly makes out a case for unlawful detainer, since petitioner's occupation of
the subject property was by mere tolerance. A person who occupies the land of another at the
latter's tolerance or permission, without any contract between them, is necessarily bound by an
implied promise that he will vacate the same upon demand, failing which a summary action for
ejectment is the proper remedy against them.
In an unlawful detainer case, the sole issue for resolution is physical or material possession
of the property involved, independent of any claim of ownership by any of the parties. Where the
issue of ownership is raised by any of the parties, the courts may pass upon the same in order to
determine who has the right to possess the property. In the case at bar, both petitioner and
respondent were claiming ownership over the subject property. Hence, the CA correctly touched
upon the issue of ownership only to determine who between the parties has the right to possess the
subject property.
GALES 34

GERMAN MANAGEMENT & SERVICES, INC. V COURT OF APPEALS


177 SCRA 495 (1989)

FACTS:
Spouses Jose issued a power of attorney in favor of petitioner for the
development of their parcel of land into a subdivision. Private respondents
were occupying the land and petitioner advised them to vacate but they
refused. Thereafter, petitioner continued their development and
construction. Respondents then filed a case for forcible entry. The trial court
dismissed the complaint and this was reversed by the CA.

ISSUE:
Whether or not private respondents are entitled to file a forcible entry case against
petitioner?

HELD:
Notwithstanding petitioners claim that it was duly authorized by the
owners to develop the subject property, private respondents as actual possessors,
can commence a forcible entry case against petitioner because ownership is not in
issue. Forcible entry is merely a quieting process, and never determines the actual title
to an estate. Title is not involved.

Although admittedly petitioner may validly claim ownership based on the


muniment of title it presented, such evidence doesnt responsively address the issue of
prior actual possession raised in a forcible entry case. It must be stated that regardless
of the actual condition of title to the property, the party in a peaceable quiet possession
shall not be turned out by a strong hand, violence or terror. Thus, a party who can
prove prior possession can recover such possession even against the owner
himself. Whatever may be the character of his prior possession, if he has in favor
priority in time, he has security that entitles him to remain on the property until he has
been lawfully ejected by a person having a better right by accion publiciana or accion
reivindicatoria.
GARCES 35
REPUBLIC VS CA
FACTS: These cases arose from the application for registration of a parcel of land filed on February
11, 1965, by Jose de la Rosa on his own behalf and on behalf of his three children, Victoria,
Benjamin and Eduardo. The land, situated in Tuding, Itogon, Benguet Province, was divided into 9
lots and covered by plan Psu-225009. According to the application, Lots 1-5 were sold to Jose de la
Rosa and Lots 6-9 to his children by Mamaya Balbalio and Jaime Alberto, respectively, in 1964.

The application was separately opposed by Benguet Consolidated, Inc. as to Lots 1-5, Atok Big
Wedge Corporation, as to Portions of Lots 1-5 and all of Lots 6-9, and by the Republic of the
Philippines, through the Bureau of Forestry Development, as to lots 1-9.
In support of the application, both Balbalio and Alberto testified that they had acquired the subject
land by virtue of prescription Balbalio claimed to have received Lots 1-5 from her father shortly
after the Liberation.
Benguet opposed on the ground that the June Bug mineral claim covering Lots 1-5 was sold to it on
September 22, 1934, by the successors-in-interest of James Kelly, who located the claim in
September 1909 and recorded it on October 14, 1909. From the date of its purchase, Benguet had
been in actual, continuous and exclusive possession of the land in concept of owner, as evidenced
by its construction of adits, its affidavits of annual assessment, its geological mappings, geological
samplings and trench side cuts, and its payment of taxes on the land.
For its part, Atok alleged that a portion of Lots 1-5 and all of Lots 6-9 were covered by the Emma
and Fredia mineral claims located by Harrison and Reynolds on December 25, 1930, and recorded
on January 2, 1931, in the office of the mining recorder of Baguio. These claims were purchased
from these locators on November 2, 1931, by Atok, which has since then been in open, continuous
and exclusive possession of the said lots as evidenced by its annual assessment work on the claims,
such as the boring of tunnels, and its payment of annual taxes thereon.
The Bureau of Forestry Development also interposed its objection, arguing that the land sought to
be registered was covered by the Central Cordillera Forest Reserve under Proclamation No. 217
dated February 16, 1929. Moreover, by reason of its nature, it was not subject to alienation under
the Constitutions of 1935 and 1973.
The trial court denied the application, holding that the applicants had failed to prove their claim of
possession and ownership of the land sought to be registered.
The applicants appealed to the respondent court, which reversed the trial court and recognized the
claims of the applicant, but subject to the rights of Benguet and Atok respecting their mining claims.
In other words, the Court of Appeals affirmed the surface rights of the de la Rosas over the land
while at the same time reserving the sub-surface rights of Benguet and Atok by virtue of their
mining claims. Both Benguet and Atok have appealed to this Court, invoking their superior right of
ownership.
ISSUE: Whether respondent courts decision, i.e. the surface rights of the de la Rosas over the land
while at the same time reserving the sub-surface rights of Benguet and Atok by virtue of their
mining claim, is correct.
HELD: No. The Court held that Benguet and Atok have exclusive rights to the property in question
by virtue of their respective mining claims which they validly acquired before the Constitution of
1935 prohibited the alienation of all lands of the public domain except agricultural lands, subject to
vested rights existing at the time of its adoption. The land was not and could not have been
transferred to the private respondents by virtue of acquisitive prescription, nor could its use be
shared simultaneously by them and the mining companies for agricultural and mineral purposes.
The Court feels that the rights over the land are indivisible and that the land itself cannot be half
agricultural and half mineral. The classification must be categorical; the land must be either
completely mineral or completely agricultural. In the instant case, as already observed, the land
which was originally classified as forest land ceased to be so and became mineral and completely
mineral once the mining claims were perfected. As long as mining operations were being
undertaken thereon, or underneath, it did not cease to be so and become agricultural, even if only
partly so, because it was enclosed with a fence and was cultivated by those who were unlawfully
occupying the surface.
This is an application of the Regalian doctrine which, as its name implies, is intended for the benefit
of the State, not of private persons. The rule simply reserves to the State all minerals that may be
found in public and even private land devoted to agricultural, industrial, commercial, residential or
(for) any purpose other than mining. Thus, if a person is the owner of agricultural land in which
minerals are discovered, his ownership of such land does not give him the right to extract or utilize
the said minerals without the permission of the State to which such minerals belong.
The flaw in the reasoning of the respondent court is in supposing that the rights over the land could
be used for both mining and non-mining purposes simultaneously. The correct interpretation is
that once minerals are discovered in the land, whatever the use to which it is being devoted at the
time, such use may be discontinued by the State to enable it to extract the minerals therein in the
exercise of its sovereign prerogative. The land is thus converted to mineral land and may not be
used by any private party, including the registered owner thereof, for any other purpose that will
impede the mining operations to be undertaken therein, For the loss sustained by such owner, he is
of course entitled to just compensation under the Mining Laws or in appropriate expropriation
proceedings.
GERMANES 36
NPC v. Ibrahim

Syllabus. Eminent Domain

The NPC constructed underground tunnels on the property of the respondents without their
knowledge and consent and without any expropriation proceeding. It contended that it constructed
an easement on the property. Was there taking of the property considering that the owners were
deprived of their beneficial use and enjoyment of the same, hence, entitled to just compensation?

Ruling. Yes. The manner in which the easement was created by the NPC, violated the due process rights
of the owners as it was without notice and indemnity to them and did not go through proper
expropriation proceedings. NPC could have, at any time, validly exercised the power of eminent domain
to acquire the easement over the property as this power encompasses not only the taking or
appropriation of title to and possession of the expropriated property but likewise covers even the
imposition of a mere burden upon the owner of the condemned property.

The owners are clearly entitled to the payment of just compensation. Notwithstanding the fact that NPC
only occupied the sub-terrain portion, it is liable to pay not merely an easement fee but rather the full
compensation for the land. This is so because; the nature of the easement practically deprived the
owners of its normal beneficial use.

Syllabus. Valuation of the property.

NPC contended that if ever it is liable, it should be made to pay the value of the land from the time it
constructed the tunnels. Is the contention correct? Why?

Ruling. No. To it to use the date it constructed the tunnels as the date of valuation would be grossly
unfair. First, it did not enter the land under warrant of legal authority or with intent to expropriate the
same. It did not notify the owners and wrongly assumed that it had the right to dig the tunnels under
their property. Secondly, the improvements introduced in no way contributed to an increase in the
value of the land. The valuation should be based at the time of the discovery of the construction of the
underground tunnels.
IBANEZ 37
PALERO-TAN vs. URDANETA

Facts:
Urdaneta found jewelry inside the RTC office. Tan is the owner of the jewelry.
Urdaneta did not return the jewelry instead he threw it away when his wife suspected that
we bought the jewelry for a mistress. However, it was found out that he knew that Tan had
lost the jewelry and that he did not bother to inform her.

Issues:
Whether or not Urdaneta can be faulted for throwing away the jewelry he found.

Held:
Yes, he can be faulted.

When a person who finds a thing that has been lost or mislaid by the owner takes the thing
into his hands, he acquires physical custody only and does not become vested with legal
possession. In assuming such custody, the finder is charged with the obligation of restoring
the thing to its owner. It is thus respondents duty to report to his superior or his
officemates that he found something. The Civil Code, in Article 719, explicitly requires the
finder of a lost property to report it to the proper authorities, thus:

Article 719: Whoever finds a movable, which is not treasure, must return it to its previous
possessor. If the latter is unknown, the finder shall immediately deposit it with the mayor of
the city or municipality where the finding has taken place.

The finding shall be publicly announced by the mayor for two consecutive weeks in the way
he deems best.

If the movables cannot be kept without deterioration, or without the expenses which
considerably diminish its value, it shall be sold at public auction eight days after the
publication.

Six months from the publications having elapsed without the owner having appeared, the
thing found, or its value, shall be awarded to the finder. The finder and the owner shall be
obliged, as the case may be, to reimburse the expenses.
JALA 38

G.R. No. 190106


January 15, 2014

MAGDALENA T. VILLASI v. FILOMENO GARCIA substituted by his heirs,


namely, ERMELINDA H. GARCIA, LIZA GARCIA-GONZALEZ, THERESA
GARCIA-TIANGSON, MARIVIC H. GARCIA, MARLENE GARCIA-MOMIN,
GERARDO H. GARCIA, GIDEON H. GARCIA and GENEROSO H. GARCIA, and
ERMELINDA H. GARCIA

Facts:
Villasi engaged the services of respondent Fil-Garcia Construction, Inc. (FGCI)
to construct a seven-storey condominium building located Cubao, Quezon City.
For failure of Villasi to fully pay the contract price despite several demands,
FGCI initiated a suit for collection of sum of money. Villasi filed an answer
specifically denying the material allegations of the complaint. Contending that
FGCI has no cause of action against her, Villasi averred that she delivered the
total amount of P 7,490,325.10 to FGCI but the latter accomplished only 28%
of the project. To enforce her right as prevailing party, Villasi filed a Motion for
Execution. To satisfy the judgment, the sheriff levied on a building located
Kalayaan Avenue, Quezon City. While the building was declared for taxation
purposes in the name of FGCI, the lots in which it was erected were registered
in the names of the Spouses Garcia. The Spouses Garcia argued that the
building covered by the levy was mistakenly assessed by the City Assessor in
the name of FGCI and that it could not be levied upon not being owned by the
judgment debtor.

The spouses Garcia filed an Affidavit of Third Party Claim and a Motion to Set
Aside Notice of Sale on Execution, claiming that they are the lawful owners of
the property.

The RTC hereby orders Deputy Sheriff Angel Doroni to suspend or hold in
abeyance the conduct of the sale on execution of the buildings levied upon by
him, until further orders from the Court.

Villasi timely filed a Petition for Certiorari before the CA. The petition was
dismissed.

Villasi filed a petition for review on certiorari pursuant to Rule 45 of the


Revised Rules of Court, assailing the decision rendered by the CA.

Issue:
WON the general rule on accession can be applied in the case at bar.
Held:
No. While it is a horn-book doctrine that the accessory follows the principal,
that is, the ownership of the property gives the right by accession to everything
which is produced thereby, or which is incorporated or attached thereto, either
naturally or artificially, such rule is not without exception. In cases where
there is a clear and convincing evidence to prove that the principal and the
accessory are not owned by one and the same person or entity, the
presumption shall not be applied and the actual ownership shall be upheld.

When there are factual and evidentiary evidence to prove that the building and
the lot on which it stands are owned by different persons, they shall be treated
separately. As such, the building or the lot, as the case may be, can be made
liable to answer for the obligation of its respective owner.

The SC granted the petition. The assailed Decision and Resolution of the Court
of Appeals are hereby REVERSED and SET ASIDE. The Deputy Sheriff is
hereby directed to proceed with the conduct of the sale on execution of the
levied building.
JALAGAT 39

BPI vs Sanchez
Case Digest GR 179518 Nov 19 2014

Facts:
The Sanchezes entered into an agreement with Garcia (doing business in the name of TSEI) to sell
for P 1.850 million their parcel of land, with an earnest money of 50k. They agreed that Garcia shall
pay the purchase price in cash once the property is vacated. The Sanchezes entrusted to Garcia the
owners copy of TCT because it was agreed that he shall take care of all the documentations
necessary for the transaction.

Immediately after the property was vacated, Garcia took possession and began constructing
townhouses thereon without the Sanchezes knowledge and consent. While these developments
were ongoing, Garcia failed to pay the purchase price. Subsequently, the Sanchezes were given six
checks representing the amount of the purchase price. Four of these checks were postdated, thus
further delaying their overdue payment. To properly document the check payments, they made an
agreement stipulating that if one of the checks were dishonored, the Sanchezes may rescind the
contract.

The last two checks were dishonored, so the Sanchezes rescinded the contract and demanded from
Garcia the return of the TCT. However, Garcia refused to return the documents and vacate the
property.

Meanwhile, the Sanchezes found out that Garcia/TSEI were selling townhouses situated in the
property. So they informed the HLURB, the City Building Official and the RD in Quezon City, of the
illegal constructions being made thereon. The HLURB issued a Cease and Decease Order enjoining
Garcia / TSEI from further developing and selling the townhouses. Such orders were left unheeded.
In fact, Garcia were already able to sell many of the units to different individuals and entities, and
even mortgaged the property. Consequently, the Sanchezes filed before the RTC a complaint for
rescission, restitution and damages with TRO.

The purchasers and mortgagee who are the intervenors in this case were found by the court to be in
bad faith. On the other hand, the Sanchezes were held to be in good faith and not negligent.

Issue 1: W/N rescission of the contract was barred by the subsequent transfer of the property
No. Under Article 1191 of the Civil Code, rescission is available to a party in a reciprocal obligation
where one party fails to comply with it. As an exception to this rule, Article 1385 provides that
rescission shall not take place if the subject matter of the prior agreement is already in the hands of
a third party who did not act in bad faith.
Here, the failure of Garcia/TSEI to pay the consideration for the sale of the property entitled the
Sanchezes to rescind the Agreement. And in view of the finding that the intervenors acted in bad
faith in purchasing the property from Garcia, the subsequent transfer in their favor did not and
cannot bar rescission.

Issue 2: W/N Article 449 450 of the Civil Code is applicable to the Sanchezes
Yes. Bad faith on the part of the purchasers leads to the application of Art 449-450.
Consequently, the Sanchezes have the following options: (1) acquire the property with the
townhouses and other buildings and improvements that may be thereon without indemnifying TSEI
or the intervenors; (2) demand from TSEI or the intervenors to demolish what has been built on
the property at the expense of TSEI or the intervenors; or (3) ask the intervenors to pay the price of
the land.

As such, the Sanchezes must choose from among these options within 30 days from finality of the
decision. Should the Sanchezes opt to ask from the intervenors the value of the land, the case shall
be remanded to the RTC for the sole purpose of determining the fair market value of the lot at the
time the same were taken from the Sanchezes. If the Sanchezes decide to appropriate the
townhouses, other structures and improvements as their own pursuant to Art 449, then the
intervenors-purchasers shall be ordered to vacate said premises within a reasonable time from
notice of the finality of the decision by the Sanchezes. They have a right to recover their investment
in the townhouses from Garcia and TSEI. If the Sanchezes do not want to make use of the
townhouses and improvements on the subject lot, then the purchasers can be ordered to demolish
said townhouses or if they dont demolish the same within a reasonable time, then it can be
demolished at their expense. On the 3rd option, if the Sanchezes do not want to appropriate the
townhouses or have the same demolished, then they can ask that the townhouse purchasers pay
to them the fair market value of the respective areas allotted to their respective townhouses subject
of their deeds of sale.
LADERA 40

40. FLOREZA VS. EVANGELISTA, 96 SCRA 130, February 21, 1980

Facts:
The Evangelistas were the owner of a residential lot in Rizal with an area of
204.08 sq.m. assessed at P410. They borrowed P100 from Floreza. Floreza
occupied the residential lot and built a house of light material (barong-barong) with
the consent of the Evangelistas. Additional loans were made by the Evangelistas.

Floreza demolished the house of light material and constructed one of strong
material. Floreza has not been paying any rentals since the beginning of their
transactions. Eventually, Evangelistas sold, with a right to repurchase within 6
years, their land to Floreza. 7 months before the expiry of the repurchase period,
the evangelistas were able to pay in full. Floreza refused to vacate the lot unless he
was first reimbursed for the value of the house he built.

Issue:
1. W/N Floreza was entitled to reimbursement of the cost of his house.
2. W/N his heirs who replaced Floreza should pay rental of the land.

HELD:
1. No. Issue of reimbursement is not moot because if Floreza has no right of
retention, then he must pay damages in the form of rentals. Art. 448 is
inapplicable because it applies only when the builder is in good faith where he
believed that he had a right to build. Art. 453 is also not applicable because it
requires both of the parties to be in bad faith. Neither Art. 1616 applicable
because Floreza is not a vendee a retro. The house was already constructed in
1945 in light materials, even before the pacto de retro was entered into in 1949.
Floreza cannot be classified as a builder in good faith nor a vendee a retro, who
made useful imporvements during the pacto de retro, he has no right to
reimbursement of the value of the house, much less to the retention of the
premisers until he is paid. He may, however, remove such improvements shoul
it be possible to do so without damage to the property.
2. Yes. From the time the redemption price was paid in January 3, 1955, Florezas
right to use the residential lot without rent ceased. He should be held liable for
damages in the form of rentals for the continued use of the lot for P10 monthly
from January 3, 1995 until the house was removed and the property vacated by
Floreza or his heir.
LAUT - CASE # 41
Mercado v. CA
G.R. No. L-44001. June 10, 1988.

TOPIC: Builder in Good Faith

FACTS:

The private respondents Bulaong Group, had for many years been individual lessees of stalls in the
public market of Baliuag, Bulacan; from 1956 to 1972. The market was destroyed by fire on February 17,
1956; the members of the Bulaong Group constructed new stalls therein at their expense; and they thereafter
paid rentals thereon to the Municipality of Baliuag.

In 1972, the members of the group sub-leased their individual stalls to other persons, referred to as
the Mercado Group. After the Mercado Group had been in possession of the market stalls for some months, as
sub-lessees of the Bulaong Group, the municipal officials of Baliuag cancelled the long standing leases of the
Bulaong Group and declared the persons comprising the Mercado Group as the rightful lessees of the stalls in
question, in substitution of the former.

The members of the Bulaong Group sued. They filed several individual complaints with the Court of
First Instance seeking recovery of their stalls from the Mercado Group as well as damages. Their theory was
anchored on their claimed ownership of the stalls constructed by them at their own expense, and their
resulting right, as such owners, to sub-lease the stalls, and necessarily, to recover them from any person
withholding possession thereof from them.

On October 24,1975, respondent Judge rendered a summary judgment in all the cases. It rejected the
claim of the Municipality of Baliuag that it had automatically acquired ownership of the new stalls
constructed after the old stalls had been razed by fire, declaring the members of the Bulaong Group to
be builders in good faith, entitled to retain possession of the stalls respectively put up by them until and
unless indemnified for the value thereof. The decision also declared that the Bulaong and Mercado Groups
had executed the subleasing agreements with full awareness that they were thereby violating Ordinance No.
14; they were thus in pari delicto, and hence had no cause of action one against the other and no right to
recover whatever had been given or demand performance of anything undertaken. The judgment therefore
decreed (1) the annulment of the leases between the Municipality and the individuals comprising the
Mercado Group (the defendants who had taken over the original leases of the Bulaong Group); and (2) the
payment to the individual members of the Bulaong Group (the plaintiffs) of the stated, adjudicated value of
the stalls, with interest.

The members of the Mercado Group are now before this Court on an appeal by certiorari, this time
timely taken, assailing the above rulings of the Court of Appeals.

ISSUE: WON the Bulaong group a builder in good faith.

HELD:

NO. The Bulaong group is not a builder in good faith. It was held that to be deemed a builder in good
faith, it is essential that a person assert title to the land on which he builds; i.e., that he be a possessor in
concept of owner, and that he be unaware that there exists in his title or mode of acquisition any flaw which
invalidates it.

Lessees cannot be considered builders in good faith (taken from Haystacks, by Berne
Guerrero)
The members of the Bulaong group were admittedly lessees of space in the public market; they
therefore could not, and in truth never did make the claim, that they were owners of any part of the land
occupied by the market so that in respect of any new structure put up by them thereon, they could be deemed
builders in good faith (in accordance with Article 526 of the Civil Code). To be deemed a builder in good faith,
it is essential that a person assert title to the land on which he builds; i.e., that he be a possessor in concept of
owner, and that he be unaware that there exists in his title or mode of acquisition any flaw which invalidates
it. It is such a builder in good faith who is given the right to retain the thing, even as against the real owner,
until he has been reimbursed in full not only for the necessary expenses but also for useful expenses. On the
other hand, unlike the builder in good faith, a lessee who makes in good faith useful improvements which are
suitable to the use for which the lease is intended, without altering the form or substance of the property
leased, can only claim payment of one-half of the value of the improvements or, should the lessor refuse to
reimburse said amount, remove the improvements, even though the principal thing may suffer damage
thereby.
LOPEZ 42
042 Balucanag v Fransisco [GR L-33422]
FACTS:
Mrs. Charvet leased to Richard Stohner a 177.50 square meter lot, wherein improvements
consisting of fillings on the land and construction of a house were made on said lot by the latter.
The lot was then sold to petitioner Rosendo Balucanag, who, through counsel, demanded Stohner to
vacate the premises due to failure to pay rent. Stohner, also through counsel, claimed that he was a
builder in good faith of the residential house constructed on the lot.
The City Court of Manila rendered judgment on the ejectment suit in favor of petitioner. However,
Judge Fransisco, who presided over the Court of First Instance of Manila where the appeal was
made, set the judgment aside saying that Stohner was indeed a builder in good faith, invoking Art.
448 and 546. Thus, Stohner cannot be ejected from the lot without being reimbursed for the value
of the improvements.
ISSUE:
W/N Stohner is a builder in good faith
HELD:
No. The Court stated that Art. 448, relied upon by respondent judge, applies only to a case where
one builds on land in the belief that he is the owner thereof and it does not apply where ones only
interest in the land is that of a lessee under a rental contract.
Also, as was decided by the Court in Lopez, Inc. v Phil. And Eastern Trading Co., Inc., a lessee cannot
be considered a possessor in good faith since he knows beforehand that he is not the owner of the
land, and cannot deny the ownership or title of his lessor.
NOTE:
#You can choose not to include the statements that follow.
The Court also stated that the law applicable to the case at bar would be Art. 1687.
CASE DIGEST 43 by: Raisa Macabando

[G.R. No. 149295. September 23, 2003]


PHILIPPINE NATIONAL BANK, petitioner, vs. GENEROSO DE JESUS

FACTS:

Respondent filed a complaint against petitioner before the Regional Trial Court of Occidental
Mindoro for recovery of ownership and possession, with damages, over the questioned
property. On 26 March 1993, he had caused a verification survey of the property and discovered
that the northern portion of the lot was being encroached upon by a building of petitioner to the
extent of 124 square meters. Despite two letters of demand sent by respondent, petitioner failed
and refused to vacate the area.

Petitioner asserted that when it acquired the lot and the building sometime in 1981 from then
Mayor Bienvenido Ignacio, the encroachment already was in existence and to remedy the
situation, Mayor Ignacio offered to sell the area in question (which then also belonged to
Ignacio) to petitioner at P100.00 per square meter which offer the latter claimed to have
accepted. The sale, however, did not materialize when, without the knowledge and consent of
petitioner, Mayor Ignacio later mortgaged the lot to the Development Bank of the Philippines.

The trial court decided the case in favor of respondent declaring him to be the rightful owner.
The Court of Appeals sustained the trial court.

ISSUE: W/N PETITIONER IS A BUILDER IN GOOD FAITH

HELD:

In reference to Article 448, et seq., of the Civil Code, a builder in good faith is one who, not
being the owner of the land, builds on that land believing himself to be its owner and unaware of
any defect in his title or mode of acquisition.

The essence of good faith lies in an honest belief in the validity of ones right, ignorance of a
superior claim, and absence of intention to overreach another. Applied to possession, one is
considered in good faith if he is not aware that there exists in his title or mode of acquisition any
flaw which invalidates it. Evidently, petitioner was quite aware, and indeed advised, prior to its
acquisition of the land and building from Ignacio that a part of the building sold to it stood on the
land not covered by the land conveyed to it.

Equally significant is the fact that the building, constructed on the land by Ignacio, has in
actuality been part of the property transferred to petitioner. Article 448, of the Civil Code refers
to a piece of land whose ownership is claimed by two or more parties, one of whom has built
some works (or sown or planted something) and not to a case where the owner of the land is
the builder, sower, or planter who then later loses ownership of the land by sale or otherwise for,
elsewise stated, where the true owner himself is the builder of works on his own land, the issue
of good faith or bad faith is entirely irrelevant.

In fine, petitioner is not in a valid position to invoke the provisions of Article 448 of the Civil
Code.

MACUMBAL #44: Javier vs Concepcion 94 scra 212


(petition for review by certiorari) ARTICLE 546 of Civil Code
FACTS: On October 17, 1959, herein respondents filed against petitioners an action for
reconveyance for a parcel of land with land improvements thereon known as Lot 12 and for an
accounting and recovery of the produce of the land from the time the latter took possession in
1945. Respondents allege that Lot 12 as part of Lot 6 covered by a TCT under their name. In their
Answer, petitioners denied such claim and pointed out that Lot 6 could not include Lot 12 because
there exists a big river more than 50 meters wide and more than 20 meters deep between the two
lots. Lot 6 is situated within Dolores, Quezon while Lot 12 is situated within the jurisdiction of
Candelaria, Quezon.
The RTC ruled in favor of the owners of Lot No. 6, herein respondents, also finding Javier to have
acted in bad faith when he filed an opposition for the registration of Lot No. 6.
Court of Appeals affirmed the decision of RTC- with a modification that Javier did not act with
evident bad faith in occupying the land in question. Motion for reconsideration was denied.
ISSUES
1. w/n there was fraud in the registration of the lot2. w/n the cause of action of the private
respondents has been barred by laches3. w/n the land should be reconveyed to petitioners4.
w/n respondents should pay for the improvements introduced by the petitioners on the land in
question
RULING: Decision of CA is AFFIRMED1. We found no trace of fraud and misrepresentation in the
procurement of the transfer
certificate of title. Fraud as a legal basis for review of a decree means actual or positive
fraud as distinguished from constructive or legal fraud. The decree of registration can no longer be
impugned on the ground of fraud, error or lack of notice to defendant, as more than one year has
already elapsed from the issuance of decree
2. Defense of laches will not apply. Here, there are no intervening rights of third persons which may
be affected or prejudiced by a decision directing the return of Lot No. 12 to plaintiffs- respondents.
3. The third assigned error does is merely a consequence of the first and second assigned errors.
The same is without merit.
4. As possessors in good faith, petitioners are entitled to the fruits received before their
possession was legally interrupted upon receipt of judicial summons in connection with the filing of
the complaint for reconveyance (but since records do not show the date, we look at the date when
they filed an Answer that is on November 11, 1959). Petitioners should also be refunded the
necessary and useful expenses, with the right to retain the land until reimbursed of the same,
pursuant to Article 546 of the Civil Code. Under the said provision, respondents have the option to
refund the amount of useful expenses or to pay the increase in value which the land may have
acquired by reason thereof. Petitioners shall be accountable for the fruits of subject property only
after 1959, not from 1945.. Respondents are entitled to the produce from November 11, 1959 to the
time possession is delivered to them.
MADRONO 45
MAGDAYAO 46
FACTS:
Eden Ballatan, together with other petitioners, is living in and registered owners of Lot No.
24. Respondent Winston Go is living in and registered owners of Lot No. 25 and 26. And Li Ching
Yao is living in and the registered owner of Lot. 27. The Lots are adjacent to each other.
When Ballatan constructed her house in her lot, she noticed that the concrete fence and side
pathway of the adjoining house of respondent Winston Go encroached on the entire length of the
eastern side of her property. She was informed by her contractor of this discrepancy, who then told
respondent Go of the same. Respondent, however, claims that his house was built within the
parameters of his fathers lot; and that this lot was surveyed by engineer Jose Quedding, the
authorized surveyor of Araneta Institute of Agriculture (AIA). Petitioner called the attention of AIA
on the matter and so the latter authorized another survey of the land by Engineer Quedding. The
latter then did the survey twice which led to the conclusion that Lots Nos. 25, 26 (owned by
respondent Go) and 27 (owned by Li Ching Yao) moved westward to the eastern boundary of Lot
24 (owned by petitioner Ballatan.) (it was later on discovered by the courts that Go encroached 42
square meters from the property of Ballatan and Yao encroached 37 square meters on Gos
property, all of which were in GOOD FAITH) Ballatan made written demands to the respondent to
dismantle and move their improvements and since the latter wasnt answering the petitioner filed
accion publiciana in court. Gos filed their Answer with Third-Party Complaint impleading as third
party defendants respondents Li Ching Yao, the AIA and Engineer Quedding.
RTC ruled in favor of the petitioner ordering respondent Go to demolish their
improvements and pay damages to Petitioner but dismissing the third-party complaint. CA affirmed
the dismissal of the third party-complaint as to AIA but reinstated the the complaint against Yao
and the Engineer. CA also affirmed the demolition and damages awarded to petitioner and added
that Yao should also pay respondent for his encroachment of respondent Gos property. Jose
Quedding was also ordered to pay attorneys fees for his negligence which caused all this fuzz.

ISSUE: What is the proper remedy in this situation (everyone was in good faith)?

RULING:
Art 448 is the proper remedy (Lower Courts are wrong in awarding the damages). It was
established in the case that the parties had no knowledge of the encroachment until Ballatan
noticed it there all of them were builders in Good faith. In that scenario they have two options. 1st
option is that the land owner will buy the improvements and the 2nd option is to oblige the
builders to buy the land given that the value of the land is not considerably more than the buildings
or tree; otherwise the owner may remove the improvements thereon.
The builder, planter or sower, however, is not obliged to purchase the land if its value is
considerably more than the building, planting or sowing. In such case, the builder, planter or sower
must pay rent to the owner of the land. If the parties cannot come to terms over the conditions of
the lease, the court must fix the terms thereof. The right to choose between appropriating the
improvement or selling the land on which the improvement of the builder, planter or sower stands,
is given to the owner. If the option chooses is to sell the lot, the price must be fixed at the prevailing
market value at the time of payment. Petitioner was given by SC 30 days to decide on what to do or
which right to exercise. Likewise, Go was also given time to do the regarding Yaos encroachment.
Engineer Quedding was still asked to pay attorneys fees.
MALINGIN 47
HEIRS OF EMILIANO NAVARRO
vs.
INTERMEDIATE APPELLATE COURT AND HEIRS OF SINFOROSO PASCUAL [G.R. No. 68166.
February 12, 1997]
The applicant owns the property immediately adjoining the land sought to be registered. The
Talisay and Bulacan rivers come from inland flowing downstream towards the Manila Bay. Between
the Talisay River and the Bulacan River is the property of applicants with both rivers acting as the
boundary to said land and the flow of both rivers meeting and emptying into the Manila Bay. The
subject land was formed at the tip or apex of appellants' land adjacent to the north in front of
Manila Bay adding thereto the land now sought to be registered.
ISSUE:
Whether or not the subject land deemed an accretion formed by the action of the two rivers that
naturally accrues in favor of the riparian owner or by the action of the sea as foreshore?
Ruling:
The land in questioned is of public domain as formed by the action of the sea. The alluvium is
deposited on the northern portion thereof which is adjacent to the Manila Bay. Accretion as a mode
of acquiring property under said Article 457, requires the concurrence of the following requisites:
(1) that the accumulation of soil or sediment be gradual and imperceptible;
(2) that it be the result of the action of the waters of the river (3) that the land where the accretion
takes place is adjacent to
the bank of the river.Clearly lacking, thus, is the third requisite of accretion, which is,
that the alluvium is deposited on the portion of claimant's land which is adjacent to the river bank.
Wherefore the herein disputed land it is not capable of being appropriated by any private person,
except through express authorization granted in due form by a competent authority.
MARCIAL 48
Vda de Nazareno v CA
Facts : Mr. Jose Salasan and Mr. Jose Rabaya leased lots owned by Petitioner Antonio
Nazareno (Petitioners 'Predecessor-in-Interest). Such lots were formed due to accretion. In
1982 Salasan allegedly stopped paying rentals which as a result Petitioners filed a case for
ejectment with the MTC of CDO. A decision was rendered against Salasan , which was affirmed
by the RTC. Before the death of Antonio Nazareno he caused the approval by the Bureau of
Lands of the survey plan with a view of perfecting his title over the accretion being claimed by
him which Salasan and Rabuya were ejected from. Before the approved survey plan could be
released to the applicant, it was protested by Salasan before the Bureau of Land. Upon
investigation of the Regional Director of the Bureau of Land it was recommended that the
survey plan in the name of Antonio Nazareno be cancelled and he was ordered to vacate the
portion of land adjudicated to him and private respondent Salasan be placed in possession
thereof. Upon the denial of the late Antonio Nazareno's Motion for reconsideration petitioners
Desamparado Vda. de Nazareno and Leticia Tapia Nazareno filed a case before the RTC. The
RTC dismissed the complaint for failure to exhaust administrative remedies which resulted in
the finality of the administrative decision of the Bureau of Land on Appeal the CA affirmed the
decision of the RTC dismissing the complaint.

Issue: Whether the subject land is public land and placing it under the jurisdiction of the Bureau
of Lands and as well as the as the office of the Secretary of Agriculture and Natural Resources.

Held: Subject Land is public land and petition of is dismissed for lack of merit.

Ratio: Petitioners claim that such land was private land because it was acquired through
accretion is wrong because for Article 457 of the Civil Code to apply the land must be formed
through natural effects and without the intervention of Man. Also the doctrine applied in
Meneses vs CA was applied in said case at bar which states :

Accretion, as a mode of acquiring property under Art. 457 of the Civil Code,
requires the concurrence of these requisites: (1) that the deposition of soil
or sediment be gradual and imperceptible; (2) that it be the result of the
action of the waters of the river (or sea); and (3) that the land where
accretion takes place is adjacent to the banks of rivers (or the sea coast).
These are called the rules on alluvion which if present in a case, give to
the owners of lands adjoining the banks of rivers or streams any
accretion gradually received from the effects of the current of waters.

Therefore making the act of Director Palad valid and binding to the parties.
NAVARRO 49

G.R. No. L-3788 December 21, 1907

PEDRO P. ROXAS, petitioner-appellee,


vs.
JULIA TUASON, THE MUNICIPALITY OF SAN PEDRO MACATI, AND ALEJANDRO AND CONSOLACION AGUIRRE, respondents-appellants.

FACTS:

Attorneys on behalf of Pedro P. Roxas, applied for the registration of the estate owned by the said Roxas, known as the Hacienda de San Pedro Macati, in accordance
with the provisions of the Land Registration Act; said hacienda was acquired by the petitioner by inheritance under the will of his late father, Jose Bonifacio Roxas, y
Ubaldo. The property consists of four different parcels of land. The building constructed of strong materials, called the "Casa-Quinta" or "Casa de Ingenieros," belonging
also to said Roxas, is erected within parcel "C,". It does not appear that said hacienda is mortgaged nor that any person has any right to or any interest therein; and it is
almost wholly occupied at the present time, under lease, by about 429 tenants.

The owners of the adjoining properties having been summoned and notified by means of subpoenas and notices published in the daily papers, one of them, Julia Tuason,
appeared and set forth her opposition to the registration and authentication of the title of the petitioner, Roxas, as regards the parcel marked "C," for the reason that two
old monuments which had separated their respective properties had been pulled down and new ones erected without her consent, and in her opinion the latter included a
considerable portion of the land owned by her.

Evidence consisting of both oral testimony and documents, which appear in the record, having been adduced by both parties in the suit, the judge, rendered his decision
and ordered the registration of the Hacienda of San Pedro Macati in favor of Pedro Roxas.

The representative of the petitioner affirms that the real boundary of the hacienda on the side that adjoins the land of Tuason was and still is a creek or sapa separating
both properties, and that in former years said creek was wider that at the present time.

The respondent, however, maintains that the boundary between the sitio called Suavoy, formerly an island of that name, and the Hacienda of San Pedro Macati is
determined by straight lines drawn between some old monuments distant a few yards from the bank of the said creek.

ISSUE:

The only subject of controversy between the petitioner, owner of the Hacienda of San Pedro Macati, and the respondent Julia Tuason is the question of the boundary line,
between their respective contiguous premises.

RULING:

The record does not show that the boundary of the land of Julia Tuason was inclosed by monuments belonging to her or that the creek which divides the sitio or Island of
Suavoy from the land of the said hacienda is included within the respondent's land, since in the bill of sale executed by the procurador general of the Augustinian friars to
Julia Tuason, no mention is made of monuments erected thereon nor of any creek existing in the large tract of land purchased by her, except that the land is situated in the
barrio of Suavoy and that it is bounded on two sides by the Hacienda of San Pedro Macati.

The proven fact that said creek was wider in 1871, when it had a width of about 4 Spanish yards, is the best explanation as to why some of the monuments of the
Hacienda of San Pedro Macati are now at some distance from the bank of the same, and no legal reason whatever exists why the slow increase which has taken place on
the hacienda's side should be considered as belonging to the respondent, inasmuch as the latter does not own the bed of the creek and because it may be assumed that
the slow decrease in the width thereof benefited both properties equally since the respondent has not been able to show or prove that her land has been thereby reduced.

Article 366 of the Civil Code in dealing with the right of accession to real property reads:

The accretions which banks of rivers may gradually receive from the effects of the currents belong to the owners of the estates bordering thereon.

The provision in this article is perfectly applicable to the strip of land, which, on account of the accretion, has come to be undeniable increase in the land of the hacienda
inasmuch as it has increased all along the bank of the creek, the gradual effect of the currents; and even though the law does not require an express act of possession of
the accretion which has enlarged the estate, it is certain that the owner of the hacienda has possessed it for more than thirty years through his tenants, who have been
cultivating their respective parcels of land together with the corresponding portion of the said strip down to the bank of said creek.

For these considerations the question of the situation of the old monuments and the placing of new ones in the intervening space is of no importance, inasmuch as it has
already been shown that the respondent has no title to the accretion which by spontaneous increase formed the strip of land between the creek and the monuments, and
no proof is offered in the record that the land of Julia Tuason reached the other side of the creek toward the Hacienda of San Pedro Macati.
The result of the evidence, as stated in the judgment appealed from, does not maintain the claim of the respondent; on the contrary, it has been shown in a convincing
manner that the present natural limit of both properties is the aforesaid creek; therefore, the opposition filed by Julia Tuason is untenable.

Therefore, by virtue of the considerations above set forth, it is our opinion that the judgment appealed from should be affirmed.
NERI 50

Cureg vs IAC 177 SCRA 313 (1989)


Facts:
Private respondents, except Domingo Apostol, are the heirs of Domingo Gerardo, the owner of
the subject land (2,500 hectares). Since July 26, 1894, Domingos father, Francisco, and
predecessors-in-interest have been in actual, open, peaceful and continuous possession of the
land, under a bona fide claim of ownership and adverse to all other claimants. Such land called
motherland was declared for taxation purposes in the name of Francisco Gerardo. The
respondents then sold the land to Domingo Apostol and executed an extra-judicial partition on
September 10, 1982. During the time such was executed, there were already signs of accretion
of 3 hectares on the north caused by the northward movement of the Cagayan River. The
accretion and the motherland was declared by Domingo for taxation purposes under his name
on September 15, 1982. On the other hand, petitioners contend that their predecessor, Antonio
Carniyan, was the owner of the accretion of the subject land. Antonio Carniyan acquired the
land from his father-in-law on October 5, 1956, as evidenced by an Absolute Deed of Sale.
Antonio declared the land for taxation purposes and even had an OCT (issued November 24,
1968) pursuant to Free Patent dated May 21, 1968. This title includes the accretion portion and
the motherland (total of 5.5 ha) being claimed by the Gerardos and Apostol
.ISSUE: WON Apostol and the Gerardos have better title over the accretion than Cureg
RULING: NO. The private respondents presented (4) tax declarations as evidence of their
ownership over the land, but these declarations are not sufficient evidence to prove ownership.
On the other hand, petitioners have an OCT under the name of Antonio Carniyan to show. Tax
declarations are not conclusive evidence of ownership and should not prevail over a decree of
registration. A decree of registration bars all claims and rights which arose or may have existed
prior the decree. The issuance of the decree quiets the land.
ISSUE 2: WON the accretion (5.5 ha) became part of the registered land of Cureg.
RULING: NO. The area covered by Antonio Carniyans OCT did not include the accretion of 5.5
hectares. The increase in the area of Cureg's land, being an accretion left by the change of
course or the northward movement of the Cagayan River does not automatically become
registered land just because the lot which receives such accretion is covered by a Torrens title.
It has not actually been placed yet in the Torrens system and for that it is but required that the
owners place it under the system.
OGUIS 51

Agne v. Director of Lands


181 SCRA 793, 805 (1990) Facts:
Private respondents filed in CFI for recovery of possesion of land
stating that they are the real owners of a land in possession of the
petitioners. The latter answered that the said land was formerly part of
a river owned by them by reason of accretion due to a big flood in
1920; and since then, they and their predecessor-in-interets has
occupied it openly and adversely as riparian owners; constrcuted
improvements, irrigation canals, planted trees and agricultural crops
with the same.
The Petioners also filed a case against the Director of Lands for the
annulment of title and for reconveyance and to clear title. They also
contended that it was 1971 that they found out it was granted by the
government to Agoon. However the said land according to the
Petioners are private therefore cannot be subject to public grant.
Issue: Whether the action for annulment of the free patent of many
years after it became final and indefeasible states no cause of action.
Held: No. The facts alleged constitue a sufficient cause of action.
Ratio: There need be no act on their part to subject the old river bed
to their ownership, as it is subject thereto ipso jure from the moment
the mode of acquisition becomes evident, without need of any formal
act of acquisition. Such abandoned riverbed had fallen to the private
ownership of the owner of the land through which the new river bed
passes even without any formal act of his will and any unauthorized
occupant thereof will be considered as a trespasser.
PACANA 52

HEIRS OF PACIFICO POCDO vs AVILA G.R. No.199146 MARCH 19,2014

FACTS:

In June 2000, Pacifico Pocdo, who was later substituted by his heirs upon his death, filed a complaint to
quiet title over a 1,728square meter property (disputed property) located in Camp 7, Baguio City. As it
appears, in 1894, Pocdo Pool, who died in 1942, began his occupation and claim on three lots that were
eventually surveyed in his name as Lot 43, Lot 44 and Lot 45. The heirs filed their ancestral land claims with
the DENR and in August 1991, Certificates of Ancestral Lands Claims (CALS) were issued by the DENR for
Lots 44 and 45, but Lot 43 was not approved due to Memorandum Order 9815 issued by the DENR
Secretary in September 1998. Polon Pocdo, an heir of Pocdo Pool, ceded his rights over the three lots to
Pacifico Pocdo in exchange for a one hectare lot to be taken from Lot 43. Polon entered into a Catulagan
with Arsenia Avila authorizing the latter to undertake the segregation of his onehectare land from Lot 43 in
exchange of 2,000 sq meters from his one-hectare land. The Catulagan and Waiver of Rights in order, the
CENRO of Baguio City issued in favor of Avila a Certificate of Exclusion from Lot 43. However, the Heirs of
Polon Pocdo and his wife filed an affidavit of cancellation which was dismissed for lack of jurisdiction and the
validity of the Amicable Settlement, Catulagan and Deed of Waiver of Rights were recognized. Upon DENRs
affirmation, Pacifico Pocdo, as the appellant, went on appeal to the Office of the President which resulted in
an affirmance of DENR Secretarys decision. Furthermore, the trial court dismissed the case for lack of
jurisdiction upon holding that the DENR had already declared the disputed property as public land. Thus, the
claim of petitioners to quiet title is not proper since they do not have title over the disputed property. The
Court of Appeals ruled that petitioners, in raising the issue of quieting of title, failed to allege any legal or
equitable title to quiet. On the validity of the Catulagan and the Waiver of Rights, the Court of Appeals ruled
that only the contracting parties are bound by the stipulations of the said documents.

ISSUE/S

Whether or not the RTC has jurisdiction over actions involving the possession of lands, even assuming
without admitting that the land is a public land

Whether or not the CA erred in upholding the dismissal of the case

Whether or not the CA erred in finding that the petitioners have no title to the property that would support
an action for quieting of title despite the existence of the records as proof that the petitioners have
rights/title over the subject property

RULING:

The Court of First Instance of Baguio and Benguet had no jurisdiction to order the registration of lands
already declared being part of the Baguio Townsite Reservation. Precisely on this bone that Lot 43 was not
awarded a Certificate of Land Ancestral Claim under DENR Circular No. 03, series of 1990, because it is
within the Baguio Townsite Reservation. This was affirmed by the Office of the President which held that
such lands belong to the public domain and are no longer registrable under the Land Registration Act.

Having established that the disputed property is public land, the trial court was therefore correct in
dismissing the complaint to quiet title for lack of jurisdiction over the disputed property which is still part of
the public domain.

In an action for quieting of title the two indispensable requisites in an action to quiet title are: (1) that the
plaintiff has a legal or equitable title to or interest in the real property subject of the action; and (2) that
there is a cloud on his title by reason of any instrument, record, deed, claim, encumbrance or proceeding,
which must be shown to be in fact invalid or inoperative. The records reveal that the heirs of Pocdo Pool,
were not even granted a Certificate of Ancestral Land Claim over Lot 43, which remains public land.

Wherefore, the petition was dismissed.


PADUGANAN 53

CAMPIT v GRIPA, et. al.


GR No. 195443
September 17, 2014

FACTS: Respondents Gripa, et. al. occupied a 2.7 hectare agricultural land
situated in Umangan, Mangatarem, Pangasinan covered by TCT No. 122237 issued in
the petitioners name. The petitioner claimed to have purchased the property from his
father Jose Campit in the year 1977. On the other hand, respondents claimed to be the
rightful owners of the subject property, as earlier adjudged by the court in Civil Case No.
11858 decided on June 12, 1961, and in Civil Case No. 15357 decided on August 8,
1978. The Court, in these cases, cancelled the titles of the petitioner and his father Jose
because they were obtained through the misrepresentation of the petitioners
grandfather, Isidro Campit. The respondents have long intended to divide the properties
among themselves but despite formal demands, the petitioner refused to surrender the
said land. The respondents filed a new case for annulment and cancellation of title with
the RTC on Aug. 15, 2003 and ruled in favor of respondents. The CA affirmed the
RTCs decision.

ISSUE: Whether or not petitioner is the rightful owner of the subject property?

HELD: No. Petitioner is not the true owner of the subject agricultural land. The
issue on the validity of the petitioners title to the subject property has long been settled
in Civil Case No. 15357, where the court, in its decision dated August 8, 1978, which
became final and executory on July 19, 1979, had found and declared the petitioners
title null and void by reason of fraud and misrepresentation. The Torrens system of
registration cannot be used to protect a usurper from the true owner, nor can it be used
as a shield for the commission of fraud, or to permit one to enrich oneself at the
expense of others.
PAGDANGANAN 54
DE GUZMAN VS. TABANGAO REALTY INC. (2015)
FACTS: Herminio De Guzman and his siblings filed a complaint for quiet title against respondent
Filipinas Shell Petroleum Corporation (FSPC) over the parcel of land they inherited from their
parents. The subject property had long been levied upon and sold to respondent at an execution
sale. The only remaining right of petitioners predecessors-in-interest over the property was the
right to redeem. Petitioners predecessors-in-interest failed to redeem the property. Petitioners
claimed that the acquisition of the subject property at the execution sale was not valid because of
respondents failure to secure a final deed of sale and/or conveyance 13 years after registration of
the Sheriffs Certificate of Title.

ISSUE:
WON the complaint for quiet title will prosper
HELD:
No. All rights, title, interest, and claim of the spouses De Guzman to the subject property was
already acquired by respondent upon the expiration of the one-year redemption period without
redemption being made. The execution of the final deed of sale and/or conveyance to respondent is
a mere formality and confirmation of the title already vested in respondent. The right
acquired by the purchaser at an execution sale is inchoate and does not become absolute until after
the expiration of the redemption period without the right of redemption having been exercised.
Since the spouses failed to redeem the subject property within the period allowed by law, they have
been divested of their rights over the property.
PASCUAL 55
FIRST DIVISION
G.R. No. 160728

CLT REALTY DEVELOPMENT CORPORATION, Petitioner,


vs.
PHIL-VILLE DEVELOPMENT AND HOUSING CORPORATION, REPUBLIC OF THE PHILIPPINES
(through the OFFICE OF THE SOLICITOR GENERAL), and the REGISTER OF DEEDS OF METRO
MANILA DISTRICT III, CALOOCAN CITY, Respondents.

FACTS:

This case started with a Complaint for Quieting of Title, Damages and Injunction filed by respondent
Phil-Ville against petitioner and the Register of Deeds of Metro Manila District III on August 28,
1991 before the RTC of Caloocan City. Respondent Phil-Ville claims that it is the registered owner
and actual possessor of sixteen (16) parcels of land in Baesa, Caloocan City which were derived
from and were part of Lot 26, Maysilo Estate originally covered by Original Certificate of Title (OCT)
No. 994 issued on May 3, 1917. Respondent Phil-Ville alleged that based on official records of the
office of respondent Register of Deeds and the Land Registration Authority, petitioner was issued
Transfer Certificate of Title (TCT) No. T-177013, covering a parcel of land situated in Caloocan City,
containing an area of 891,547.43 m2, which overlaps respondent Phil-Villes aforesaid parcels of
land. Phil-Ville traced its title from OCT 994, upon which was issued TCT 4210 dated August 21,
1918, for subdivisions 1 and 3 of Lot 26, and TCT 4211, dated September 9, 1918, for subdivision 2
of Lot 26, to co-owners A. Ruiz and M. Leuterio, totally conveying Lot 26. Subsequently, the entire
Lot 26 was conveyed to F. Gonzales thru TCT 5261. When F. Gonzales died, the land was transferred
to his six (6) children. Such land was expropriated by the Government and was subdivided into 77
lots and sold to the vendees of NHA. Phil-Ville was then able to acquire 8 lots thru sale and deeds of
exchange and registered it under its name.

Petitioner traced its title from OCT 994 dated April 19, 1917. The first conveyance was recorded in
TCT R-15166 when Atty. Dimson was awarded, thru a court order date June 16, 1966, as attorneys
fees 25% of whatever remained under Lot 25-A, 26, 27, 28 and 29 undisposed of the intestate
estate of decedent Concepcion Vidal. Lot 26 was subsequently conveyed to E. Hipolito under TCT R-
17994, and was later acquired by the petitioner thru TCT 177013. TCT R-17994 was annotated
with "(P)ursuant to Ministry Opinion No. 239 dated November 4, 1982, Notice is hereby given that
this titles (sic) is subject to the verification by the LRC Verification Committee on questionable
titles, plan, decrees and other documents".

ISSUE:

The only issue for the Supreme Court is the question of whether or not petitioners TCT No. T-
177013 imposes a cloud on respondent Phil-Villes titles to the 16 parcels of land subject matter of
this case, as provided in Article 476 of the Civil Code.

RULING:
The Court ruled in favor of respondent Phil-Ville. In the case of Manotok Realty vs. CLT Realty
(2009), TCT 177013 was already declared null and void. OCT 994 is dated May 3, 1917 based on the
records of the ROD and OCT 994 dated April 19, 1917 was inexistent. Thus all titles that traced its
roots from OCT 994 dated April 19, 1917were invalidated, including TCT 177013. Moreover, the
acquisition by Atty. Dimson of the interests of the late Maria De La Concepcion Vidal in Lot 26 and
other lots of the Maysilo Estate was subject to the condition, that something remained of said lot in
the intestate estate of said deceased that have not been disposed of. Therefore, the subsequent
conveyances to E. Hipolito and then to the peititoner were still subject to such condition.
PEDROZA 56

56. Chung vs. Mondragon, GR 179754, Nov. 21, 2012


Facts: Respondent, Jack Mondragon, sold a portion of the land in dispute to his
co-respondent. Petitioner filed a case to annul the sale arguing that
respondent had no right to sell a portion of the land and the sale to Regis
Schmitz created a cloud upon their title, and for quieting of title. RTC
dismissed the case ruling that since respondent was a co-owner of the land, he
has the right to dispose his undivided share. CA affirmed RTC.
ISSUE: WON respondent can dispose a portion of the said land.
RULING: Yes. In issues for quieting of title, the plaintiff need to prove only two
things: (1) the plaintiff or complainant has a legal or an equitable title to or
interest in the real property subject of the action; (2) that the deed, claim,
encumbrance or proceeding claimed to be casting a cloud on his title must be
shown to be in fact invalid or inoperative despite its prima facie appearance of
validity or legal efficacy. It is evident that the land belongs to Andrea, Rafaels
second wife. The land could not have belonged to Rafael, because he is not
even named in the OCT. The fact that Rafael died ahead of Andrea, and that he
is not even named in the title, gives the impression that the land belonged
solely to the heirs of Andrea, to the exclusion of Rafael. Add to this is the fact
that petitioners are not in possession of the land. Thus, petitioners, being
descendants of the first wife of Rafael, do not have a legal or an equitable title
to or interest in the land and thus could not maintain an action for quieting of
title. Respondent being a descendant of Andrea could sell a portion of the land.
Although the RTC and CA properly dismissed the case, they did so by an
erroneous appreciation of the facts and evidence.
PELAEZ 57
#57 Bahais v Pascual (GR 169272)
Facts:
On December 11, 2000, Bahais filed a complaint with RTC for quieting of title and other claims
against Silverio Songcuan, his heirs and the DENR. Bahais alleged that he is the lawful and absolute
owner of two parcels of land and that he had been in possession for 30 years.

Bureau of Lands rejected the sales applications of Bahaiss predecessors-in-interest for the lots and
ordered all those in privity to vacate the lots and to remove their improvements. DENR secretary
affirmed with this decision. Recourse to the office of the President had been unavailing, so DENR
issued writs of execution.

Pascual moved to dismiss saying that petitioner had no legal right to file the complaint since the
decision of the Bureau of Lands has become final and executory.

RTC denied the motion to dismiss; it was not yet final since President's ruling on the appeal was
unavailable. Respondent elevated this case to the CA, questioning the propriety of the RTC's denial.
CA set aside the RTCs ruling.

Issue: Whether CA committed a reversible error in when it set aside RTCs ruling

Held:
CA committed no reversible error.

We find it clear that the petitioner no longer had any legal or equitable title to or interest in the lots.
The petitioners status as possessor and owner of the lots had been settled in the final and
executory December 4, 1985 decision of the Bureau of Lands that the DENR Secretary and the OP
affirmed on appeal. Thus, the petitioner is not entitled to the possession and ownership of the lots.
They no longer have a cause of action.
Jurisprudence teaches us that the decisions and orders of administrative agencies, such as the
Bureau of Lands, rendered pursuant to their quasi-judicial authority, upon finality, have the force
and binding effect of a final judgment within the purview of the doctrine of res judicata.
Accordingly, the petitioner is now barred from challenging the validity of the final and executory
Bureau of Lands December 4, 1985 decision.
PEPITO 58
VDA. DE AVILES v. CA

An action to quiet title or to remove cloud may not be brought for the purpose of settling a
boundary dispute.

FACTS:
Eduardo Aviles, the predecessor of the petitioners is the bother of defendant Camilo. They
inherited their lands from their parents and have agreed to subdivide the same amongst
themselves. The area alloted (sic) to Eduardo Aviles is 16,111 square meters more or less, to
Anastacio Aviles is 16,214 square meters more or less, while the area alloted to defendant
Camilo Aviles is 14,470 square meters more or less.

Defendants land composed of the riceland portion of his land is 13,290 square meters, the
fishpond portion is 500 square meters and the residential portion is 680 square meters, or a
total of 14,470 square meters.

The Petitioners claim that they are the owners of the fish pond which they claim is within their
area. Defendant Camilo Aviles asserted a color of title over the northern portion of the property
with an area of approximately 1,200 square meters by constructing a bamboo fence (thereon)
and moving the earthen dikes, thereby molesting and disturbing the peaceful possession of the
plaintiffs over said portion.

Petitioners say that the fences were created to unduly encroach to their property but the
defendant said that he merely reconstructed the same.

Petitioners brought an action to quiet title but were denied thus this case.

ISSUE: Whether or not Petitioners filed the right action

RULING:
No, Petitioners filed the wrong action. This is obviously a boundary dispute and as such the
action must fail.

Art. 476. Whenever there is a cloud on title to real property or any interest therein, by reason of
any instrument, record, claim, encumbrance or proceeding which is apparently valid or effective
but is, in truth and in fact, invalid, ineffective, voidable, or unenforceable, and may be prejudicial
to said title, an action may be brought to remove such cloud or to quiet the title.
An action may also be brought to prevent a cloud from being cast upon a title to real property or
any interest therein.

Petitioners fail to point out any any instrument, record, claim, encumbrance or proceeding that
could been a cloud to their title. In fact, both plaintiffs and defendant admitted the existence of
the agreement of partition dated June 8, 1957 and in accordance therewith, a fixed area was
allotted to them and that the only controversy is whether these lands were properly measured.

A special civil action for quieting of title is not the proper remedy for settling a boundary dispute,
and that petitioners should have instituted an ejectment suit instead. An action for forcible entry,
whenever warranted by the period prescribed in Rule 70, or for recovery of possession de facto,
also within the prescribed period, may be availed of by the petitioners, in which proceeding the
boundary dispute may be fully threshed out.
PIMENTEL 59
Fernandez vs Villegas, GR No. 200191, Aug. 20, 2014

FACTS:

Petitioner Lourdes C. Fernandez (Lourdes) and her sister, Cecilia Siapno (Cecilia) filed a Complaint for
Ejectment before the Municipal Trial Court in Cities of Dagupan City (MTCC) against respondent Norma
Villegas (Norma) seeking to recover possession of a parcel of land covered by Transfer Certificate of
Title (TCT) No. 1917 (subject property).

In their complaint, Lourdes and Cecilia (plaintiffs) averred that they are the registered owners of the
subject property on which both Lourdes and respondents previously lived under one roof. However,
when their house was destroyed by typhoon Cosme, Lourdes transferred to a nipa hut on the same
lot, while Norma, Cecilias daughter-in-law, and her family were advised to relocate but, in the
meantime, allowed to use a portion thereof. Instead, respondents erected a house thereon over
plaintiffs objections and, despite demands, refused to vacate and surrender possession of the subject
property.

ISSUE: WON Lourdes lone signature on the verificaftion attached to the CA petition constituted
substantial compliance with the rules.

RULING: YES. Article 487 of the Civil Code explicitly provides that any of the co-owners may bring an
action for ejectment, without the necessity of joining all the other co-owners as co-plaintiffs because the
suit is deemed to be instituted for the benefit of all. To reiterate, both Lourdes and Cecilia are co-
plaintiffs in the ejectment suit. Thus, they share a commonality of interest and cause of action as against
respondents. Notably, even the petition for review filed before the CA indicated that they are the
petitioners therein and that the same was filed on their behalf. Hence, the lone signature of Lourdes on
the verification attached to the CA petition constituted substantial compliance with the rules.
QUINLOG 60

60 EXTRAORDINARY DEVELOPMENT CORPORATION (EDC), vs.


HERMINIA F. SAMSON-BICO AND ELY B. FLESTADOS
G.R. No. 191090, 13 October 2014

Facts:
During his lifetime, Apolonio owned a parcel of land consisting of 29,748 square meters situated at Barangay
Pantok, Binangonan, Rizal. When Apolonio and Maria died, the property was inherited by Juan and Irenea.
When the latter died, the heirs of Juan and Irenea became co- owners of the property.

On 16 April 2002, the heirs of Juan, without the consent of respondents, and the heirs of Irenea executed in
favor of EDC a Deed of Absolute Sale covering the subject property for P2,974,800.00. EDC was able to
register the Deed of Absolute Sale and transfer the tax declaration in its name. This prompted respondents to
file the Complaint for Annulment of Contract and Reconveyance of Possession with Damages.

In its Answer, EDC alleged that it is a buyer in good faith and for value because of the honest belief that the
heirs of Juan are the only heirs of the late Apolonio. EDC counterclaimed for damages. However, the heirs of
Juan asserted that respondents were aware of and were parties to the contract to sell entered into by them
and EDC.
Issue: Whether the sale by a co-owner of a physical portion of an undivided property be valid.
Held:
Yes. The fact that the agreement in question purported to sell a concrete portion of the hacienda does not
render the sale void, for it is a well-established principle that the binding force of a contract must be
recognized as far as it is legally possible to do so. When a thing is of no force as I do it, it shall have as much
force as it can have.
As a general rule, a co-owner cannot rightfully dispose of a particular portion of a co-owned property prior to
partition among all the co-owners. However, this should not signify that the vendee does not acquire anything
at all in case a physically segregated area of the co-owned lot is in fact sold to him. Since the co-
owner/vendors undivided interest could properly be the object of the contract of sale, what the vendee
obtains are the same rights as the vendor had as co-owner. Thus, the vendee steps into the shoes of the
vendor as co-owner and acquires a proportionate abstract share in the property held in common.
RAMOS 61
TORRES vs. LAPINID G.R. No. 187987 November 26, 2014
Facts:
Petitioners and Jesus are co-owners of the disputed lot. An action for partition of the parcels of land
against the petitioners and other co-owners were filed. A judgement was rendered based on a
compromise agreement signed by the parties wherein they agreed that Jesus, Mariano and Vicente
were jointly authorized to sell the said properties and receive the proceeds thereof and distribute
them to all the co-owners. The agreement was later amended to exclude Jesus as an authorized
seller. The petitioners inspected the property and discovered that Lapinid was occupying a specific
portion of the lot by virtue of a deed of sale executed by Jesus in favor of Lapinid. Consequently, a
forcible entry case was filed against Lapinid. The petitioners prayed that the deed of sale be
declared null and void arguing that the sale of a definite portion of a co-owned property without
notice to the other co-owners is without force and effect.
Issue:
Whether Jesus, as a co-owner, can validly sell a portion of the property he co-owns in favor of
another person.
Held:
A co-owner has an absolute ownership of his undivided and pro indiviso share in the co-owned
property. He has the right to alienate, assign and mortgage it, even to the extent of substituting a
third person in its enjoyment provided that no personal rights will be affected. This is evident from
Art. 493 of the Civil Code.
A co-owner is an owner of the whole and over the whole he exercises the right of dominion, but he
is at the same time the owner of a portion which is truly abstract. Hence, his co-owners have no
right to enjoin a coowner who intends to alienate or substitute his abstract portion or substitute a
third person in its enjoyment.
In this case, Jesus can validly alienate his co-owned property in favor of Lapinid, free from any
opposition from the co-owners. Lapinid, as a transferee, validly obtained the same rights of Jesus
from the date of the execution of a valid sale. Absent any proof that the sale was not perfected, the
validity of sale subsists. In essence, Lapinid steps into the shoes of Jesus as co-owner of an ideal and
proportionate share in the property held in common. Thus, from the perfection of contract, Lapinid
eventually became a co-owner of the property.
RODRIGO 62

62 ALEJANDRINO v CA
G.R. No. 114151. September 17, 1998
Facts:
Late spouses Alejandrino left their six children named Marcelino, Gregorio, Ciriaco, Mauricia,
Laurencia and Abundio a 219-square-meter Lot 2798 in Mambaling, Cebu City. Upon the
demise of the Alejandrino spouses, the property should have been divided among their children
with each child having a share of 36.50 square meters. However, the estate of the Alejandrino
spouses was not settled in accordance with the procedure outlined in the Rules of Court.
Petitioner Mauricia (one of the children) acquired a total of 97.43 square meters after allegedly
purchasing some of her siblings share. It turned out, however, that a third party named Licerio
Nique, the private respondent, also purchased portions of the siblings property, totaling to
121.67 square meters from Mauricias sister, Laurencia.

Issue:
Whether as an heir of the Alejandrino property, Laurencia may validly sell specific portions
thereof to a third party

Ruling:
No. In the instant case, Laurencia was within her hereditary rights in selling her pro
indiviso share in Lot No. 2798. However, because the property had not yet been partitioned in
accordance with the Rules of Court, no particular portion of the property could be identified as
yet and delineated as the object of the sale. Thus, interpreting Article 493 of the Civil Code
providing that an alienation of a co-owned property shall be limited to the portion which may be
allotted to (the seller) in the division upon the termination of the co-ownership, the Court said:
x x x (p)ursuant to this law, a co-owner has the right to alienate his pro-indiviso share
in the co-owned property even without the consent of the other co-owners.
Nevertheless, as a mere part owner, he cannot alienate the shares of the other co-
owners. The prohibition is premised on the elementary rule that `no one can give what
he does not have (Nemo dat quod non habet)
ROSETTE 63
SABUERO 64

GR No. 189420, March 26, 2014


Raul Arambulo & Teresita Dela Cruz vs Genero Nolasco & Jeremy Nolasco

Facts:
Petitioners, together with their siblings and their mother co-owned a 233 square meters land in
Tondo, Manila. When their mother died, she was succeeded by her husband, Genero Nolasco and
their children.
On January 8, 1999, petitioners filed a petition for relief alleging that all co-owners, except for
Nolasco, have authorized to sell their respective shares to the properties, saying that in the Civil
Code, if one or more co-owners shall withhold their consent to the alterations in the thing owned
in common, the courts may afford adequate relief.
Nolasco responded that they did not know about the intention to sell, because they were not
called to participate in the negotiations regarding the sale of the property.

Issue: Whether the respondents are withholding their consent and whether this withholding is
prejudicial to the petitioners.

Held:
From the foregoing, it may be deduced that since a co-owner is entitled to sell his undivided
share, a sale of the entire property by one co-owner without the consent of the other co-owners is
not null and void. However, only the rights of the co-owner-seller are transferred, thereby
making the buyer a co-owner of the property.
To be a co-owner of a property does not mean that one is deprived of every recognition of the
disposal of the thing, of the free use of his right within the circumstantial conditions of such
judicial status, nor it is necessary, for the use and enjoyment, or the right of free disposal, that the
previous consent of all the interested parties be obtained.
SINDIONG 65

DE GUIA V. CA
FACTS:
Abejo instituted an action for recovery of possession with damages against De
Guia. In, his complaint, Abejo alleged that he is the owner of the undivided portion of
the property used as a fishpond and that De Guia continues to possess and use the fish
pond without any contract and without paying rent to Abejos damage and prejudice. De
Guia also claims ownership over an undivided portion of the fishpond and claims that
partition and not recovery of possession was the proper remedy under the
circumstances.

ISSUE: Whether a co-owner can file an ejectment case against a co-owner?


RULING:
Yes. Any co-owner may file an action under Article 487 not only against a third
person, but also against another co-owner who takes exclusive possession and asserts
exclusive ownership of the property. However, the only purpose of the action is to obtain
recognition of the co-ownership. The plaintiff cannot seek exclusion of the defendant
from the property because as a co-owner he has a right of possession. The plaintiff
cannot recover any material or determinate part of the property.
SIRAD 66
BAILON-CASILAO vs CA GR L-78178April 14, 1988
FACTS:-Rosalia and Gaudencio Bailon sold a portion of a parcel of land to Delgado.-They sold the
remainder to Lanuza.-Lanuza acquired from Delgado his part of the land.-Lanuzas husband,
acting under SPA, sold the whole of the land to Afable in 1975, as evidenced by Deeds of Sale,
stating therein that the lands were unregistered.-Heirs of Bailon filed a case of recovery of
property with damages against Afable, et. al. covering subject land in 1981, asserting that the
subject were co-owned by them and is covered by OCT issued on 1931.-Afable contended that he
acquired the land through prescription and that petitioners were guilty of laches.RTC:-Found
and declared Afable as co-owner with 2/6 respective undivided share, having validly bought fromr
R & G Bailon.-Found petitioners as co-owners with 1/6 share each.-Ordered segregation of the
undivided interests in the property to terminate co-ownership-Ordered Afable to restore plaintiffs
to their respective shares and to pay damagesCA: affirmed RTCs decisionISSUE: WON laches
barred the petitioners action for recovery of subject property against respondentHELD: NO-
Basic requirements of laches:
1. conduct on the part of the defendant or of one under whom he claims, giving rise to the situation
of which complaint is made and for which the complainant seeks a remedy;
2. delay in asserting the complainants rights, the complainant having had knowledge or notice of
the defendants conduct and having been afforded an opportunity to institute suit;
3. lack of knowledge or notice on the part of the defendant that the complainant would assert the
right on which he bases his suit; and
4. injury or prejudice to the defendant in the event relief is accorded to the complainant or the suit
is not held to be barred.-mere fact of delay in asserting right is insufficient to constitute laches.
The complainant must have had a knowledge of the conduct of the defendant and that he must have
been afforded an opportunity to institute suit.-while there was a delay of petitioners in asserting
their rights, such delay was not attended with any knowledge of the sale nor with any opportunity
to bring suit. It is undisputed that the petitioners co- owners had entrusted the care and
management of the parcel of land to Rosalia Bailon who was the oldest among them. In view of the
lack of knowledge by the petitioners of the conduct of Rosalia in selling the land in 1975 without
their consent and the absence of the opportunity to institute the proper action until 1981, laches
may not be asserted against the petitioners.-Afable was guilty of bad faith in purchasing the land
as there are only 2 signatories to the deed out of 6 co-owners and the absence of special
authorization to sell. Even as the land was misrepresented as unregistered, the truth was that
Afable already had notice that the land was titled in the name of 6 persons by virtue of the
Certificate of Title which was already in his possession even before the sale, as apparent from his
testimony.
TAHANLANGIT 67
GAPACAN V. OMIPET, 387 SCRA 383

FACTS:

Paicat Gapacan is the primitive possessor of an unregistered land in Mt. Province, divided into 3 parcels
of riceland and another planted to camote and declared by him for taxation purposes. He had two children
Maria and Antonio. Antonio left for a long while to try his luck in the mines Benguet. Maria remained,
took care of their father and eventually took over the cultivation of the land.

Antonio Gapacan returned to and executed an Affidavit of Transfer of Real Property showing that the
property had been transferred to him by his sister Maria Gapacan-Omipet (Omipet) making him in effect
the legal owner of the property in question. Since then, Antonio Gapacans family (Gapacans) had been
occupying and cultivating the property.

In 1992, Omipet hired laborers to clear and cultivate portions of the disputed property. Gapacans
prohibited them Gapacans and ordered the defendants to vacate the land and restore possession to
plaintiffs. This caused Omipet to file an action to quiet title in RTC and that she be declared the lawful
owner. RTC adjudged that Gapacans have right of possession over the land. On appeal CA, declared that
the land is common property of both Omipet and Gapacans and ordered its partition.

Both parties appealed. Gapacans alleged that CA cannot declare that the land is common property since it
deviates from the cause of action in the trial court. Omipets appeal is mostly factual.

ISSUE: Whether or not property rights can be decided in an action to quiet title?

HELD: Yes.
Art. 476 of the Civil Code provides that an action to quiet title may be brought when there exists a cloud
on the title to a real property or any interest therein. In the case of Bautista v. Exconde, we held that the
property owner whose property rights were being disturbed may ask a competent court for a proper
determination of the respective rights of the party-claimants, not only to place things in their proper place,
that is, to require the one who has no right to refrain from acts injurious to the peaceful enjoyment of the
property not only of the rightful owner but also for the benefit of both with the view of dissipating any
cloud of doubt over the property. It goes without saying therefore that the appellate court in resolving the
present controversy is well within its authority to adjudicate on the respective rights of the parties, that is,
to pass upon the ownership of the property; hence to declare the same as common property.
TALA 68
VILMA QUINTOS, represented by her Attorney-in-Fact FIDEL I. QUINTOS, JR.; FLORENCIA I. DANCEL, represented by
her Attorney-in-Fact FLOVY I. DANCEL; and CATALINO L. IBARRA, Petitioners,
vs.
PELAGIA I. NICOLAS, NOLI L. IBARRA, SANTIAGO L. IBARRA, PEDRO L. IBARRA, DAVID L. IBARRA, GILBERTO L.
IBARRA, HEIRS OF AUGUSTO L. IBARRA, namely CONCHITA R., IBARRA, APOLONIO IBARRA, and NARCISO IBARRA,
and the spouses RECTO CANDELARIO and ROSEMARIE CANDELARIO, Respondents.

Facts:

Petitioner and respondent are siblings. Their parents were the owners of the subject property, a 281 sqm. parcel of
land. Upon the death of their parents, all ten siblings acquired ownership of the subject property. Sometime in
2002, respondent siblings brought an action for partition against petitioners. However, the case was dismissed for
failure of the parties to appear.

Thereafter, respondent siblings instead resorted to executing a Deed of Adjudication to transfer the property in
favour of the ten (10) siblings. As a result, the old TCT was cancelled and was replaced with a new TCT by the
Registry of Deeds of Tarlac in the names of the ten (10) heirs of the Ibarra spouses. Respondent siblings sold their
7/10 undivided share over the property in favour of their co-respondents, the spouses Recto and Rosemarie
Candelario.

Petitioners filed a complaint for Quieting of Title and Damages against respondents. According to them, they have
been in adverse, open, continuous, and uninterrupted possession of the property for over four decades and are
entitled to equitable title thereto. Petitioners also denied any participation in the execution of the deed of
adjudication and the agreement of subdivision.

Respondents countered that petitioners cause of action was already barred by estoppel when sometime in 2006,
one of petitioners offered to buy the 7/10 undivided share of the respondent siblings. They point out that this is an
admission on the part of petitioners that the property is not entirely theirs.

RTC dismissed the petition and declared respondent Rosemarie Candelario and Recto Candelario as the absolute
owners. Petitioner appealed with the court of appeals but was denied. Hence, this case.

Issue:

Whether or not the petitioners were able to prove ownership over the property;

Whether or not the respondents counterclaim for partition is already barred by laches or res judicata; and

Ruling: The petition is meritorious in part.

According to the SC, petitioners were not able to prove equitable title or ownership over the property and as a
result they lack cause of action to file an action for quieting of title. Their alleged open, continuous, exclusive and
uninterrupted possession of the subject property was negated by the fact that respondents entered into a contract
of lease over the subject lot without any objection from the petitioners. The cardinal rule is that bare allegation of
title is not enough to establish ownership. The burden of proof is on the plaintiff to establish his or her case by
preponderance of evidence.

The SC also held that the counter claim for partition was not barred by prior judgment. In their answer to the
counterclaim, petitioners countered that the action for partition has already been barred by res judicata.
Petitioners cited Rule 17, Sec.3 which enumerated the instances when a complaint may be dismissed due to the
plaintiff's fault. First is if the plaintiff fails to appear on the date for the presentation of his evidence in chief on the
complaint. Second is if he fails to prosecute his action for an unreasonable length of time. And third is if he fails to
comply with the Rules or any order of the court. According to the SC, the general rule is that dismissal of a case for
failure to prosecute is to be regarded as an adjudication on the merits and with prejudice to the filing of another
action, and the only exception is when the order of dismissal expressly contains a qualification that the dismissal is
without prejudice. In the case at bar, petitioners claim that the Order does not say that the dismissal is without
prejudice.

However, the SC provided that the dismissal with prejudice under Rule 17, Sec. 3 of the Rules of Court cannot
defeat the substantive right of a co-owner to ask for partition at any time. According to Art. 494 of the civil code,
No co-owner shall be obliged to remain in the co-ownership. Each co-owner may demand at any time the partition
of the thing owned in common, insofar as his share is concerned.

Based on the provision. It is clear that the law does not favour the retention of co-ownership as a property
relation. Between dismissal with prejudice under Rule 17, Sec. 3 and the right granted to co-owners under Art. 494
of the Civil Code, the latter must prevail. It is very important to remember that a substantive law cannot be
defeated by a procedural rule.

Thus, petition is partially granted. Decision of CA is affirmed with modification and the case is remanded to the RTC
for the purpose of partitioning the property.
TALON 69

ADILLE vs. COURT OF APPEALS, 157 SCRA 455, January 29, 1988

FACTS:

The property in dispute was originally owned by Felisa Alzul who got married
twice. Her child in the first marriage was petitioner Rustico Adile and her children
in the second marriage were respondents Emetria Asejo et al. During her
lifetime, Felisa Alzul sold the property in pacto de retro with a three-year
repurchase period. Felisa died before she could repurchase the property. During
the redemption period, Rustico Adille repurchased the property by himself alone
at his own expense, and after that, he executed a deed of extra-judicial partition
representing himself to be the only heir and child of his mother Felisa.
Consequently, he was able to secure title in his name alone. His half-siblings,
herein respondents, filed a case for partition and accounting claiming that
Rustico was only a trustee on an implied trust when he redeemed the property,
and thus, he cannot claim exclusive ownership of the entire property.

ISSUES:

1) Whether or not a co-owner may acquire exclusive ownership over the property
held in common.
2) Whether or nor Rustico had constituted himself a negotiorum gestor

HELD:

1) No. The right to repurchase may be exercised by a co-owner with respect to his
share alone. Although Rustico Adille redeemed the property in its entirety,
shouldering the expenses did not make him the owner of all of it.

2) Yes. The petitioner, in taking over the property, did so on behalf of his co-heirs, in
which event, he had constituted himself a negotiorum gestor under Art 2144 of
the Civil Code, or for his exclusive benefit, in which case, he is guilty of fraud,
and must act as trustee, the respondents being the beneficiaries, pursuant to Art
1456.
TAN 70
Sanchez v CA

Facts: Lilia Sanchez constructed a house on a 76-square meter lot. The lot was registered with six co-
owners, including petitioner. Later, the lot was registered to Teria, respondent, by virtue of a Deed of
Absolute Sale supposed to have been executed by all six (6) co-owners in her favor. Petitioner claimed
that she did not affix her signature on the document and subsequently refused to vacate the lot. MeTC
declared the sale was valid only to the extent of 5/6 of the lot and the other 1/6 remaining as the property
of petitioner, on account of her signature in the Deed of Absolute Sale having been established as a
forgery. A writ of execution was issued and a Notice to Vacate was served by the sheriff upon petitioner
who however refused to heed the Notice.

Issue: WON the sale is valid as to petitioners share?

Ruling: No. Although assigned an aliquot but abstract part of the property, the metes and bounds of
petitioners lot has not been designated. As she was not a party to the Deed of Absolute Sale voluntarily
entered into by the other co-owners, her right to 1/6 of the property must be respected. Partition needs to
be effected to protect her right to her definite share and determine the boundaries of her property. Such
partition must be done without prejudice to the rights of private respondent Virginia Teria as buyer of the
5/6 portion of the lot under dispute.
TIMOG 71

Pangan v CA

G.R. No. L-39299

October 18, 1988

FACT:

The petitioner filed an application for the registration of the land in their names by virtue of their
continuous and exclusive possession by them which application was approved. The private
respondent then filed a petition to set aside the said decision on the ground that she as an heir
too, is entitled to the share of the property. However the petition was dissmissed on the ground
that the private respondent right over the property had been forfeited by extinctive prescription.
On appeal, the decision was reversed.

ISSUE: Whether the private respondent is entitled to the share of the property.

HELD:

Yes. Private respondent as an heir is a co-owner of the property. The petitioners alleged that
private respondents failure to assert her rights over the years allowed them to perfect their claim
of ownership by acquisitive prescription and so excluded the private respondent from her share
in the subject matter. For title to prescribe in favor of the co-owner, however, there must be a
clear showing that he has repudiated the claims of the other co-owners and that they have been
categorically advised of the exclusive claim he is making to the property in question. It is only
when such unequivocal notice has been given that the period of prescription will begin to run
against the other co-owners and ultimately divest them of their own title if they do not
seasonably defend it. There was no adequate notice by the petitioners to the private respondent of
the rejection of her claim to her share in the subject property. Noticeably absent here is a
categorical assertion by the petitioners of their exclusive right to the entire property that barred
her own claim of ownership of one-half thereof nor is there any explanation as to why they said
she had no right to a share.
TINSAY 72
72. Heirs of Restar vs-Heirs of Cichon ~Diana

FACTS:

1. Emilio Restar owns a 5,918 sqm land in Aklan. He has 8 children: Flores Restar (dead); Dolores Restar-
Cichon (dead) -------heirs as co-plaintiffs/defendants; Perpetua Restar-Sta. Maria (dead) -------heirs as co-
plaintiffs/defendants; Paciencia Restar-Manares (plaintiff); Dominica Restar-Relojero (plaintiff); Policarpio
Restar; Maria Rose Restar (dead) -------heirs as co-plaintiffs/defendants; Adolfo Restar In 1935, Emilio
died intestate. Since then, Flores (the eldest son) took over the land. He gave his siblings the shares of palay
harvested from the land (up to 1991). In 1945, the lands owned by Emilio (aside from subject land) were
divided among the siblings. In 1960, Flores executed a joint affidavit to have: a. the tax declaration under
Emilio to be cancelled b. The tax declaration be issued under his name. In 1989, Flores died. In 1998, the
plaintiffs discovered the change in the tax declarations. In 1999, the plaintiffs filed a case against the heirs
of Flores for partition [of the lot], declaration of nullity of documents, ownership with damages and
preliminary injunction They contend that: After Flores death, his heirs were only allowed to stay in the
land until all of his children has finished studying and they have agreed that the lot will be partitioned to
the 8 siblings thereafter Heirs of Flores argued that: They have been in possession of the lot for more than
30 years; By 1977, all of the children were already done with their studies; and they have been paying the
real estate taxes since time immemorial

RTC-Aklan ruled in favor of Flores heirs:

there was repudiation of co-ownership


lot acquired by prescription
CA reversed the RTC decision
- The possession was during the existence of co-ownership
- no adequate notice by Flores to his other co-heirs/co-owners of the repudiation of the co-
ownership
- no categorical assertion by the defendants of their exclusive right to the entire lot that barred
the plaintiffs' claim of ownership.

ISSUE: Can there be prescription of the action to demand partition of a co-owned property?

HELD: Generally, this is not allowed, EXCEPT if theres a clear repudiation of the co-ownership and the co-owners
were apprised of the claim of adverse and exclusive ownership

-Petition Granted-

RULING:

1. Jurisprudence allows a co-owner to acquire ownership by prescription of a co-owned property.


While the action to demand partition of a co-owned property does not prescribe, a co-
owner may acquire ownership thereof by prescription where there exists a clear repudiation of the
co-ownership, and the co-owners are apprised of the claim of adverse and exclusive ownership.

Requisites:
a. Clear Repudiation of Co-Ownership
b. Co-owners apprised of the claim of adverse and exclusive ownership
2 types of acquisitive prescription:
a. Ordinary: possession of things in good faith and with just title for a period of ten years
b. Extraordinary: (No good faith and Just title) - uninterrupted adverse possession for thirty
years

2.There is Extraordinary Prescription in this case


Flores took possession of the land after Emilios death and exercised acts of dominion over it
Statutory Prescription started in 1960:
- Flores applied for a tax declaration under his name, with neither title or good faith
- His siblings are deemed aware of it public document
After 30 years of possession, his possession ripened into ownership through acquisitive
prescription [ Apply Art. 1137, CC]
Flores and his heirs had for more than 38 years possessed the land in open, adverse
and continuous possession in the concept of owner which length of possession had
never been questioned, rebutted or disputed by any of respondents, being thus duly
supported by substantial evidence, he and his heirs have become owner of the lot by
extraordinary prescription.
Other evidence: Tax declarations and receipts, when coupled with actual possession, can be the
basis of a claim of ownership through prescription
3. Flores committed acts showing adverse possession to his co-heirs:
Cancellation of the tax declaration certificate in the name of Emilio and securing another in his
name;
Execution of a Joint Affidavit stating that he is the owner and possessor thereof to the exclusion of
respondents;
Payment of real estate tax and irrigation fees without respondents having ever contributed any
share therein;
Continued enjoyment of the property and its produce to the exclusion of respondents
**Flores' adverse possession was continued by his heirs.
LAWS APPLIED:

A. Co-ownership
ART. 494. No co-owner shall be obliged to remain in the co-ownership. Each co-owner may
demand at any time the partition of the thing owned in common, insofar as his share is concerned.
xxx
No prescription shall run in favor of a co-owner or co-heir against his co-owners or co-heirs so
long as he expressly or impliedly recognizes the co-ownership.

B. Acquisitive Prescription
ART. 1117. Acquisitive prescription of dominion and other real rights may be ordinary or
extraordinary.

Ordinary acquisitive prescription requires possession of things in good faith and with just title
for the time fixed by law.

ART. 1134. Ownership and other real rights over immovable property are acquired by ordinary
prescription through possession of ten years.

ART. 1137. Ownership and other real rights over immovables also prescribe through uninterrupted
adverse possession thereof for thirty years, without need of title or of good faith.
TORAYNO 73
EPITACIO DELIMA, PACIANO DELIMA, FIDEL DELIMA, ET AL. VS. COURT OF APPEALS
G.R. NO. L-46296 201 SCRA 641 SEPTEMBER 24, 1991
PONENTE: MEDIALDEA, J.

Doctrine: No prescription shall run in favor of a co-owner against his co-owners or co-heirs as long as he
expressly or impliedly recognizes the co-ownership. The exception however is that the from the moment
one of the co-owners claims that he is the absolute and exclusive owner of the properties and denies the
others any share therein, the question is no longer one of partition but of ownership.

Facts: Lino Delima acquired Lot. No. 7758 of the Talisay-Minglanilla Friar Lands Estate in Cebu by sale
on installments from the government. After his demise in 1921 he had his three brothers and a sister
listed as his heirs. The heirs were Eulalio Delima, Juanita Delima, Galileo Delima, and Vicente Delima. A
new Transfer Certificate of Title was issued in the name of the Legal Heirs of Lino Delima represented by
Galileo Delima. On September 22, 1953, Galileo executed an affidavit of Extra-judicial Declaration of
Heirs adjudicating to himself the subject property excluding the other heirs. He declared the lot to be of
his own and paid for its taxes. On February 29, 1968, the surviving heirs of Eulalio and Juanita Delima,
filed with the Court of First Instance of Cebu an action for reconveyance and partition of property and for
the annulment of the certificate of title issued plus damages against their Uncle Galileo. Vicente Delima
was also later included as party defendant for his refusal to help in the action.

The trial court decided in favor of the petitioners rendering the TCT No. 3009 null and void and
declaring Vicente, the Heirs of Juanita, the Heirs of Eulalio and the Heirs of Galileo to be owners of the
property, each sharing a pro-indiviso share of one-fourth of the whole. The respondents, Heirs of Galileo
Delima, appealed to the Court of Appeals which reversed the decision in their favor. It upheld the claim of
Galileo that the other brothers and sisters have already waived their rights to the property being that it
was Galileo alone that paid for the balance of the purchase price and the realty taxes for the property.

Issue: Whether or not petitioners action for partition is already barred by the statutory period provided by
law which shall enable Galileo Delima to perfect his claim of ownership by acquisitive prescription to the
exclusion of petitioners from their shared in the disputed property?

Held: Yes, Article 494 (5) of the Civil Code provides that: No prescription shall run in favor of a co-owner
or co-heir against his co-owners or co-heirs so long as he expressly or impliedly recognized the co-
ownership. By this it is therefore understood that possession by a co-owner will not be presumed to be
adverse to the others, but will be held to benefit all. Being that Galileo was holding the property in
representation of the co-owners; he was therefore acting as an administrator who took care of the
property yet still having the ultimate obligation to deliver the property to his co-owners.

However this rule shall no longer apply when one of the co-owners begin to claim the absolute
and exclusive ownership and denies the others any share therein. The imprescriptability of the action for
partition shall no longer apply since Galileo is adversely claiming lone ownership over the property. In
order that a possession be considered adverse amounting to a repudiation of the co-ownership, the
following elements must concur: (1) that the trustee has performed the unequivocal acts amounting to an
ouster of the cestui que trust; (2) that such positive acts of repudiation had been made known to the
cestui que trust; and (3) that the evidence thereon should be clear and conclusive.

Since Galileo, having executed a deed of partition and obtained subsequent to that the
cancellation of the old title and the creation of a new one wherein he appears as the new owner of the
property, he thereby in effect denied and repudiated the ownership of the other co-owners over their
shares. From this act, the statute of limitations started to run. Since an action for reconveyance
UBAY UBAY 74
Mariategui v. C.A.
205 SCRA 337

FACTS:
Lupo Mariategui died without a will. During his lifetime, Lupo Mariategui contracted three (3)
marriages. With his first wife he begot four (4) children. With his second wife, he begot a
daughter. And with his third wife, he begot three children.

At the time of his death, Lupo Mariategui left certain properties which he acquired when he was
still unmarried. Lupo's descendants by his first and second marriages executed a deed of
extrajudicial partition whereby they adjudicated unto themselves lots in the Muntinglupa Estate.
Thereafter, an Original Certiicate of Title (OCT) was issued in the name of the said heirs.
Subsequently, the registered owners caused the subdivision of the said lot into for which
separate transfer certificates of title were issued to the respective parties.

Lupo's children by his third marriage filed with the lower court an amended complaint claiming
that the lots were owned by their common father, Lupo Mariategui, and that, with the
adjudication of the lots to their co-heirs, they (children of the third marriage) were deprived of
their respective shares in the lots. Plaintiffs pray for partition of the estate of their deceased
father and annulment of the deed of extrajudicial partition. The defendants filed a motion to
dismiss on the grounds of lack of cause of action and prescription. The motion to dismiss was
denied by the trial court. The plaintiffs elevated the case to the Court of Appeals (CA) but the
CA upheld the trial courts decision.

ISSUE:
Whether or not prescription barred private respondents' right to demand the partition of the
estate of Lupo Mariategui.

HELD:
No. Prescription does not run against private respondents with respect to the filing of the action
for partition so long as the heirs for whose benefit prescription is invoked, have not expressly or
impliedly repudiated the co-ownership. In other words, prescription of an action for partition
does not lie except when the co-ownership is properly repudiated by the co-owner.

Petitioners contend that they have repudiated the co-ownership when they executed the
extrajudicial partition excluding the private respondents and registered the properties in their
own names. However, no valid repudiation was made by petitioners to the prejudice of private
respondents. Assuming petitioners' registration of the subject lot was an act of repudiation of the
co-ownership, prescription had not yet set in when private respondents filed the present action
for partition. Petitioners' registration of the properties in their names did not operate as a valid
repudiation of the co-ownership.
USOP 75
75. Lacbayan vs. Samoy, G.R. No. 165427, March 21, 2011

FACTS: The respondent, despite being already married, had an illicit relationship with the
petitioner. And that together with other three persons, established a company. Five parcels
of land were acquired and ostensibly registered under the petitioners and respondents
name as husband and wife. Eventually they decided to part ways and agreed to divide said
properties and terminate their business partnership as well by executing a Partition
Agreement. The respondent and petitioner agreed that the latter will have two parcels of
land and the other three will go to the respondent. However, the petitioner had additional
demands to which the respondent refused. Feeling aggrieved, the petitioner filed a
complaint for judicial partitioning of the said properties. The RTC dismissed the complaint
for lack of merit and was then elevated to the CA. The petitioners assertion was that she is
the pro indiviso owner of one-half of the properties in dispute and that the RTC subjected
the certificates of title over the said properties to collateral attack which is contrary to law
and jurisprudence. Petitioner also contended that it is improper to thresh out the issue on
ownership in an action for partition. The CA later denied the appeal.
ISSUE(S): 1.Whether an action for partition precludes a settlement on ownership.
2. Whether the Torrens title over the disputed properties was collaterally attacked in the
action for partition.
HELD:

The petition is bereft of merit and was denied.

1. The court held that the determination as to the existence of co-ownership is necessary in
the resolution of an action for partition. While the complaint involves partition, the same is
premised on whether there is existence of co-ownership between the parties. Petitioner
insists she is a co-owner pro indiviso of the based on the TCTs covering the subject
properties. Respondent maintains otherwise. Therefore, until the issue of co-ownership is
resolved, it would be premature to effect the partition. The complaint will not even lie if the
petitioner does not even have any rightful interest over the subject properties.

2. There is no dispute that a Torrens certificate of title cannot be collaterally attacked, but
that rule is not material to the case at bar. What cannot be collaterally attacked is the
certificate of title and not the title itself. Petitioner apparently confuses title with the
certificate of title. Title as a concept of ownership should not be confused with the
certificate of title as evidence of such ownership although both are interchangeably used.
VALMORES 76

LEONOR B. CRUZ,
v.
TEOFILA M. CATAPANG,
G.R. No. 164110
2008 February 12,

FACTS:
Petitioners Leonor Cruz, Luz Cruz and Norma Maligaya are the co-owners of aparcel of land.
Sometime, Teofila, with the consent of Norma(co-owner), built a house on a lot adjacent to the
subject parcel of land. The house built by Catapang intruded on a portion of the co-owned
property. Cruz learned about the intrusion and made demands for demolitio the part of the
structure encroaching upon their property. However, Catapang disregarded the demands Cruz
then filed a complaint for forcible entry against. The MCTC decided in favor of Cruz, ruling that consent
of only one of the co-owners is not sufficient to justify defendants construction of the house. On appeal, the
RTC affirmed the decision of the MCTC. CA reversed the RTCs decision and ruled OTHERWISE.

ISSUE:
1.Whether the consent given by one of the co-owners is sufficient to warrant thedismissal of a
complaint for forcible entry.

RULING:
No, Co-owners cannot devote common property to his or her exclusive use to theprejudice of the co-ownership.
In this case, the act is tantamount to devoting the property to her exclusive use. none of the co-owners
w/o the consent of the others, make alterations in the thing owned in common. In addition, each co-owner
may use the thing owned in common provided he does so in accordance with the purpose for which it is
intended and must not to injure the interest of the co-ownership. As such, respondents acts
constitute forcible entry.
The petition was GRANTED
VILLAMOR 77

#77 SANTOS v. LUSTRE G.R. No. 151016

FACTS:
In Civil Case No. 1330, both heirs of Dominga Lustre, filed with the RTC, a complaint for
Declaration of the Inexistence of Contract, Annulment of Title, Conveyance and
Damages against Froilan Santos, son of the appellant spouses.
Civil Case No. 2115 was instituted while Civil Case No. 1330 was pending. It was filed
by Lustres other heirs against the parties of this case. They averred that the sale of the
property to Natividad Santos was simulated, spurious or fake, and that they discovered
that the spouses Santos transferred the property to Froilan Santos when an ejectment
suit was filed against them.
The RTC denied the respondents petition to dismiss. The petitioners claimed that the
second action must be dismissed based on it being barred by litis pendentia,
prescription, and laches.

ISSUE:
Whether the action for reconveyance on the ground that the certificate of title was
obtained by means of a fictitious deed of sale is virtually an action for the declaration of
its nullity which does not prescribe.

RULING:
Yes. When an action for reconveyance is filed, it would be in the nature of a suit for
quieting of title, an action that is imprescriptible. It follows then that the respondents
present action is not barred by laches.
VILLARUBIA 78
G.R. No. 179914 June 16, 2014
SPOUSES REYNALDO AND HILLY G. SOMBILON, Petitioners, vs. ATTY. REY FERDINAND
GARAY AND PHILIPPINE NATIONAL BANK, Respondents.
FACTS
Spouses Sombilon were the previous owners of a 601-square meter property in Maramag,
Bukidnon. The said property was foreclosed and sold at public auction. PNB emerged as the winning
bidder. The one-year redemption period lapsed but spouses Sombilon failed to redeem the property.
In 2005, they sought the help of Atty. Garay. Both parties were informed by PNB that the property
could be purchased at P2,938,000.00. The following day, Atty. Garay bought the property for himself
by making a down payment of P587,600.00. Upon learning, spouses Sombilon offered to buy back
the property from PNB. Subsequently, when a Final Deed of Conveyance was issued in favor of
PNB, the latter decided to approve the purchase offer of Atty. Garay since spouses Sombilon failed
to make the required down payment.
PNB filed an Ex-Parte Petition for Issuance of a Writ of Possession. Spouses Sombilon moved for a
reconsideration arguing that Atty. Garay, was barred from purchasing the property pursuant to
paragraph 5, Article 1491 of the Civil Code. Judge Venadas, Sr. issued an Order holding in
abeyance the implementation of the Writ of Possession. Aggrieved, Atty. Garay and PNB elevated
the case to the CA via a Petition for Certiorari with prayer for issuance of a Temporary Restraining
Order (TRO) and/or Injunction. CA rendered a Decision granting the Petition for Certiorari.
ISSUES
(1) Whether or not Judge Venadas, Sr. committed grave abuse of discretion in holding in abeyance
the implementation of the Writ of Possession;
(2) Whether or not he should be administratively sanctioned for holding in abeyance the
implementation of the Writ of Possession and for disregarding Sections 4, 5, and 6, Rule 15 of the
Rules of Court.
HELD
Yes, he committed grave abuse of discretion because PNB, as the registered owner, is entitled to
the possession of the subject property as a matter of right. The redemption period had long
lapsed when PNB applied for the issuance of the Writ of Possession. In fact, the title over the
subject property had already been consolidated in PNBs name. The claim that the sale
between PNB and Atty. Garay was invalid as it was done in violation of paragraph 5, Article
1491 of the Civil Code is wrong. First, it was not a ground to defer the issuance of the Writ of
Possession. Second, it does not affect PNBs right to possess the subject property.
Yes, Judge Venadas, Sr. is guilty of grave abuse of authority bordering on gross ignorance of
procedure for blatantly disregarding Sections 4, 5, and 6, Rule 15 of the Rules of Court.
Records show that spouses Sombilon failed to comply with the three-day notice rule under
the Rules of Court, thereby rendering the motion fatally defective. Despite this, Judge
Venadas, Sr. still took cognizance of the motion filed by spouses Sombilon, depriving PNB
and Atty. Garay of their right to due process. To exculpate himself from the charges against
him, Judge Venadas, Sr. claims that the motion was personally served on PNB and its
counsel, but the latter parties refused to receive the same.
VISTAL 79
Javate v. Sps. Tiotuico
G.R. No. 187606
March 9, 2015
Peralta, J..

Facts of the case;

Petitioner owned a parcel of land which was mortgaged to Guagua bank as security, upon
failure of the petitioner to pay obligation it was foreclosed. Petitioners were given a period to
redeem but failed to redeem thus ownership consolidated in the bank. Respondent then bought
the subject property from the bank and petition the court for a writ of possession, which was
granted. Petitioner contend that only the bank has the right to issuance of writ of possession,
respondents as a subsequent buyer must resort to appropriate judicial remedy, which is
ejectment or accion reinvindicatoria in order to gain possession.

Issue:
W/N respondent are entitled to issuance of writ or possession

Ruling:
Yes, respondent are entitled to a writ of possession.

Rationale:
The respondents are correct for filing a petition for writ of possession. If the purchaser is the
mortgagee or third party during redemption period, writ may issue ex-parte. The remedy is
available to subsequent purchaser but only after hearing and after determining that subject
property is still in the possession of the mortgagor. If the property is in the possession of the
mortgagor, a writ of possession could be issued. If otherwise, the remedy is no longer available
but he may wrest possession through action of ejectment. There is no dispute that petitioner is
in possession of the property thus, issuance of the writ is proper. To require subsequent
purchaser to file a separate case of ejectment will only prolong and unduly deny possession of
property which already bought.
Waban 80
GR No. 160914
DELA CRUZ vs SPS HERMANO

Facts: On Sept. 27, 2001, Respondent spouses, the registered owner of subject house and lot, filed
an ejectment suit against petitioner. Respondent claims that said property is used as a
rest/vacation house. In his defense, petitioner alleged that the property was already sold by
respondent to a certain Benitez who sold the same to herein petitioner. Furthermore, the case at
bar is not one of an unlawful detainer or forcible entry to which an ejectment suit is based.

Issue: Whether the ejectment suit will prosper?

Held: No, it will not. The Supreme Court agrees that this was an action for ejectment in the nature
of accion reivindicatoria, the case was actually for forcible entry and sufficient in form. However,
the respondents failed to provide sufficient evidence in their allegation of prior physical possession
thereof, which is an element in forcible entry. Ownership certainly carries the right of possession,
but the possession contemplated is not exactly the same as that which is in issue in a forcible entry
case, which refers to possession de facto, or actual or material possession, and not one flowing out
of ownership.
Yap 81

PNB V. DE JESUS
411 SCRA 557

FACTS:
On 10 June 1995, respondent filed a complaint against petitioner before the Regional Trial Court of
Occidental Mindoro for recovery of ownership and possession, with damages, over the questioned property. In his
complaint, respondent stated that he had acquired a parcel of land situated in Mamburao, Occidental Mindoro, with
an area of 1,144 square meters covered by TCT No. T-17197, and that on 26 March 1993, he had caused a
verification survey of the property and discovered that the northern portion of the lot was being encroached upon by a
building of petitioner to the extent of 124 square meters. Despite two letters of demand sent by respondent, petitioner
failed and refused to vacate the area.
Petitioner, in its answer, asserted that when it acquired the lot and the building sometime in 1981 from then
Mayor Bienvenido Ignacio, the encroachment already was in existence and to remedy the situation, Mayor Ignacio
offered to sell the area in question (which then also belonged to Ignacio) to petitioner at P100.00 per square meter
which offer the latter claimed to have accepted. The sale, however, did not materialize when, without the knowledge
and consent of petitioner, Mayor Ignacio later mortgaged the lot to the Development Bank of the Philippines. He also
contends that he is a builder in good faith.

ISSUE:
Whether or not being a builder in good faith matters under article 448.

HELD:
NO. Article 448, of the Civil Code refers to a piece of land whose ownership is claimed by two or more
parties, one of whom has built some works (or sown or planted something) and not to a case where the owner of the
land is the builder, sower, or planter who then later loses ownership of the land by sale or otherwise for, elsewise
stated, where the true owner himself is the builder of works on his own land, the issue of good faith or bad faith is
entirely irrelevant.
Aban 82
PARILLA v. PILAR

G.R. No. 167680, November 30, 2006

FACTS:

Petitioners, as dealers of Pilipinas Shell Petroleum Corporation (Pilipinas Shell), have been in
possession of a parcel of land which was leased to them by the respondent. When the lease
contract expired, and despite demands to vacate, petitioners remained in possession of the
property on which they built improvements. Hence, respondent filed a complaint for ejectment
before the MTC which ordered the petitioners to vacate and to pay respondent a reasonable
compensation for the use of the property. It also ordered respondent to reimburse the petitioners
the amount Two Million Pesos representing the value of the improvements introduced on the
property.

RTC affirmed the MTCs Decision. However, the CA set aside the lower courts decision.

ISSUES:

Whether or not the petitioners are entitled to reimbursement for the improvements being builders
in good faith under Art. 453 of the Civil Code.

HELD:

Article 448 covers only cases in which the builders, sowers or planters believe themselves to be
owners of the land or, at least, have a claim of title thereto, but not when the interest is merely
that of a holder, such as a mere tenant, agent or usufructuary. A tenant cannot be said to be a
builder in good faith as he has no pretension to be owner.

It is Article 1678 of the New Civil Code which applies to the present case. However, the
petitioners claim for reimbursement of the alleged entire value of the improvements still does
not thus lie under Article 1678 there being no substantial evidence, e.g., receipts or other
documentary evidence detailing costs of construction. Besides, of the structures they originally
built, only the bodega-like, sari-sari store, and the parking lot now exist.

At all events, under Article 1678, it is the lessor who is given the option, upon termination of the
lease contract, either to appropriate the useful improvements by paying one-half of their value
at that time, or to allow the lessee to remove the improvements. This option solely belongs to the
lessor as the law is explicit that [s]hould the lessor refuse to reimburse said amount, the lessee
may remove the improvements, even though the principal thing may suffer damage thereby. It
appears that the lessor has opted not to reimburse.
Abragan 83

ABALOS VS. HEIRS OF TORIO


FACTS:

On July 24, 1996, herein respondents filed a Complaint for Recovery of Possession and Damages with
the Municipal Trial Court (MTC) of Binmaley, Pangasinan against Jaime Abalos (Jaime) and the spouses
Felix and Consuelo Salazar. Respondents contended that they are the children and heirs of one Vicente
Torio (Vicente) who died intestate, leaving behind a parcel of land measuring 2,950 square meters, more
or less, which is located at San Isidro Norte, Pangasinan. During the lifetime of Vicente and through his
tolerance petitioners. were allowed to stay and build their respective houses on the subject parcel of land.
Even after the death of Vicente, herein respondents allowed The petitioners to remain on the disputed lot.
However, in 1985, respondents asked Jaime and the Spouses Salazar to vacate the subject lot, but they
refused to heed the demand of respondents forcing respondents to file the complaint.

Jaime and the Spouses Salazar filed their Answer with Counterclaim, denying the material allegations in
the Complaint and asserting in their Special and Affirmative Defenses that respondents' cause of action is
barred by acquisitive prescription. Petitioners claim the court has no jurisdiction over the nature of the
action and the persons of the defendant. They also alleged that they are in actual, continuous and
peaceful possession of the subject lot as owners since time immemorial. They also said that they have
been paying real property taxes and have been introducing improvements on the said land.

ISSUES:
(1) Whether or not the petitioners and their predecessors-in-interest possessed the disputed lot in the
concept of an owner.
(2) Whether or not the possession of the petitioners is by mere tolerance of respondents and their
predecessors-in-interest.
(3) Whether or not the petitioners qualify to own the land through acquisitive prescription.

HELD:

(1) No.

In the instant case, it is clear that during their possession of the property in question, petitioners
acknowledged ownership thereof by the immediate predecessor-in-interest of respondents. This is clearly
shown by the Tax Declaration in the name of Jaime for the year 1984 wherein it contains a statement
admitting that Jaime's house was built on the land of Vicente, respondents' immediate predecessor-in-
interest.21 Petitioners never disputed such an acknowledgment. Thus, having knowledge that they nor
their predecessors-in-interest are not the owners of the disputed lot, petitioners' possession could not be
deemed as possession in good faith as to enable them to acquire the subject land by ordinary
prescription.

(2) Yes.
In this respect, the Court agrees with the CA that petitioners' possession of the lot in question was by
mere tolerance of respondents and their predecessors-in-interest. Acts of possessory character executed
due to license or by mere tolerance of the owner are inadequate for purposes of acquisitive
prescription.Possession, to constitute the foundation of a prescriptive right, must be en concepto de dueo,
or, to use the common law equivalent of the term, that possession should be adverse, if not, such
possessory acts, no matter how long, do not start the running of the period of prescription.

(3) No.

In the case at bar, the CA correctly held that even if the character of petitioners' possession of the
subject property had become adverse, as evidenced by their declaration of the same for tax purposes
under the names of their predecessors-in-interest, their possession still falls short of the required period
of thirty (30) years in cases of extraordinary acquisitive prescription. Records show that the earliest Tax
Declaration in the name of petitioners was in 1974. Reckoned from such date, the thirty-year period was
completed in 2004. However, herein respondents' complaint was filed in 1996, effectively interrupting
petitioners' possession upon service of summons on them.24 Thus, petitioners possession also did not
ripen into ownership, because they failed to meet the required statutory period of extraordinary
prescription.
Absuelo 84
Abudante 85
Suarez vs. Emboy, G.R. No. 187944, March 12, 2014

Facts:
A parcel of land was partitioned into 5 among the heirs of the Carlos and Asuncion. Lot No.
1907-A-2 was occupied by Felix and Marilou Emboy, who were claiming that they inherited it
from their mother Claudia Emboy, who inherited it from her parents Carlos and Asuncion.

Felix and Marilou were asked by their cousins to vacate Lot No. 1907-A-2 and transfer to Lot
No. 1907-A-5. They refused to comply and insisted that Claudia's inheritance pertained to Lot
No. 1907-A-2. In 2004, Felix and Marilou received a demand letter from Carmencita requiring
them to vacate the lot and informed them that she had already purchased the lot from the
former's relatives.

Felix and Marilou argued that the complaint for unlawful detainer was fundamentally inadequate.
There was practically no specific allegation as to when and how possession by tolerance of
them began.

Issue: Whether or not the complaint for unlawful detainer was inadequate.

Held:

In a complaint for unlawful detainer, the following requisites must be alleged:


(1) initially, possession of property by the defendant was by contract with or by tolerance of the
plaintiff;

(2) eventually, such possession became illegal upon notice by plaintiff to defendant of the
termination of the latters right of possession;

(3) thereafter, the defendant remained in possession of the property and deprived the plaintiff of
the enjoyment thereof; and

(4) within one year from the last demand on defendant to vacate the property, the plaintiff
instituted the complaint for ejectment. When the complaint fails to state the facts constituting a
forcible entry or unlawful detainer, as where it does not state how entry was effected or how the
dispossession started, the remedy should either be an accion publiciana or accion
reinvidicatoria.

In this case, the first requisite was absent. Carmencita failed to clearly allege and prove how
Emboy entered the lot and constructed a house upon it.

Hence, the complaint should not have been for unlawful detainer and the CA did not commit an
error in dismissing Carmencita's complaint.
AGCOPRA 86

G.R. No. 176055 March 17, 2014

SPOUSES EDMUNDO DELA CRUZ and AMELIA CONCIO-DELA CRUZ, Petitioners,


vs.
SPOUSES RUFINO R. CAPCO AND MARTY1 C. CAPCO, Respondents.

FACTS:
Spouses Dela Cruz filed a Complaint for Unlawful Detainer against the spouses Capco before the
MeTC of Pateros. They alleged that Teodora T. Concio, mother of petitioner Amelia Concio-Dela
Cruz, acquired ownership over a piece of land by virtue of a Decision rendered by the RTC of Pasig,
Branch 151 in Land Registration Case No. 9511. Petitioners further alleged that Teodora, out of
neighborliness and blood relationship, tolerated the spouses Capcos occupation thereof.
The property was subsequently conveyed to the petitioners. Intending to construct a house thereon
and utilize the space for their balut and salted eggs business, petitioners thus demanded that the
respondents vacate the property. The respondents refused.
In their answer, respondents pointed out that the complaint is defective for failing to allege the
exact metes and bounds of the property. Neither is a title attached thereto to show that petitioners
are the owners of the disputed property.
ISSUE:
Whether or not the spouses Dela Cruz have the better right to possess the property.
HELD:
The spouses Dela Cruz are able to establish by preponderance of evidence that they are the rightful
possessors of the property.
"The only issue in an ejectment case is the physical possession of real property possession de
facto and not possession de jure." But "where the parties to an ejectment case raise the issue of
ownership, the courts may pass upon that issue to determine who between the parties has the
better right to possess the property." However, it must be emphasized that "[t]he adjudication of
the issue of ownership is only provisional, and not a bar to an action between the same parties
involving title to the property."
The spouses Dela Cruz were able to prove by preponderance of evidence that they are the owners
of the lot. Their allegation is supported by a copy of the decision in Land Registration Case No.
9511, title of land issued to Teodora and extra-judicial settlement of the estate of Teodora
conveying the said property to Amelia.
Petitioners are better entitled to the material possession of the subject property. As its present
owners, they have a right to the possession of the property which is one of the attributes of
ownership.
Petition is granted.
ALAGAO 87
CASE No. 87
PIEDAD vs. GURIEZA, G.R. No. 207525, June 18, 2014

Facts:
Bonifacio Piedad alleged that he is the absolute owner of the 1/3 middle portion of a parcel of
residential land located at La Torre, Bayombong, Nueva Vizcaya which he acquired from his late
father. He also claimed that his ownership of lot took place even before his fathers death and was
validated through a Deed of Confirmation of an Adjudication and Partition by the legal heirs of said
land. Further, Bonifacio alleged that before migrating to Hawaii, he built a bungalow on the lot and
assigned numerous caretakers to look after it, the last of which were Spouses Gurieza.

However, Sps. Gurieza allegedly took interest of the bungalow and the subject lot after learning that
said property is public land. Using such information, Sps. Gurieza had the subject lot declared under
their name for tax purposes.

When Bonifacio learned of Sps. Guriezas acts, he filed a protest before the DENR. He then sent his
daughter to the country to personally demand that Sps. Gurieza vacate the subject lot
unconditionally; and for this purpose, initiated a complaint before the barangay court. However,
during the mediation proceedings, Sps. Gurieza refused to heed his daughters demand and even
challenged her to go to higher courts.

In their defense, Sps. Gurieza maintained they acquired the lot through acquisitive prescription.
They likewise assailed the authenticity and validity of the Deed of Confirmation.

The MTC ruled that Bonifacio had a better right of possession over the subject lot as evidenced by
the bungalow he built thereon and affidavits of witnesses attesting to Bonifacios claim.

The RTC affirmed the MTC Ruling. However, the CA reversed the ruling on the ground that Gurieza,
whom Bonifacio recognized as one of the heirs of the subject lot, continues to be a co-owner
thereof, because the lot was never partitioned since Gurieza never signed the Deed of Confirmation.

Issue:
Whether or not the CA correctly reversed the RTC ruling

Held:
SC grants Bonifacios petition and reverses the CAs resolution.

An ejectment case, based on the allegation of possession by tolerance, falls under the category of
unlawful detainer. Where the plaintiff allows the defendant to use his property by tolerance
without any contract, the defendant is necessarily bound by an implied promise that he will vacate
on demand, failing which, an action for unlawful detainer will lie.

In this light, the Court shall solely resolve the issue as to who between the parties has the better
right of possession de facto over the subject lot. Corollary thereto, issues pertaining to ownership
are better threshed out in another action instituted for such purpose.

The Court holds that Bonifacio had clearly established his cause of action for unlawful detainer.
(NOTE: YOU MAY OR MAY NOT INCLUDE THE ENUMERATION BELOW IN THE DIGEST)
The following established facts impel this conclusion:

1) Bonifacio already had possession of the subject lot which he never relinquished despite his
migration to Hawaii, as seen by his appointment of numerous caretakers, the last being Sps.
Gurieza.

2) When Bonifacio learned of Sps. Gurieza acts, he immediately took steps to terminate their
tolerated stay and demanded that they leave immediately, rendering the Sps. Guriezas stay on the
subject lot illegal.

3) Instead of vacating the subject lot, Sps. Gurieza defied Bonifacios demand and asserted their
ownership over the same. Sps. Gurieza were able to unlawfully withhold possession of the subject
lot from Bonifacio.

4) Bonifacio made his final demand to Sps. Gurieza and filed his complaint within the one (1) year
period from his last demand.

Original Case Link:


http://www.lawphil.net/judjuris/juri2014/jun2014/gr_207525_2014.html

PS.
Sorry, I couldnt limit it to one page.
ALAPAG 88, FE U. QUIJANO vs ATTY. DARYLL A. AMANTE

FACTS:

The petitioner and her siblings, namely: Eliseo, Jose and Gloria, inherited from their
father a parcel of land. Prior to any partition among the heirs, Eliseo sold a portion of his share
to respondent.

Fe, Eliseo, Jose and Gloria executed a deed of extrajudicial partition to divide their
father's estate among themselves. The partition resulted in the portions earlier sold by Eliseo to
the respondent being adjudicated to the petitioner instead of to Eliseo. The respondent's
occupation had become illegal following his refusal to vacate despite repeated demands. Hence,
this action for unlawful detainer.

The MTCC rendered its decision in favor of the petitioner. The parcel of land left by their
father had not yet been partitioned, rendering Eliseo a mere co-owner of the undivided estate
who had no right to dispose of a definite portion thereof.

On appeal, the Regional Trial Court (RTC) reversed the judgment of the MTCC, and
dismissed the complaint holding that the summary proceeding for ejectment was not proper.
Serious question of ownership of the disputed property was involved and which cannot be
determined in a summary proceeding for ejectment.

The CA affirmed the decision of the RTC on the ground that the respondent was either a
co-owner or an assignee holding the right of possession over the disputed property; that the
respondent became a co-owner along with Eliseo and his co-heirs, giving him the right to
participate in the partition of the estate owned in common by them; that because the
respondent was not given any notice of the project of partition or of the intention to effect the
partition, the partition made by the petitioner and her co-heirs did not bind him.

ISSUE:

Whether or not review on certiorari is correctly sought by the petitioner.

RULING:

The petition for review on certiorari lacks merit. To show that the possession was
initially lawful, the basis of such lawful possession must then be established. The acts of
tolerance must be proved, for bare allegation of tolerance did not suffice. At least, the petitioner
should show the overt acts indicative of her co-heirs' permission for him to occupy the disputed
property. But she did not adduce such evidence. Instead, she appeared to be herself not clear
and definite as to the respondents possession of the disputed property being merely tolerated
by Eliseo.

In contrast, the respondent consistently stood firm on his assertion that his possession
of the disputed property was in the concept of an owner, not by the mere tolerance of Eliseo
and actually presented the deeds of sale transferring ownership of the property to him.
Unlawful detainer action is dismissed for being an improper remedy.
ALIMAN 89

Case Number 89:

De Leon v. Dela Llana


GR No. 212277
February 11, 2015

FACTS:
The case stemmed from an unlawful detainer complaint filed by Gilbert dela Llana against
Robert de Leon on March 7, 2005. The said complaint, Dela Llana averred that sometime in
1999, he, through an undated contract of lease, leased a portion of a 541 square-meter property,
registered in his name, to Robert de Leon, which the latter intended use as a lottery. Upon failure
to pay rental, Dela Llana filed an ejectment suit to De Leon, but the latter posited that the
aforementioned lease contract was simulated, and hence, not binding on the parties. The lower
court dismissed the ejectment holding that the undated lease contract was a relatively simulated
contract and, as such, not binding. Decision had already become final and executory. De Llana
filed a second complaint for unlawful detainer. De Leon raised the defense of res judicata,
particularly averring that the second ejectment complaint should be dismissed given that it was
already barred by prior judgment.

ISSUE:
Whether the principle of res judicata applies that is, whether the second ejectment complaint
was barred by prior judgment.

RULING:
Res judicata (meaning, a matter adjudged) is a fundamental principle of law which precludes
parties from re-litigating issues actually litigated and determined by a prior and final judgment.
There is a bar by prior judgment where there is identity of parties, subject matter, and causes of
action between the first case where the judgment was rendered and the second case that is sought
to be barred. There is conclusiveness of judgment, on the other hand, where there is identity of
parties in the first and second cases, but no identity of causes of action.

With the identity of the parties, subject matter, and cause of action between the first and second
ejectment suit, it cannot thus be seriously doubted that the final and executory judgment in the
first case had already barred the resolution of the second. Hence, the second ejectment
complaint of dela Llana is dimissed.
ANGELES 90

90) Bunyi v. Factor

FACTS:

Respondent Fe S. Factor and Gloria Factor-Labao are co-owners of an 18-hectare piece of land. Gloria
Factor-Labao who was married to Ruben Labao died thus the administration and management of the
Factor compound including the subject rest house, passed on to respondent Fe S. Factor as co-owner of
the property.

As an act of goodwill and compassion, considering that Ruben Labao was sickly and had no means of
income, respondent allowed him to stay at the rest house for brief, transient and intermittent visits as a
guest of the Factor family. Ruben Lobao married petitioner Precy Bunyi, tenant in one of the houses
inside the compound. Eventually Ruben died. Petitioners forcibly opened the doors of the rest house and
stole all the personal properties owned by the Factor family and then audaciously occupied the premises.
Respondent alleged that petitioners unlawfully deprived her and the Factor family of the subject propertys
lawful use and possession. Hence, respondent Fe S. Factor filed a complaint for forcible entry against
herein petitioners Precy Bunyi and Mila Bunyi.

ISSUE:

WON the petioner has a better right of physical and material possesion of the subject property.

RULING:

For one to be considered in possession, one need not have actual or physical occupation of every square
inch of the property at all times. Possession can be acquired not only by material occupation, but also by
the fact that a thing is subject to the action of ones will or by the proper acts and legal formalities
established for acquiring such right. Possession can be acquired by juridical acts.
ARELLANO 91
PEOPLE vs. PENAFLORIDA
GR NO. 175604
April 10, 2008
Facts:
SPO3 Vicente Competente narrated that in his capacity as chief of the Investigation and Operation
Division of the PNP station in Tigaon, Camarines Sur, that he received a tip from an asset that a
bundle of marijuana was being transported by appellant to Huyon-huyon from another barangay in
Tigaon, Camarines Sur. Major Domingo Agravante,chief of police in Tigaon, then organized a team
composed of Competente as team leader, SPO2 Callo, SPO1 Portugal,PO3 Pillos and PO2 Edgar
Latam. The team boarded the police mobile car and proceeded to Sitio Nasulan in Barangay Huyon-
huyon. They overtook appellant who was on a bicycle. The police officers flagged appellant down
and found marijuana wrapped in a cellophane and newspaper together with other grocery items.
The amount of P1550.00 was also
found in appellants possession. The police officers confiscated these items and took photographs
thereof. Appellant was then brought to the headquarters where he was booked.
Major Lorlie Arroyo, a forensic chemist at the PNP Crime Laboratory Regional Office No. V was
presented as an expert witness to identify the subject marijuana leaves. She related that after taking
a representative sample from the 928-gram confiscated dried leaves, the same was tested positive
of marijuana. Appellant denied the accusations against him. That on his way home, they met Boyet
Obias (Obias) who requested appellant to bring a package wrapped in a newspaper to Jimmy
Gonzales, he placed it in the basket in front of his bicycle and Gonzales proceeded to the Tiagon
town proper. On his way home, he was flagged down by the police and was invited to go with them
to the headquarters. TC ruled that there was violation of Section 4, Article II of Republic Act (R.A.)
No. 6425, otherwise known as The Dangerous Drugs Act of 1972, hence, the instant case is now
before this Court on automatic review.
In assailing his conviction, appellant submits that there is doubt that he had freely and consciously
possessed marijuana. One of the issues raised is that, upon receipt of the information from the
asset, the police officers should have first investigated and tried to obtain a warrant of arrest
against appellant, instead of arbitrarily arresting him.

Issue:
Whether or not the contention of the appellant is tenable?

Ruling:
No.The police was tipped off at around 1:00pm that appellant was transporting marijuana to
Huyon-huyon.Certainly, they had no time to secure an arrest warrant as appellant was already in
transit and already committing a crime. The arrest was affected after appellant as caught in
flagrante delicto. He was seen riding his bicycle and carrying with him the contraband, hence,
demonstrating that a crime was then already being committed. Under the circumstances, the
police had probable cause to believe that appellant was committing a crime. Thus, the warrantless
arrest is justified. Article II, Section 4 of RA No. 6425, as amended by RA No 7659, states: Sec. 4.
Sale, Administration, Delivery, Distribution and Transportation of Prohibited Drugs The penalty of
reclusion perpetua to death and a fine ranging from five hundred thousand pesos to ten million
pesos shall be imposed upon any person who, unless authorized by law, shall sell, administer,
deliver, give away to another, distribute, dispatch in transit or transport any prohibited drug, or
shall act as broker in any of such transactions.

Jurisprudence defines transport as to carry or convey from one place to another. In the instant
case, appellant was riding his bicycle when he was caught by the police. He admitted that he was
about to convey the package, which contained marijuana, to a certain Jimmy Gonzales. Appellant
denies any knowledge that the package in his possession contained marijuana. But TC rejected his
contention, noting that it was impossible for appellant not to be aware of the contents of the
package because marijuana has a distinct sweet and unmistakable aroma which would have
alarmed him.

Taking one step further, the appellate court went on to declare that being mala prohibita, one
commits the crime under RA No. 6425 by mere possession of a prohibited drug without legal
authority. Intent, motive or knowledge thereof is not necessary. Finally, the lower courts correctly
sentenced appellant to suffer the penalty of reclusion perpetua and to pay a fine of one
million pesos by virtue of the amendment to Section 4, RA No. 6425 by RA No. 7659. TCs decision is
affirmed.
AVENIDO 92
MARK SOLEDAD y CRISTOBAL,
- versus -
PEOPLE OF THE PHILIPPINES,
The facts of the case, as narrated by the CA, are as follows:
Sometime in June 2004, private complainant Henry C. Yu received a call on his mobile phone from a
certain Tess or Juliet Villar (later identified as Rochelle Bagaporo), a credit card agent, who offered
a Citifinancing loan assistance at a low interest rate. Enticed by the offer, private complainant
invited Rochelle Bagaporo to go to his office in Quezon City. While in his office, Rochelle Bagaporo
indorsed private complainant to her immediate boss, a certain Arthur [later identified as
petitioner]. In their telephone conversation, [petitioner] told private complainant to submit
documents to a certain Carlo (later identified as Ronald Gobenchiong). Private complainant
submitted various documents, such as his Globe handyphone original platinum gold card,
identification cards and statements of accounts. Subsequently, private complainant followed up his
loan status but he failed to get in touch with either [petitioner] or Ronald Gobenchiong.

Petitioner was thus charged with Violation of Section 9(e), R.A. No. 8484 for possessing a
counterfeit access device or access device fraudulently applied for.

ISSUE: Whether or not petitioner was legally in possession of the credit card subject of the case.
No merit

The trial court convicted petitioner of possession of the credit card fraudulently applied for,
penalized by R.A. No. 8484. The law, however, does not define the word possession. Thus, we use
the term as defined in Article 523 of the Civil Code, that is, possession is the holding of a thing or the
enjoyment of a right. The acquisition of possession involves two elements: the corpus or the
material holding of the thing, and the animus possidendi or the intent to possess it.[12] Animus
possidendi is a state of mind, the presence or determination of which is largely dependent on
attendant events in each case. It may be inferred from the prior or contemporaneous acts of the
accused, as well as the surrounding circumstances.[13]

In this case, prior to the commission of the crime, petitioner fraudulently obtained from private
complainant various documents showing the latters identity. He, thereafter, obtained cellular
phones using private complainants identity. Undaunted, he fraudulently applied for a credit card
under the name and personal circumstances of private complainant. Upon the delivery of the credit
card applied for, the messenger (an NBI agent) required two valid identification cards. Petitioner
thus showed two identification cards with his picture on them, but bearing the name and forged
signature of private complainant. As evidence of the receipt of the envelope delivered, petitioner
signed the acknowledgment receipt shown by the messenger, indicating therein that the content of
the envelope was the Metrobank credit card.

Petitioner materially held the envelope containing the credit card with the intent to possess.
Contrary to petitioners contention that the credit card never came into his possession because it
was only delivered to him, the above narration shows that he, in fact, did an active part in acquiring
possession by presenting the identification cards purportedly showing his identity as Henry Yu.
Certainly, he had the intention to possess the same. Had he not actively participated, the envelope
would not have been given to him. Moreover, his signature on the acknowledgment receipt
indicates that there was delivery and that possession was transferred to him as the recipient.
Undoubtedly, petitioner knew that the envelope contained the Metrobank credit card, as clearly
indicated in the acknowledgment receipt, coupled with the fact that he applied for it using the
identity of private complainant.
BANTUAS - 93 - CHUA-BURCE vs. CA
[G.R. No. 109595. April 27, 2000]

Facts:

A bank cash custodian was found guilty by the trial court in the criminal case charging her of the
crime of estafa and was also adjudged liable for the amount of P150,000 in a civil case. Petitioner
seasonably appealed her conviction in the criminal case to the Court of Appeals and filed a separate
appeal in the civil case. The CA affirmed the trial courts decision in toto, hence, the present petition.
Petitioner questions the validity of trial procedures in the criminal proceeding and insists that there
can be no presumption of misappropriation when there were other persons who had access to the
cash in vault.

Issue:

Whether all the elements of estafa were proven beyond reasonable doubt.

Held:

No. The elements of Estafa, ART. 315 (1) (b), are the following:
a) The personal property is received in trust, on commission, for administration, or any other
circumstances, with the duty return.
b) There is a conversion/diversion of such property or denial that he received it.
c) Such conversion/diversion is to the injury of another
d) There is demand for such property

Among these, the Court finds the first element absent. It requires that the offender acquires both
material or physical possession and juridical possession of the thing received. Juridical possession
means a possession which gives the transferee a right over the thing which the transferee may set
up even against the owner, which is absent in this case. The petitioner was not an agent of the bank
but a cash custodian who was primarily responsible for the cash-in-vault. Her possession of the cash
belonging to the bank is akin to that of a bank teller, both being mere bank employees. They are
mere custodian or keeper of the funds received, and has no independent right or title to retain or
possess the same as against the bank. In comparison, an agent can assert, as against his own
principal, an independent, autonomous, right to retain money or goods received in consequence of
the agency; as when the principal fails to reimburse him for advances he has made, and indemnify
him for damages suffered without his fault.

In other words, petitioner being a mere cash custodian had no juridical possession over the missing
funds. Hence, the element of juridical possession being absent, petitioner cannot be convicted of the
crime of estafa.
BANAS 94
GABRIEL
VS
CRISOLOGO
G.R. NO. 204626, JUNE 9, 2014
______________________________________________________________________________
_______
FACTS:

Carmelita Crisologo alleged that she was the registered owner of two parcels of land (TCT) Nos.
T-13935 and T-13936 properties were covered by an assessment of real property and payments
of realty taxes were updated. She discovered that petitioners unlawfully entered and occupied her
properties by stealth, force and without her prior consent and knowledge and constructed houses
on the said lots. Upon the discovery, Atty Carmelita Crisologo(daughter of Carmeling Crisologo)
demanded that petitioners vacate the premises and remove their structures thereon. Petitioners
promised to be the said properties but failed to do so.Petitioners refused to vacate the said
properties despite repeated demands made by Crisologo. Crisologo filed a complaint for recovery
of possession and/or ownership with damages against petitioners.

ISSUE:

Whether the petitioners have a better right of possession over the subject parcels of land.

HELD:

Crisologo has a better right of possession over the subject parcels of land.
The nature and purpose of Accion Publiciana:

This is also known as Accion Plenaria de Posesion. It is an ordinary civil proceeding to determine the
better right of possession of realty independently of title. It refers to an ejectment suit filed after
the expiration of one & year from the accrual of the cause of action or from the unlawful withholding
of possession of the realty. The objective of the plaintiffs in accion publiciana is to recover possession
only, not ownership.
Biruar 95
Subic Bay Legend Resorts and Casinos Inc. vs. Fernandez
September 29, 2014 G.R. No. 193426=

FACTS:
On July 1, 1997, Bernard Fernandez, a brother of Ludwig and Deoven, filed a complaint for recovery
of sum of money and damages against the company. According to him, he went to the casino on June
13, 1997; he handed to his brothers $6,000.00 worth of chips belonging to him, for use at the
casino. Thereat, the company personnel accosted his brothers and confiscated his casino chips
worth $5,900.00, and failed to return the same to him despite demand. Brothers Deoven and
Ludwig Fernandez was accused of stealing casino chips from Subic Bay Legend Resorts and Casinos
Inc. They were made to confess that the chips were supplied by a casino employee, Michael Cabrera.

ISSUE:
Whether or not Bernard is the lawful possessor of the casino chips entitling him to collect from the
casino and award of damages.

RULING:
There is no basis to suppose that the casino chips found in Ludwin and Deovens possession were
stolen; petitioner acted arbitrarily in confiscating the same without basis. If it cannot be proved, in
the first place, that Cabrera stole the chips, then there is no more reason to suppose that Ludwin
and Deoven were dealing in or possessed stolen goods; unless the independent fact that Cabrera
stole the chips can be proved, it cannot be said that they must be confiscated when found to be in
Ludwin and Deovens possession.

Though casino chips do not constitute legal tender, there is no law which prohibits their use or
trade outside of the casino which issues them. Since casino chips are considered to have been
exchanged with their corresponding representative value it is with more reason that the Court
should require the casino to prove convincingly and persuasively that the chips it confiscated from
Ludwin and Deoven were indeed stolen from it. If Subic Bay Legend cannot prove its loss, then
Article 599 cannot apply; that the presumptions that the chips were exchanged for value remains.
BUCAY 96

Penta Pacific Realty Corp. vs Ley Construction and Development Corp


Nov. 24, 2014, GR No. 161589
Facts: Penta Pacific leased its properties to Ley Construction. Both parties then entered into a
contract to sell. Ley Construction failed to pay its amortizations prompting Penta Pacific to file
an action for ejectment. The CA affirmed the ruling of the RTC that the MeTC had no
jurisdiction over the case.
Issue: Whether the complaint was for unlawful detainer, or accion publiciana, or accion
reivindicatoria.
Ruling: The SC ruled that a defendants claim of possession de jure or his averment of
ownership does not render the ejectment suit either accion publiciana or accion reivindicatoria.
The suit remains accion interdictal, a summary proceeding that can proceed independently of
any claim of ownership. Even when the question of possession cannot be resolved without
deciding the issue of ownership, the issue of ownership is to be resolved only to determine the
issue of possession.
CADAG 97
Javier vs Lomuntad

Facts: In his forcible entry complaint, petitioner Homer C. Javier, represented by his mother and natural
guardian SusanG. Canencia, alleged that he is one of the sons of the late Vicente T. Javier (Vicente), who
was the owner of a 360-square meter (sq. m.) parcel of land located at Corner Malaya and Gonzaga
Streets, Barangay Dolores, Taytay Rizal (subject land). Petitioners family contented that since Javier was
born, he has lived in the residential house erected thereon. Upon Vicentes death, petitioner, together
with his mother, continued their possession over the same. On March 26, 2007, despite the petitioners
vigorous objections, respondent Susan Lumontad gained entry into the subject land and started to build
a two (2)-storey building (subject building) on a 150 sq. m. portion thereof. The dispute was submitted
to barangay conciliation but no amicable settlement was reached between the parties. Thus, petitioner
was constrained to file against respondent the instant forcible entry complaint, averring, in addition to
the foregoing, that reasonable compensation for the use and occupancy of the above-said portion may
be fixed at 5,000.00 per month.

The MTC dismissed the complaint for want of cause of action and lack of jurisdiction.

It was found that Vicente actually subdivided the subject land into two (2) lots: the first lot, with an area
of 187.20sq. m., was given to petitioner, while the second lot, with an area of 172.80 sq. m. and where
the subject building was erected, was given to one Anthony de la Paz Javier (Anthony), son of Vicente by
a previous failed marriage, but was eventually acquired by respondent from the latter through
sale. Based on this finding, the MTC concluded that petitioner had no cause of action against respondent
since she was merely exercising her rights as the owner of the 172.80 sq. m. subdivided lot.

Dissatisfied, petitioner appealed to the RTC. RTC reversed and set aside the MTC ruling, and ordered
respondent to vacate the disputed portion and surrender possession thereof to petitioner.

On the merits, the RTC found that petitioner, being the owner and possessor of the property in
question, has the right to be respected in his possession and that respondent forcibly and unlawfully
deprived him of the same.

Unconvinced, respondent moved for reconsideration, which was denied. Thus the petitioner filed
an appeal before the CA. The CA Ruling Set aside the RTC ruling and remanded the case to the latter
court for trial on the merits.
CAGAMPANG 98
98. EDCA Publishing and Distributing Corp. v The Spouses Leonor and Gerardo Santos
GR No. 80298
April 26, 1990
Facts:
On October 5, 1981, a person who identified himself as Professor Jose Cruz ordered by telephone
406 books with EDCA. Upon delivery, Cruz issued a personal check covering the purchase price of
P8,995.65. Cruz thereafter sold 120 books to Leonor Santos. Santos first verified the ownership of
the books from the invoice Cruz showed her and paid P1,700.00.
Suspicious since Cruz placed another order even before clearing his first check, EDCA inquired with
the De la Salle College where he claimed that he was a dean. The college informed EDCA that there
was no such person in its employ. It was also found out that Cruz had no more account with the
bank against which he had drawn the payment check.
With the help of the policemen, Cruz was trapped. Investigation disclosed his real name as Tomas
de la Pea. It was also found out that 120 of the books he ordered were sold to Santos. EDCA,
accompanied by the police, thereafter went to Santos Bookstore and seized the said books.
The Santoses sued for the recovery of the books after EDCA rejected their demand for its return.
MTC decided in favor of the Santoses. RTC sustained MTCs ruling and CA also affirmed it. Hence,
this petition.
EDCA argues that it still is the owner of the books since the impostor acquired no title that he could
validly transfer to the Santoses. EDCA reasoned that as the payment check bounced for lack of
funds, there was a failure of consideration that nullified the contract of sale between it and Cruz.
Issue:
Whether EDCA has been unlawfully deprived of the books because the check issued by the impostor
in payment therefor was dishonored.
Held:
The contract of sale is consensual and is perfected once agreement is reached between the parties
on the subject matter and the consideration.
According to the Civil Code:
Art. 1475. The contract of sale is consensual and is perfected at the moment there is a meeting of
minds upon the thing which is the object of the contract and upon the price.
From that moment, the parties may reciprocally demand performance, subject to the provisions of
the law governing the form of contraf=cts.
Art. 1477. The ownership of the thing sold shall be transferred to the vendee upon the actual or
constructive delivery thereof.
Art. 1478. The parties may stipulate that ownership in the thing shall not pass to the purchaser
until he has fully paid the price.
From the above provisions, it is clear that the ownership of the thing sold shall not pass to the
buyer until full payment of the purchase price only if there is a stipulation to that effect. Otherwise,
the rule is that ownership is transferred from the seller to the buyer upon actual or constructive
delivery of the thing even if the purchase price has not yet been paid. Non-payment only creates a
right to demand payment or to rescind the contract, or to criminal prosecution in case of bouncing
checks. But absent such stipulation, delivery of the thing sold effectively transfers ownership to the
buyer who can in turn transfer it to another.
In this case, there was actual delivery of the books. Thus, Cruz acquired ownership over the subject
of the sale which he could validly transfer to the Santoses. The fact that Cruz had not paid EDCA for
the books was a matter between him and EDCA and did not impair the title acquired by the
Santoses.
Art. 559 of the Civil Code states that the possession of movable property acquired in good faith is
equivalent to a title, thus dispensing with further proof. Leonor Santos first ascertained that the
books belonged to Cruz before she purchased them by looking into the invoice. Santos did not have
to go beyond that invoice to satisfy herself that the books offered for sale by Cruz belonged to him;
yet she did.
Petition is denied.
CAPUYAN 99

BPI Family Bank v Franco


G.R. No. 123498, November 23, 2007

Facts:
Franco opened 3 accounts with BPI with the total amount of P2,000,000.00. The said
amount used to open these accounts is traceable to a check issued by Tevesteco. The funding for
the P2,000,000.00 check was part of the P80,000,000.00 debited by BPI from FMICs account
(with a deposit of P100,000,000.00) and credited to Tevestecos account pursuant to an
Authority to Debit which was allegedly forged as claimed by FMIC.

Tevesteco effected several withdrawals already from its account amounting to


P37,455,410.54 including the P2,000,000.00 paid to Franco. Franco issued two checks which
were dishonoured upon presentment for payment due to garnishment of his account filed by BPI.

BPI claimed that it had a better right to the amounts which consisted of part of the money
allegedly fraudulently withdrawn from it by Tevesteco and ending up in Francos account. BPI
urges us that the legal consequence of FMICs forgery claim is that the money transferred by BPI
to Tevesteco is its own, and considering that it was able to recover possession of the same when
the money was redeposited by Franco, it had the right to set up its ownership thereon and freeze
Francos accounts.

Issue: WON the bank has a better right to the deposits in Francos account.

Held:
No. Significantly, while Article 559 permits an owner who has lost or has been
unlawfully deprived of a movable to recover the exact same thing from the current possessor,
BPI simply claims ownership of the equivalent amount of money, i.e., the value thereof, which it
had mistakenly debitedfrom FMICs account and credited to Tevestecos, and subsequently
traced to Francos account.

Money bears no earmarks of peculiar ownership, and this characteristic is all the more
manifest in the instant case which involves money in a banking transaction gone awry. Its
primary function is to pass from hand to hand as a medium of exchange, without other evidence
of its title. Money, which had been passed through various transactions in the general course of
banking business, even if of traceable origin, is no exception.
CASTILLO -100
DEMETRIA DE GUZMAN v. FILINVEST DEVELOPMENT CORPORATION
G.R. NO. 191710, 2015

Facts: Petitioners were co-owners of a parcel of land which is enclosed and surrounded by other
real properties belonging to various owners. One of its adjoining properties is owned and
developed by respondent Filinvest(respondent) which, from petitioners' property, has a potential
direct access to Marcos highway. Petitoners filed a complaint for an Easement Right of Way and
argue that it is unfair to require them to pay the value of the affected road lots since the same is
tantamount to buying the property without them being issued titles and not having the right to
exercise dominion over it.

Issue: WON payment of indemnity in a right of way results to alienation of subject lots.

Held: No. Payment of the value of the land for permanent use of the easement does not mean an
alienation of the land occupied. Unlike in purchase of a property, should the right of way no longer
be necessary and the servient estate demands that the easement be extinguished, the value of the
property received by the servient estate by way of indemnity shall be returned in full to the
dominant estate. This only reinforces the concept that the payment of indemnity is merely for the
use of the right of way and not for its alienation.
CENIZA 101

Reyes vs. Spouses Ramos


February 11, 2015 G.R. No. 194488

FACTS:
On March 2006, petitioner Reyes filed a Complaint for easement of right of way against
respondents, Spouses Ramos alleging that she was the registered owner of a 450-square-meter
parcel of land, and that the property used to be a portion of Lot No. 3-B8 and was surrounded by
estates belonging to other persons. Petitioner claims that respondents' 1,500-square-meter
property surrounded her property, and that it was the only adequate outlet from her property to
the highway. A 113-square-meter portion of respondents' property as the easement sought was
also the "point least prejudicial to the respondents.

ISSUE: whether petitioner has the compulsory easement of right of way over respondents'
property

DECISION:
NO. ART. 650 provides that: The easement of right of way shall be established at the point least
prejudicial to the servient estate, and, insofar as consistent with this rule, where the distance from
the dominant estate to a public highway may be the shortest.

Based on these provisions, the following requisites need to be established before a person
becomes entitled to demand the compulsory easement of right of way:
1. An immovable is surrounded by other immovables belonging to other persons,
and is without adequate outlet to a public highway;
2. Payment of proper indemnity by the owner of the surrounded immovable;
3. The isolation of the immovable is not due to its owner's acts; and
4. The proposed easement of right of way is established at the point least prejudicial
to the servient estate, and insofar as consistent with this rule, where the distance
of the dominant estate to a public highway may be the shortest.

Based on the Ocular Inspection Report, petitioner's property had another outlet to the highway.
Furthermore, the convenience of the dominant estate's owner is not the basis for granting an
easement of right of way, especially if the owner's needs may be satisfied without imposing the
easement. The petitioner failed to establish that there was no other adequate outlet to the public
highway and that the proposed easement was the least prejudicial to respondents' estate as
improvements has been introduced thereon. Access to the public highway can be satisfied
without imposing an easement on the spouses' property.
CHUN 102

102. FAJARDO VS FREEDOM TO BUILD


FACTS:Freedom To Build, Inc., an owner-developer and seller of low-cost housing
sold to petitioner-spouses a house and lot and in the contract to sell, there contained a
Restrictive Covenant providing for some prohibitions regarding the expansion to the
house which the owners would like to do with their house. The controversy arose when
the petitioner-spouses extended the roof of their house to a point directly above the
original front wall wherein respondent filed an action to demolish the unauthorized
structures on the RTC and when it favored the respondent, the spouses filed an appeal
but the appellate court affirmed the decision of the lower court, hence this petition.

ISSUE:

(1) Whether or not the Restrictive Covenant is valid

(2)Whether or not the structures built by the petitioner- spouses are legal and valid.

RULING:1. While it may be correct to state that restrictive covenants on the use of
land or the location or character of buildings or other structures thereon may broadly be
said to create easements or rights, it can also be contended that such covenants, being
limitations on the manner in which one may use his own property, do not result in true
easements, but a case of servitudes (burden), sometimes characterized to be negative
easements or reciprocal negative easements.

In its Memorandum, respondent states in arguing for the validity of the restrictive
covenant that the -

"x x x restrictions are not without specific purpose. In a low cost socialized housing, it is
of public knowledge that owners developers are constrained to build as many number of
houses on a limited land area precisely to accommodate marginalized lot buyers,
providing as much as possible the safety, aesthetic and decent living condition by
controlling overcrowding. Such project has been designed to accommodate at least 100
families per hectare."

2. There appears to be no cogent reasons for not upholding restrictive covenants aimed
to promote aesthetics, health, and privacy or to prevent overcrowding.

The Court held that since the extension constructed exceeds the floor area limits of the
Restrictive Covenant, petitioner-spouses can be required to demolish the structure to
the extent that it exceeds the prescribed floor area limits for as Article 1168 of the New
Civil Code provides that when the obligation consists in not doing and the obligor does
what has been forbidden him, it shall be undone at his expense.
CHUN 103
103. Abellana v. CA, G.R. No. 97039, Apr. 24, 1992

Facts: The petitioners who live on a parcel of land abutting the northwestern side of the Nonoc
Homes Subdivision, sued to establish an easement of right of way over a subdivision road which,
according to the petitioners, used to be a mere footpath which they and their ancestors had been
using since time immemorial, and that, hence, they had acquired, through prescription, an easement
of right of way therein. The construction of a wall by the respondents around the subdivision
deprived the petitioners of the use of the subdivision road, which gives the subdivision residents
access to the public highway. They asked that the high concrete walls enclosing the subdivision and
cutting off their access to the subdivision road be removed and that the road be opened to them.

The private respondents denied that there was a pre-existing footpath in the place before it was
developed into a subdivision. They alleged furthermore that the Nonoc Subdivision roads are not
the shortest way to a public road for there is a more direct route from the petitioners land to the
public highway.

Issue: Whether or not the use of a footpath is a continuous easement.

Held: The use of a footpath or road may be apparent but it is not a continuous easement because its
use is at intervals and depends upon the acts of man. It can be exercised only if man passes or puts
his feet over somebody elses land.
CONSTANTINO 104

104. Bicol Agro-Ind vs Obias

G.R. No. 172077

Oct. 09, 2009

Facts:

Bicol Sugar Development Corporation (BISUDECO) constructed a disputed road and


the road was used by BISUDECO in hauling and transporting sugarcane to and from its
mill site. Respondents barricaded the disputed road and thus preventing the petitioners
vehicles from passing through. Petitioner alleged that BISUDECO constructed the road
pursuant to an agreement with the owners of the ricefields that the road traversed.
Petitioner contends that BISUDECO acquired a right of way over the properties of the
landowners, which right of way in turn was acquired by the petitioner when it bought
BISUDECOs assets.

Held:

In order for the petitioner to acquire the disputed road as an easement of right-of-way,
it is incumbent upon them to show its right by title or by an agreement with the owners
of the land that said land traversed.
CUSTODIO 105
QUINTANILLA V. ABANGAN
G.R. No. 160613 February 12, 2008

FACTS:
This controversy flows from a case for Easement of Right of Way filed by petitioner against
respondent Pedro Abangan (Pedro) and respondent Daryl's Collection International, Inc. (DARYL'S).

Petitioners sought the imposition of an easement of right of way.

The RTC dismissed the case for lack of merit. The RTC held that petitioners failed to establish
that the imposition of the right of way was the least prejudicial to the servient estate. The RTC noted that
there is already a concrete fence around the area and that six (6) meters from the said concrete fence was a
concrete warehouse. Thus, substantial damage and substantial reduction in area would be caused the
servient estate. Aggrieved, petitioners went to the CA on appeal. The CA affirmed the RTC Decision,

ISSUE: W/N Mere convenience is the basis required by law for setting up a compulsory easement

RULING:

No, it is not. As between a right of way that would demolish a fence of strong materials to
provide ingress and egress to a public highway and another right of way which although longer will only
require a van or vehicle to make a turn, the second alternative should be preferred. Mere convenience is
not what is required by law as basis for setting up a compulsory easement.

Furthermore, Apolinardito failed to discharge the burden of proving the existence and
concurrence of all the requisites in order to validly claim a compulsory right of way against respondents.
DABA 106

QUIMEN V. CA- Easement

FACTS:

The shares of Anastacia and 3 other siblings on a piece of property were next to the municipal road. A portion of
the lots behind Anastacias were sold by her (as her brothers administrator) brother to Yolanda.

Yolanda was convinced to buy back the property and constructed a house thereon despite having no access to the
public road because Anastacia assured her that she would give her a right of way on her adjoining property for
p200 per square meter, but she was thereafter barred by Anastacia from passing through her property.

Yolanda filed an action with the proper court praying for a right of way through Anastacias property. The
proposed right of way was at the extreme right of Anastacias property facing the public highway, starting from
the back of the sari-sari store and extending inward by 1m to her property and turning left for about 5m to avoid
the store in order to reach the municipal road. The way was unobstructed except for an avocado tree standing in
the middle.

ISSUE:

Whether or not there was a valid grant of an easement and the right of way proposed by Yolanda is the least
onerous/least prejudicial to the parties

HELD: YES

1) The conditions for a valid grant of an easement of right of way:


(a) the dominant estate is surrounded by other immovables without an adequate outlet to a public highway;

(b) the dominant estate is willing to pay the proper indemnity;

(c) the isolation was not due to the acts of the dominant estate; and,

(d) the right of way being claimed is at a point least prejudicial to the servient estate.

were clearly present. The evidence clearly shows that the property of private respondent is hemmed in by the
estates of other persons including that of petitioner; that she offered to pay P200.00 per square meter for her
right of way as agreed between her and petitioner; that she did not cause the isolation of her property; that the
right of way is the least prejudicial to the servient estate.

LEAST DAMAGE > SHORTEST DISTANCEWhen the easement may be established on any of several tenements
surrounding the dominant estate, the one where the way is shortest and will cause the least damage should be
chosen. However, as elsewhere stated, if these two (2) circumstances do not concur in a single tenement, the way
which will cause the least damage should be used, even if it will not be the shortest.

DADANG 107
107. Unisource Commercial and Development Corporation v. Chung et,al.
GR. No. 173252 July 17,2009

Facts:
A memorandum of encumbrance of a voluntary easement entered in favor of
Hidalgo was annotated at the back of the OCT of Sandico. Sandico sold his land
to herein petitioner carrying over the encumbrance on the Transfer Certificate
Title. Hidalgo also sold his land to herein respondents Chung. The petitioner
then filed a petition to Cancel the Encumbrance contending that the easement
is not necessary on the ground that the dominant estate now has an adequate
access to a public road.

Issue: Whether or not the Memorandum of Encumbrance can be cancelled?

Held: No, the opening of an adequate access to a public road extinguishes only
legal or compulsory easements, not voluntary easements. This easement of
right of way, like any other contract, could be extinguished only by mutual
agreement or by renunciation of the owner of the dominant estate. (Art. 631,
NCC)
DARIMBANG 108

108. Linda Rana vs Teresita Lee Wong, Sps. Ong, and Sps. Uy
June 30, 2014
G.R. No. 192861
[Abatement of Nuisance and Damages

Facts:
Wong and Sps. Ong are co-owners of a land in Lahug, Cebu City bordering a subdivision road.
Across the Wong-Ong property are the adjacent lots of Sps. Uy and Sps. Rana. In 1997, Sps. Rana
elevated and cemented a portion of the road that runs between the Rana and Wong-Ong properties
to level the said portion with their gate. Sps. Rana backfilled a portion of the perimeter fence
without erecting a retaining wall.
Wong, et al availed of the remedy of judicial abatement of damages against Sps. Rana, claiming that
the elevated and cemented subject portion affected their usual manner of ingress and egress, and
that the subject backfilling may cause sudden collapse of land, and thus should be declared
nuisances which curtailed their use and enjoyment of their properties.

Issues:

1. WON the cemented subject portion and subject backfilling constituted a nuisance per se and
thus should be abated

Ruling:
No, the subject portions are not nuisances per se. It is not injurious to the health or comfort of the
community, as defined under Art. 694 of the Civil Code. It was primarily built to facilitate the
ingress and egress of Sps. Rana. Since the subject portion is not a nuisance per se, it cannot be
summarily abated. But for introducing a nuisance per accidens that particularly transgressed the
aforesaid rights, Sps. Rana should be liable for nominal damages.
DATUDACULA 109
Case 109: GR NO. 211356 September 29, 2014
CRISOSTOMO B. AQUINO, Petitioner, v. MUNICIPALITY OF MALAY, AKLAN,
REPRESENTED BY HON. MAYOR JOHN P. YAP, et.al, Respondents.

FACTS: To close and demolish the Boracay West Cove Hotel, the Municipal Mayor of Aklan
issued Executive Order 10. The antecedents of the case reveal that Crisostomo, the
president and CEO of Boracay Island West Cove Management Philippines, Inc applied for a
zoning compliance to build a three-storey hotel with the municipal government, which the
Municipal Zoning Administrator denied because the construction violates the no-build
zone of Municipal Ordinance No. 2000-131, which prohibits constructions 25 meters from
the edge of the mean high water mark. Crisostomo filed an appeal with the Municipal
Mayor, but no action was taken thereon. On April 5, 2011, a Notice of Assessment was sent
to Crisostomo demanding payment for unpaid real estate estate taxes and other liabilities
under pain of closure because of its continuous operation without the required building
permit, zoning permit, and business and mayors permit. Crisostomo expressed willingness
to pay the companys obligation but the municipal treasurer refused to accept his tender of
payment. After a Cease and Desist Order was issued by the municipal government,
enjoining the expansion of the resort, the Office of the Mayor of Malay, Aklan issued the EO
10, ordering the closure and demolition hotel. EO 10 was partially implemented.
Crisostomo filed a petition for certiorari with the Court of Appeals, alleging that the order
was issued with grave abuse of discretion; that judicial proceedings are first necessary
before the hotel may be closed and demolished; that the hotel is a grantee of FLAGt., that
the area is a forestland thus the DENR had jurisdiction over it. In its comment, the
municipality argued that the FLAGt does not excuse the company from compliance with the
Ordinance and the National Building Code, and that the mayor is granted express powers
under the Local Government Code to demolish illegally built structures.
The CA dismissed the petition for certiorari and held that the exercise of the power of the
mayor was not done as a quasi-judicial function; hence not correctible by certiorari. The
proper remedy for Crisostomo was to file a petition for declaratory relief with the Regional
Trial Court. Crisostomo elevated his case to the Supreme Court, alleging that the
demolition of the hotel was beyond the municipal mayors powers.

ISSUE: Whether or not respondent mayor committed grave abuse of discretion when he
issued EO 10.

HELD: No. The Court ruled that the property involved cannot be classified as
a nuisance per se which can therefore be summarily abated. It is merely the hotels
particular incident- its location- because it was constructed in a no build zone and not its
inherent qualities that rendered it a nuisance. Even if the hotel is not a nuisance per se, it is
still a nuisance per accidens. Hence, the LGU may nevertheless properly order the hotels
demolition in the exercise of its police power and the general welfare clause. The
property rights of individuals may be subjected to restraints and burdens in order to fulfill
the objectives of the government. Moreover, the Local Government Code authorizes city
and municipal governments, acting through their local chief executives, to issue demolition
orders. The office of the mayor has quasi-judicial powers to order the closing and
demolition of establishments.
DEL CASTILLO 110
BUSTAMANTE VS TELMO

Facts:

A complaint was filed by respondent Luciano M. Bustamante against petitioner Guillermo Telmo, Municipal
Engineer of Naic, Cavite, among others. The complaint alleged that respondent is a co-owner of a real property in
Naic, Cavite. Petitioners (Telmos) are the owners of the two (2) parcels of land located at the back of respondents
lot. The complaint further alleged that respondent caused the resurvey of his lot in the presence of the Telmos, and
that the resurvey showed that the Telmos encroached upon respondents lot. Subsequently, Respondent put up
concrete poles on his lot. However, the Telmos and their men allegedly destroyed the concrete poles. Respondent
complained that he and his co-owners did not receive any just compensation from the government when it took a
portion of their property for the construction of the Noveleta-Naic-Tagaytay Road. Worse, they could not enjoy the
use of the remaining part of their lot due to the abusive, Illegal, and unjust acts of the Telmos. Thus, respondent
charged the latter criminally and administratively.

Held:

Petitioner Guillermo Telmo is found administratively guilty of discourtesy in the course of official duties.

Ratio:

First. Petitioner claims that his act of summarily removing respondents concrete posts was authorized under the
National Building Code (Presidential Decree No. 1096). The provision he cites correctly pertains to Section 215. To
better understand this provision, we refer to Section 214 of the same law, which defines what are dangerous and
ruinous buildings or structures susceptible of abatement.

The court disagrees with Petitioners contention. A careful reading of the foregoing provisions would readily
show that they do not apply to the respondents situation. Nowhere was it shown that the concrete posts put up by
respondent in what he believed was his and his co-owners property were ever declared dangerous or ruinous, such
that they can be summarily demolished by petitioner.

Second. Petitioner contends that respondents concrete posts were in the nature of a nuisance per se, which may be
the subject of summary abatement sans any judicial proceedings. Again, we disagree.

Again, the court disagrees with Petitioners contention. A nuisance per se is that which affects the immediate
safety of persons and property and may be summarily abated under the undefined law of necessity. Evidently, the
concrete posts summarily removed by petitioner did not at all pose a hazard to the safety of persons and properties,
which would have necessitated immediate and summary abatement.

Third. Petitioner likewise maintains that his authority to perform the assailed official act sprang from Section 23 of the
Revised Philippine Highway Act. He posits that this provision is particularly implemented by Department Order No.
52, Series of 2003 of the Department of Public Works and Highways for the Removal of Obstructions and Prohibited
Uses within the Right-of-Way of National Roads.

Gauging the action of petitioner based on the guidelines set by Department Order No. 52, from which he
claims his authority, the court cannot but conclude that petitioner went beyond the scope of his official power because
it is the concerned District Engineer of the Department of Public Works and Highways who should have ordered
respondent to remove the concrete posts. The petitioner failed to show that he was duly authorized by the District
Engineer to implement the Department Order in Naic, Cavite. More importantly, even assuming that petitioner had
been duly authorized to order the removal of the concrete posts of respondent, he failed to prove that he issued the
required notice to respondent to remove the said structures before he did the removal himself. Note that petitioner, in
fact, admitted in his pleadings that he summarily removed the said posts.
DURIAS 111
G.R. NO. 177807/G.R. NO. 177933, October 11, 2011
EMILIO GANCAYCO VS CITY GOVERNMENT OF QUEZON CITY AND MMDA

FACTS:
In 1950s, retired justice Emilio Gancayco bought a parcel of land located in EDSA. Then on
March 1956, Quezon City Council issued Ordinance No. 2904 requiring the construction of
arcades for commercial buildings to be constructed. Under this particular ordinance, the city
council required that the arcade is to be created by constructing the wall of the ground floor
facing the sidewalk a few meters away from the property line. Thus, the building owner is not
allowed to construct his wall up to the edge of the property line, thereby creating a space or
shelter under the first floor.
The ordinance covered the property of Justice Gancayco. Subsequently, sometime in 1965,
Justice Gancayco sought the exemption of a two-storey building being constructed on his
property from the application of Ordinance No. 2904 that he be exempted from constructing an
arcade on his property.
Decades after, in March 2003, MMDA conducted operations to clear obstructions along EDSA,
in consequence, they sent a notice of demolition to Justice Gancayco alleging that a portion of
his building violated the National Building Code.
Gancayco did not comply with the notice and filed a petition for TRO with the RTC Quezon City
to prohibit the MMDA from demolishing his property. The RTC rendered its Decision on 30
September 2003 in favor of Justice Gancayco.
MMDA appealed with the CA. CA held that the MMDA went beyond its powers when it
demolished the subject property. It further found that Resolution No. 02-28 only refers to
sidewalks, streets, avenues, alleys, bridges, parks and other public places in Metro Manila, thus
excluding Justice Gancaycos private property. Lastly, the CA stated that the MMDA is not
clothed with the authority to declare, prevent or abate nuisances.

ISSUE: WON MMDA legally demolished the property of Justice Gancayco.

HELD:
MMDA illegally demolished the property of Justice Gancayco.
MMDA alleges that by virtue of MMDA Resolution No. 02-28, Series of 2002, it is empowered
to demolish Justice Gancaycos property. It insists that the Metro Manila Council authorized the
MMDA and the local government units to clear the sidewalks, streets, avenues, alleys, bridges,
parks and other public places in Metro Manila of all illegal structures and obstructions. It further
alleges that it demolished the property pursuant to the Building Code in relation to Ordinance
No. 2904 as amended.
However, the Building Code clearly provides the process by which a building may be
demolished. The authority to order the demolition of any structure lies with the Building
Official.
Additionally, the penalty prescribed by Ordinance No. 2904 itself does not include the
demolition of illegally constructed buildings in case of violations. Instead, it merely prescribes a
punishment. The ordinance itself also clearly states that it is the regular courts that will determine
whether there was a violation of the ordinance.
EBDAO 112
G.R. No. 184478 March 21, 2012 PEREZ V. MADRONA
FACTS: Respondent-spouses Fortunito Madrona and Yolanda B. Pante are registered owners of a
residential property located in Greenheights Subdivision, Phase II, Marikina City and covered by
TCT No. 169365 of the Registry of Deeds of Marikina. In 1989, respondents built their house thereon
and enclosed it with a concrete fence and steel gate. In 1999, respondents received the following
letter dated May 25, 1999 from petitioner Jaime S. Perez, Chief of the Marikina Demolition Office
stating that the structure that they built encroached on the sidewalk and that is in violation of PD
1096 of the National Building Code and RA 917 on Illegally occupied/constructed improvements
within the road right-of-way. The respondent-spouses are given 7 days to remove the said structure.
As response, respondent Madrona sent petitioner a letter stating that the May 25, 1999 letter (1)
contained an accusation libelous in nature as it is condemning him and his property without due
process; (2) has no basis and authority since there is no court order authorizing him to demolish
their structure; (3) cited legal bases which do not expressly give petitioner authority to demolish; and
(4) contained a false accusation since their fence did not in fact extend to the sidewalk. More than a
year later or on February 28, 2001, petitioner sent another letter with the same contents as the May
25, 1999 letter but this time giving respondents ten days from receipt thereof to remove the
structure allegedly protruding to the sidewalk. This prompted respondents to file a complaint for
injunction before the Marikina City RTC on March 12, 2001. Respondents likewise sought the
issuance of a temporary restraining order (TRO) and a writ of preliminary injunction to enjoin
petitioner and all persons acting under him from doing any act of demolition on their property and
that after trial, the injunction be made permanent. On March 16, 2001, the RTC issued a TRO
against petitioner. On July 27, 2004, the RTC rendered a Decision in favor of respondents. The RTC
decision permanently enjoined defendant Perez from performing any act which would tend to
destroy or demolish the perimeter fence and steel gate of the respondents property. The RTC held
that respondents, being lawful owners of the subject property, are entitled to the peaceful and open
possession of every inch of their property and petitioners threat to demolish the concrete fence
around their property is tantamount to a violation of their rights as property owners who are entitled
to protection under the Constitution and laws. The RTC also ruled that there is no showing that
respondents fence is a nuisance per se and presents an immediate danger to the communitys
welfare, nor is there basis for petitioners claim that the fence has encroached on the sidewalk as to
justify its summary demolition. CA affirmed.
ISSUE: W/N respondents structure is a nuisance per se that presents immediate danger to the
communitys welfare and can be removed without need of judicial intervention since the clearing of
the sidewalks is an infrastructure project of the Marikina City Government and cannot be restrained
by the courts as provided in Presidential Decree No. 1818.
HELD: No. If petitioner indeed found respondents fence to have encroached on the sidewalk, his
remedy is not to demolish the same summarily after respondents failed to heed his request to
remove it. Instead, he should go to court and prove respondents supposed violations in the
construction of the concrete fence. Indeed, unless a thing is a nuisance per se, it may not be abated
summarily without judicial intervention. Our ruling in Lucena Grand Central Terminal, Inc. v. JAC
Liner, Inc., on the need for judicial intervention when the nuisance is not a nuisance per se, is well
worth mentioning. In said case, we ruled: Respondents can not seek cover under the general
welfare clause authorizing the abatement of nuisances without judicial proceedings. That tenet
applies to a nuisance per se, or one which affects the immediate safety of persons and property and
may be summarily abated under the undefined law of necessity (Monteverde v. Generoso, 52 Phil.
123 [1982]). The storage of copra in the quonset building is a legitimate business. By its nature, it
can not be said to be injurious to rights of property, of health or of comfort of the community. If it be a
nuisance per accidens it may be so proven in a hearing conducted for that purpose.It is not per se a
nuisance warranting its summary abatement without judicial intervention. [Underscoring supplied.]
In Pampanga Bus Co., Inc. v. Municipality of Tarlac where the appellant-municipality similarly argued
that the terminal involved therein is a nuisance that may be abated by the Municipal Council via an
ordinance, this Court held: Suffice it to say that in the abatement of nuisances the provisions of the
Civil Code (Articles 694-707) must be observed and followed. This appellant failed to do.
Respondents fence is not a nuisance per se.By its nature, it is not injurious to the health or comfort
of the community. It was built primarily to secure the property of respondents and prevent intruders
from entering it. And as correctly pointed out by respondents, the sidewalk still exists. If petitioner
believes that respondents fence indeed encroaches on the sidewalk, it may be so proven in a
hearing conducted for that purpose.Not being a nuisance per se, but at most a nuisance per
accidens, its summary abatement without judicial intervention is unwarranted.
FELICIANO 113

BUMAGAT VS ARRIBAY

CHARLES BUMAGAT, et al. v. REGALADO ARRIBAY G.R. No. 194818, 9 June 2014, SECOND
DIVISION, (Del Castillo, J.) A case involving agricultural land does not immediately qualify it as an
agrarian dispute. The mere fact that the land is agricultural does not ipso facto make the possessor
an agricultural lessee or tenant. There are conditions before he can qualify as an agricultural lessee
or tenant, and the subject being agricultural land constitutes just one condition. In order to qualify
as an agrarian dispute, there must likewise exist a tenancy relation between the parties. Bumagat
and others are the registered owners of about eight hectares of agricultural land. They filed a
complaint for forcible entry against Arribay before the Municipal Circuit Trial Court (MCTC)
alleging that with the aid of armed goons and through the use of intimidation and threats of
physical harm, the latter entered the formers parcels of land and ousted them from their lawful
possession. Arribay sought for the dismissal of the complaint, claiming that the subject properties
are agricultural lands which renders the dispute an agrarian matter and subject to the exclusive
jurisdiction of Department of Agrarian Reform Adjudication Board (DARAB). The Municipal
Agrarian Reform Office (MARO) denied the motion for failure to show the existence of a tenancy or
agrarian relationship between the parties. The Municipal Circuit Trial Court (MCTC) found that no
tenancy or other agrarian relationship existed between the parties. The Regional Trial Court (RTC)
affirmed. The Court of Appeals (CA) reversed the RTC and agreed that the dispute fell under the
jurisdiction of the DARAB.
ISSUE: Is the dispute within the jurisdiction of DARAB?
RULING: No. The CA failed to realize the fact that as between the parties, there is no tenurial
arrangement, not even an implied one. For the DARAB to acquire jurisdiction over the case, there
must exist a tenancy relation between the parties. In order for a tenancy agreement to take hold
over a dispute, it is essential to establish all its indispensable elements, to wit: 1) that the parties
are the landowner and the tenant of agricultural lessee; 2) that the subject matter of the
relationship is an agricultural land; 3) that there is consent between the parties to the relationship;
4) that the purpose of the relationship is to bring about agricultural production; 5) that there is
personal cultivation on the part of the tenant or agricultural lessee; and 6) that the harvest is
shared between the landowner and the tenant or agricultural lessee. In the present case, it is quite
evident that not all of these conditions are present. For one, there is no tenant, as both parties claim
ownership over the property
GACUS 114
Republic of the Philippines, represented by the Secretary of Agriculture
-versus-
Federico Daclan, et al.

G.R. No. 197115 23 March 2015

FACTS:
In 1972, the respondents donated four (4) parcels of land in La Union to the government in
order to establish the Agoo Breeding Station. The donation was subject to the following conditions:
1) shall be used solely for the establishment of a breeding station; 2) shall not be used for any other
purposes, except with the previous consent of the donors or their heirs; and 3) in case of non-use,
abandonment or cessation of the activities of the Bureau of Animal Industry, possession or
ownership shall automatically revert to the donor and all permanent improvements existing
thereon shall become the property of the donor. After the donations were made, the La Union
Medical Center was constructed on 1.5-hectare portion of the 13 hectare donated property.
In 1991, the establishment of the Local Government Code empowered the Province of La
Union to assumed powers and functions of the Dept. of Agriculture in the operation of the breeding
station. In 2003, the respondents demanded the return of their donated lands on the ground that
the breeding station has ceased operations and the land has been abandoned.

ISSUE: Whether the property can be reverted to the respondents/donors.

HELD:
No. The preponderance of evidence points to the fact that the breeding station remained
operational even after its transfer from the Republic to the Province. The activities of the Bureau of
Agricultural Industry did not cease even after it was dissolved after the government adopted the
policy of devolution under the Local Government Code of 1991; these activities were merely
transferred to the Province.
The court cannot validly order the return the donated 1.5-hectare portion where the LUMC
is situated to the respondent because they did not donate that portion. Respondent admitted that
the 1.5-hectare portion where the LUMC is constructed does not form part of the lands they
donated to the government, but belonged other donors who are not parties to the instant case. As
far as the
Daclans are concerned, whatever they donated remains part of the breeding station and so long as
it remains so, no right of reversion accrues to them. Only the original owner-donor of the 1.5-
hectare portion where the LUMC is constructed is entitled to its return.
GALES 115
Esperanza C. Carinan vs Spouses Gavino Cueto and Carmelita Cueto
GR No. 198636
October 8, 2014
Facts:
The respondent spouses alleged that in May 1986 that the petitioner and her husband
acquired the rights to a parcel of land in Laguna under the name of the GSIS and were
to assume payment of the monthly amortizations. The petitioner and her husband did
not pay several amortizations thus the GSIS planned to cancel the conditional sale in
2005. Petitioner, who was widowed by then went to Gavino, her brother, for financial
help and the respondents paid the total obligation to GSIS along with the payment
for transfer and renovation of the residential house. The petitioner and Jazer, the
petitioners son, executed a Deed of Absolute Sale with the condition to buy the
land back within three years. The petitioner surrendered the TCT to the respondents.
The respondents demanded from the petitioner to fulfill the transfer of the property
through the deed of sale by the petitioner and her son did not comply. The respondents
filed with the RTC seeking refund of the total expenses paid plus damages and
attorneys fees.
Petitioner countered saying that it was donation and the payment was made out of
generosity and pity towards the petitioner and there was no written or oral agreement
for transfer or repayment.
The RTC ruled in favor of the respondents and CA affirmed the decision of the RTC.
Issue:
Whether or not the agreement was a contract of loan or donation.
Ruling:
The SC affirmed the decision of the RTC and CA.
The SC ruled that the purely gratuitous act was not supported by records citing the Civil
Code that any donation over Php500 shall be in writing otherwise it is void. The formal
requirements are mandatory according to Article 748 and non compliance results in a
void donation. The petitioner imputed deceit on the part of the respondent, a serious
charge that cannot be proven by mere allegation.
GARCES - 116
116. REPUBLIC VS GUZMAN
FACTS: David Rey Guzman, a natural-born American citizen, is the son of the spouses Simeon
Guzman (naturalized American) and Helen Meyers Guzman (American citizen). In 1968, Simeon
died leaving to his heirs, Helen and David, an estate consisting of several parcels of land in Bulacan.
In 1970, Helen and David executed a Deed of Extrajudicial Settlement of the Estate, dividing and
adjudicating to themselves all of the property, and registered it to the Register of Deeds a year after.
In 1981, Helen executed a Deed of Quitclaim, assigning, transferring and conveying her share of
the properties to David. But since it was not registered, she executed another Deed of Quitclaim to
confirm the first.
In 1994, Atty. Batongbacal wrote the Office of the Solicitor General and furnished it with
documents showing that Davids ownership of of the estate was defective. He argued that Art. XII
of the Constitution only allows Filipinos to acquire private lands in the country. The only instances
when a foreigner may acquire private property are by hereditary succession and if he was formerly
a natural-born citizen who lost his Filipino citizenship. Moreover, it contends that the Deeds of
Quitclaim executed by Helen were really donations inter vivos.
Republic filed with RTC a Petition for Escheat praying that of Davids interest be forfeited in its
favor. RTC dismissed. CA affirmed.
ISSUE: Whether or not there was a donation inter vivos.
HELD: No. Not all the elements of a donation are present. The transfer of the properties by virtue of
a Deed of Quitclaim resulted in the (1) reduction of Helens patrimony as donor and the (2)
consequent increase in the patrimony of David as donee. However, Helens (3) intention to perform
an act of liberality in favor of David was not sufficiently established. The 2 Quitclaims reveal that
Helen intended to convey to her son certain parcels of land and to re-affirm it, she executed a
waiver and renunciation of her rights over these properties. It is clear that Helen merely
contemplated a waiver of her rights, title, interest over the lands in favor of David, not a donation.
She was also aware that donation was not possible.
Moreover, the essential element of acceptance in the proper form and registration to make the
donation valid is lacking. The SPA executed by David in favor of Atty. Abela was not his acceptance,
but an acknowledgment that David owns the property referred to and that he authorizes Atty. Abela
to sell the same in his name. Further, there was nothing in the SPA to show that he indeed accept
the donation.
However, the inexistence of a donation does not make the repudiation of Helen in favor David valid.
There is NO valid repudiation of inheritance as Helen had already accepted her share of the
inheritance when she, together with David, executed a Deed of Extrajudicial Settlement of the
Estate, dividing and adjudicating between them all the properties. By virtue of that settlement, the
properties were registered in their names and for 11 years, they possessed the land in the concept
of owner. Thus, the 2 Quitclaims have no legal force and effect. Helen still owns of the property.
GERMANES 117
Villanueva vs Branoco

Syllabus. Donation

Branoco claimed ownership over a property through purchase from Rodriguez, who had acquired the
said property from Rodrigo by way of donation. Rodrigo subsequently sold the same property to Vere.

Issue. Whether or not the contract of donation of Rodriguez and Rodrigo was effectively cancelled when
Rodrigo sold the property to Vere.

Ruling. No. Rodrigo passed naked title to Rodriguez under a perfected donation inter vivos. Rodrigo
stipulated that if the herein Donee predeceases me, the property will not be reverted to the Donor, but
will be inherited by the heirs of Donee, signaling the irrevocability of the passage of title to
Rodriguezs estate, waiving Rodrigos right to reclaim title.

Hence, the subsequent sale of the property did not destroy the acquired vested right of Rodriguez to the
property. Consequently, the claim of ownership by Branoco is lawful as the sale of the property between
Rodriguez and Branoco is valid.
IBANEZ 118
Central Philippines University vs C.A.
Facts:
In 1939, late Don Ramon Lopez. Sr. executed a deed of donations in favor of Central
Philippines University (CPU) of a parcel of land with the following conditions:
The land shall be exclusively use for the establishment of and use of a medical college.
CPU shall not sell, transfer or convey to any third party or any way the land.
The said land shall be called Ramon Lopez Campus
However in 1989, the respondents who are heirs of Don Ramon filed an action for
annulment of donation, re-conveyance and damages against CPU alleging that since 1939
has not complied with the conditions of the donation and that the University negotiate with
National House Authority (NHA) to exchange the donated land with another land.
ISSUE:
Whether or not all the rights of the done are deemed lost and extinguished because of the
non-fulfillment of the conditions?

SUPREME COURT RULING:


Supreme Court finds that since the records are clear and facts are undisputed that since the
execution of the deed of donation until filing of instant action, petitioner has failed to
comply with its obligation as done. CPU has slept its obligation for unreasonable length of
time so it is only just and equitable to declare the donation ineffective.
Supreme Court declared that the conditions set forth in the donation were resolutory
conditions which mean the fulfillment of the conditions extinguishes the obligation.

Under Art. 1181 of the Civil Code, in conditional obligations, the acquisition of rights,
as well as extiguishment or loss of those already acquired shall depend upon
happening of the event which constitutes the condition. Thus, when a person donated a
land to another on the condition that the order would build on the land a school. The
condition imposed was not a condition precedent or suspensive condition but a
resolutory condition.
When obligations does not fix a period from its nature and circumstances it can be inferred
that the period was intended, the general rule provided in Art. 1197 of the Civil Code
applies, which provides that the court may fix the durations thereof because the
fulfillment of the obligations itself cannot be demanded until the court has fixed the
period for compliance therewith and such period has arrived.
However, this general rule cannot be applied in this case. More than a reasonable period of
50 years has been allowed for petitioner to avail of the opportunity to comply with the
obligations
Art. 1191 of Civil Code, when one of the obligators cannot comply with what is
incumbent upon him, the oblige may seek rescission and the court shall decree the
same unless there is just cause authorizing the fixing of the period
JALA 119

G.R. No. L-44059


October 28, 1977

THE INSULAR LIFE ASSURANCE COMPANY, LTD. v. CARPONIA T.


EBRADO and PASCUALA VDA. DE EBRADO

Facts:
Cristor Ebrado was issued by The Life Assurance Co., Ltd., a policy for P
5,882.00 with a rider for Accidental Death. He designated Carponia T. Ebrado
as the revocable beneficiary in his policy. He referred to her as his wife. Cristor
was killed when he was hit by a failing branch of a tree. Insular Life was made
liable to pay the coverage in the total amount of P 11, 745.73, representing the
face value of the policy in the amount of P 5, 882.00 plus the additional
benefits for accidental death.

Carponia T. Ebrado filed with the insurer a claim for the proceeds as the
designated beneficiary therein, although she admitted that she and the insured
were merely living as husband and wife without the benefit of marriage.
Pascuala Vda. de Ebrado also filed her claim as the widow of the deceased
insured. She asserts that she is the one entitled to the insurance proceeds.

Insular commenced an action for interpleader before the trial court as to who
should be given the proceeds. The lower court declared Carponia as
disqualified. Carponia filed an appeal.

Issue:
WON a common-law wife named as beneficiary in the life insurance policy of
a legally married man can claim the proceeds in case of death of the latter.

Held:

No. The SC affirmed the decision of the lower court that Corponia, a common-
law wife, cannot claim the proceeds stated in the life insurance policy.

Section 50 of the Insurance Act provides that "the insurance should be applied
exclusively to the proper interest of the person in whose name it is made". The
word "interest" highly suggests that the provision refers only to the "insured"
and not to the beneficiary, since a contract of insurance is personal in
character. Otherwise, the prohibitory laws against illicit relationships especially
on property and descent will be rendered nugatory, as the same could easily be
circumvented by modes of insurance.
When not otherwise specifically provided for by the Insurance Law, the contract
of life insurance is governed by the general rules of the civil law regulating
contracts. And under Article 2012 of the same Code, any person who is
forbidden from receiving any donation under Article 739 cannot be named
beneficiary of a fife insurance policy by the person who cannot make a
donation to him. Common-law spouses are barred from receiving donations
from each other.
JALAGAT 120
LADERA 121

121. QUIJADA VS. CA, G.R. NO. 126444, DEC. 4, 1998

Facts:
a.Subject Land located in Talacagon
b.Trinidad Quijada inherited it from her father

On April 5, 1956, Trinidad Quijada, together with her siblings executed a


Conditional Deed of Donation in favor of the municipality of Talacagon, the condition is
being that the land shall be used exclusively for the construction of a provincial high
school. Trinidad remained in possession of the Land. On July 29, 1962, Trinidad sold
the land to Regalado Mondejar. In 1980, the heirs to Trinidad (herein Petitioners) filed a
complaint for forcible entry against the respondents. In 1987, the proposed campus did
not materialized and the Sangguniang Bayan enacted a resolution donating back the
land to the donor. In the meantime, Mondejar conveyed portions of the land to complaint
for quieting of the title recovery of possession and ownership of the land.

Issue:
W/N the sale between Trinidad and Mondejar is valid considering the capacity of
the vendor to execute the contract in new of the conditional deed of donation.

Held:
The Donor may have an inchoate interest in the donated property during the time
that ownership of the land has not reverted to her. Such inchoate interest maybe the
subject of contracts including a contract of sale. In this case, what the donor sold was
the land itself which she no longer owns. It would have been different if the donor-seller
sold her interest over the property under the deed of donation which is subject to the
possibility of reversion of ownership arising from the non-fulfillment of the resolutory
condition.
LAUT 122

CASE #122
Metropolitan Fabrics Inc. and Enrique Ang vs. Prosperity Credit Resources Inc., Domingo Ang
and Caleb Ang
G.R. 154390. 17 March 2014

TOPIC: Action to Assail Mortgage

FACTS:
Metropolitan Fabrics Inc. (MFI) owned a 5.8 ha industrial compound in Quezon City. Pursuant to a
P2M, 10-year loan agreement with Manpil Investment Corporation (Manpil) dated April 6, 1963, the lot was
subdivided into 11 lots, with Manpil retaining 4 lots as mortgage security while the remaining 7 lots were
released to MFI.
In July 1984, MFI obtained a loan from PCRI in the amount of P3.5M, represented by herein
respondents Domingo and Caleb Ang. The blank loan forms had no entries specifying the rate of interest and
schedules of amortization. In order to return the trust and gesture of early release of the loan by the
respondents, herein petitioner Enrique Ang, together with his daughter Vicky Ang, entrusted to the
respondents their seven (7) titles covering an aggregate area of 3.36 ha and left it to said respondents to
choose from among the 7 titles those which would be sufficient to secure the P3.5M loan.
An appraisal report put the value of four (4) of the said properties at P6.8M. Vicky also stated that it
was agreed that once PCRI had chosen the lots to be covered by the mortgage, the respondents would return
the remaining titles to the petitioners. Thereafter, twenty four (24) checks bearing no dates and amounts and
signed in blank by Enrique and Natividad, were delivered to PCRI to cover the amortization payments. In
September 1984, the first amortization check bounced for insufficient fund due to MFIs continuing losses. It
was then that the petitioners learned that PCRI had filled up the said checks with dates and amounts reflected
at 35% interest rate per annum, instead of just 24%, and a two-year repayment period, instead of 10 years. It
was only upon such time that PCRI finally furnished MFI with its copy of the loan documents. Petitioners
found the terms to be prohibitive, burdensome, and unconscionable, and further averred that had they known
them they would have either negotiated or rejected the terms of the loan and withdrew the application. Due
to losses, petitioners business operations stopped.
An offsetting agreement was executed by the parties to cover the loan obligation amounting to
P4.1M. Thereafter, Vicky furnished respondents a copy of the appraisal report prepared by Integrated
Appraisal Corporation. However, PCRIs statement showed that all seven (7) titles were placed as collateral
for their P3.5M loan. Petitioners averred that as per the appraisal report, the value of the properties covered
by the said titles were largely in excess of the loan obligation.
On September 1986, petitioner Enrique received a Notice of Sherriffs Sale announcing the auction of
the seven lots due to an unpaid indebtedness of P10.5M. Vicky insisted that prior to the notice, they never
received any statement or demand letter from the defendants to pay the said amount, nor did the
respondents inform them of the intended foreclosure. The auction was then reset to a later date after
petitioners assured PCRI that they had found a serious buyer for the lots.
In the meeting held between the parties and the said buyer, Winston Wang, it was agreed that the
mortgage was to be released upon payment of P3.5M with an initial down-payment of P500,000.00 to be paid
by MFI to PCRI as partial settlement of the P3.5M loan. Thereafter, Winston Wang confronted Vicky about the
sale agreement and PCRIs refusal to accept the P3M payment because according to the respondent Caleb, the
three lots had been foreclosed. However, the said foreclosure was executed before the lapse of the agreed 60-
day period for the payment of the balance. At the auction, PCRI was the sole bidder. Subsequent agreements
were further held for the release of the disputed three lots involving all three parties. Upon failure to raise the
required money for the payments on account of such agreements, MFI was ultimately forced to vacate the
lots. The RTC ruled in favour of the petitioners. However, the CA reversed the decision and dismissed the
complaint.

ISSUE: Whether or not the action to assail the mortgage already prescribed.
HELD:
YES. The Court held that in order to resolve the issue of prescription, it is important to first
determine if the mortgage was void or merely voidable. As held by the CA, the petitioners contention of
absence of consent which would make the mortgage void was untenable. Herein petitioners failed to prove
that they had been forced or coerced to enter into the mortgage. Where consent was given through fraud
alone, the contract was voidable, not void ab initio. With the contract being voidable, petitioners action to
annul the real estate mortgage already prescribed. Article 1390, in relation to Article 1391 of the Civil
Code, provides that if the consent of the parties was obtained through fraud, the contract is
considered voidable and may be annulled within four (4) years from the time of the discovery thereof.
The discovery of said document was reckoned from the time the document was registered in the Register of
Deeds in view of the rule that registration was notice to the whole world. Thus, because the mortgage
involving the seven lots was registered on September 5, 1984, they had until September 5, 1988 within which
to assail the validity of said mortgage. But their complaint was instituted in the RTC on October 10, 1991.
Hence, the action had already prescribed.
LOPEZ 123

Rural Bank of Cabadbaran v Melecio-Yap [GR 178451]


FACTS:
Erna Melecio-Yap and her 5 siblings inherited a 3,044 square meter residential lot from their
parents. Said lot had been a subject of a real estate mortgage following a loan procured from
petitioner RBCI, as authorized by a notarized Special Power of Attorney (SPA) purportedly
executed by Ernas parents. Upon default in payment, the lot was sold to RBCI as the highest bidder
in the public auction sale.
Respondents claimed that they were not aware of the loan obtained by Erna and did not authorize
the mortgage transaction over the lot which they co-owned, and that the SPA bore their falsified
signatures. RBCI replied that the SPA was valid, the same being a notarized document carrying the
presumption of regularity. Having relied upon such document, RBCI claims to be a mortgagee in
good faith.
RTC declared the mortgage and the consequential foreclosure proceedings to be valid and binding,
but the CA held otherwise, saying that the respondents were able to prove, by preponderance of
evidence, that their signatures were indeed forged. Thus, the mortgage transaction only affects
Ernas 1/6 share in the property.
ISSUE:
W/N the subject property was validly mortgaged and foreclosed
W/N RBCI was a mortgagee in good faith
HELD:
No, on both issues.
The forged SPA is sufficient grounds to render the mortgage null and void, insofar as the shares of
the other co-owners, whose consent thereto were not actually obtained, are concerned. Pursuant to
Art. 493 of the Civil Code, Erna, as a co-owner, has the right to mortgage or sell her own undivided
interest in the property, but not the entire property itself, the disposal of which requires the
consent of the other co-owners. Thus, RBCI may validly own, by virtue of the foreclosure
proceedings, only that portion of the property belonging to Erna.
The RBCI cannot be considered a mortgagee in good faith since (1) such doctrine applies only to
lands registered under the Torrens system and not to unregistered lands, such as the property in
the suit; and (2) the principle is inapplicable to banking institutions who are called upon to exercise
greater care and prudence before entering into a contract of mortgage.
MACABANDO 124

G.R. No. 197857 September 10, 2014


SPOUSES FRANCISCO SIERRA vs.
PAIC SAVINGS AND MORTGAGE BANK, INC

FACTS:

On May 31, 1983, Goldstar Conglomerates, Inc. (GCI), represented by Guillermo Zaldaga
(Zaldaga), obtained from Summa Bank, now respondent Paic Savings and Mortgage Bank, Inc.
(PSMB) a loan in the amount of P1,500,000.00 as evidenced by a Loan Agreement. As security
therefor, GCI executed in favor of PSMB six (6) promissory notes6 in the aggregate amount of
P1,500,000.00 as well as a Deed of Real Estate Mortgage over a parcel of land covered by
Transfer Certificate of Title (TCT) No. 308475.7 As additional security, petitioners Francisco
Sierra, Rosario Sierra, and Spouses Felix Gatlabayan and Salome Sierra mortgaged four(4)
parcels of land in Antipolo City. Eventually, GCI defaulted in the payment of its loan to PSMB.
The latter to extrajudicially foreclose the mortgage on the subject properties. Since petitioners
failed to redeem the subject properties within the redemption period, their certificates of title
were cancelled and new ones were issued in PSMBs name.

Petitioners averred that under pressing need of money, with very limited education and lacking
proper instructions, they fell prey to a group who misrepresented to have connections with
Summa Bank and, thus, could help them secure a loan. Petitioners likewise lamented that they
were not furnished copies of the loan and mortgage documents, or notified/apprised of the
assignment to PSMB, rendering them unable to comply with their obligations under the subject
deed. They further claimed that they were not furnished a copy of the statement of account nor
a copy of the petition for foreclosure prior to the precipitate extrajudicial foreclosure and auction
sale which failed to comply with the posting and notice requirements.

PSMB maintained that: (a) it acted in good faith with respect to the subject transactions and that
petitioners action should be directed against the group who deceived them;27 (b) the subject
properties were mortgaged to securean obligation covered by the loan agreement with GCI;28
(c) the mortgage was valid, having been duly signed by petitioners before a notary public;29 (d)
the foreclosure proceedings were regular, having complied with the formalities required by
law;30 and (e) petitioners allowed time to pass without pursuing their purported right against
Summa Bank and/or PSMB.

ISSUE: 1. WHETHER OR NOT PETITIONERS ARE MERE ACCOMODATION


MORTGAGAGORS
2. W/N THE ACTION HAS PRESCRIBED AND BARRED BY LACHES

HELD:

Petitioners claim of lack of "proper instruction on the intricacies in securing [the] loan from the
bank" is belied by the fact that petitioners Francisco and Rosario Sierra had previously
mortgaged two (2) of the subject properties twiceto the Rural Bank of Antipolo. Moreover,
petitioners did not: (a) demand for any loan document containing the details of the transaction,
and (b) offer to pay the purported partial loan proceeds they received at any time, complaining
thereof only in 1991 when they filed their complaint. Indeed, the foregoing circumstances clearly
show that petitioners are aware that they were mere accommodation mortgagors, debunking
their claim that mistake vitiated their consent to the mortgage.
As jurisprudence states, an accommodation mortgagor is a third person who is not a debtor to a
principal obligation but merely secures it by mortgaging his or her own property.

Anent the 2nd issue, Petitioners contends that the applicable provision is the ten-year
prescriptive period of mortgage actions under Article 1142 of the Civil Code. Based on case law,
a "mortgage action" refers to an action to enforce a right necessarily arising from a mortgage.59
In the present case, petitioners are not "enforcing"their rights under the mortgage but are, in
fact, seeking to be relieved therefrom.The complaint filed by petitioners is, therefore, not a
mortgage action as contemplated under Article 1142.

Since the complaint for annulment was anchored on a claim of mistake, i.e., that petitioners are
the borrowers under the loan secured by the mortgage, the action should have been brought
within (4) years from its discovery. The discovery of the averred mistake should appear to be
reckoned from June 19, 1984.

As the records disclose, despite notice on June 19, 1984 of the scheduled foreclosure sale,
petitioners, for unexplained reasons, failed to impugn the real estate mortgage and oppose the
public auction sale for a period of more than seven (7) years from said notice.
MACUMBAL 125

FACTS:

Respondent domestic corporation is engaged in the real estate business and is the developer of
the St. Francis Square Commercial Center (built sometime in 1992).It filed separate complaints
against petioners before the IPO - BLA, namely:

(a) IPV Case an intellectual property violation case for unfair competion, false or fraudulent
declaration, and damages arising from petioners use and filing of applications for the
registration of the marks THE ST. FRANCIS TOWERS and THE ST. FRANCIS SHANGRI-LA
PLACE,; and(b) St. Francis Towers IP Case an inter partes case opposing the petioners
application for registration of the mark THE ST. FRANCIS TOWERS for use relative to the
latters business, particularly the construction of permanent buildings or structures for
residential and office purposes; and

(c) St. Francis Shangri-La IP Case an inter partes case opposing the petioners application for
registration of the mark THE ST. FRANCIS SHANGRI-LA PLACE,.

Respondent alleged that it has used the mark ST. FRANCIS to identify its numerous property
development projects located at Ortigas Center, such as the aforementioned St. Francis Square
Commercial Center, a shopping mall called the St. Francis Square, and a mixed-use realty
project plan that includes the St. Francis Towers. Respondent added that as a result of its
continuous use of the mark ST. FRANCIS in its real estate business, it has gained substantial
goodwill with the public that consumers and traders closely identify the said mark with its
property development projects. Accordingly, respondent claimed that petioners could not have
the mark THE ST. FRANCIS TOWERS registered in their names, and that petioners use of the
marks THE ST. FRANCIS TOWERS and THE ST. FRANCIS SHANGRI-LA PLACE in their own real
estate development projects constitutes unfair competion as well as false or fraudulent
declaration.

Petioners denied committng unfair competion and false or fraudulent declaration, maintaining
that they could register the mark THE ST. FRANCIS TOWERS and THE ST. FRANCIS SHANGRI-
LA PLACE under their names. They contended that respondent is barred from claiming
ownership and exclusive use of the mark ST. FRANCIS because the same is geographically
descriptive of the goods or services for which it is intended to be used. This is because
respondents as well as petioners real estate development projects are located along the
streets bearing the name St. Francis, particularly, St. Francis Avenue and St. Francis Street
(now known as Bank Drive), both within the vicinity of the Ortigas Center.

ISSUE:

Whether or not petioners are guilty of unfair competition in using the marks THE ST. FRANCIS
TOWERS and THE ST. FRANCIS SHANGRI-LA PLACE.

RULING:
NO.

RATIO DECIDENDI:

The unfair competion concept refers to the the passing off (or palming off) or attempting to
pass off upon the public of the goods or business of one person as the goods or business of
another with the end and probable effect of deceiving the public. Passing off (or palming off)
takes place where the defendant, by imitative devices on the general appearance of the goods,
misleads prospective purchasers into buying his merchandise under the impression that they
are buying that of his competitors. [In other words], the defendant gives his goods the general
appearance of the goods of his competitor with the intention of deceiving the public that the
goods are those of his competitor. The true test of unfair competion has thus been whether
the acts of the defendant have the intent of deceiving or are calculated to deceive the ordinary
buyer making his purchases under the ordinary conditions of the particular trade to which the
controversy relates. Based on the foregoing, it is therefore essential to prove the existence of
fraud, or the intent to deceive, actual or probable, determined through a judicious scrutiny of
the factual circumstances attendant to a particular case. Here, the Court finds the element of
fraud to be wanting; hence, there can be no unfair competion.
MADRONO - 126
ABADIANO 127 G.R. No. 195956, 1 March 2015

FACTS
A criminal complaint was filed by ABS-CBN against GMA for violating Sections 177 and 211
of the Intellectual Property Code, as amended by RA 8293. This was because respondent GMA
aired footage of the arrival and homecoming of Angelo Dela Cruz from Iraq at NAIA without the
consent of the petitioner. The petitioner claimed that there was an agreement between them and
Reuters in exchange of the latters news and video material. Additionally, Reuters ensured the
petitioner the latters materials cannot be aired in the country. GMA was a subscriber of Reuters
and CNN live feeds. After having received the live feed of Angelo Dela Cruzs arrival and
homecoming, it immediately aired the video from that news feed. However, the respondent not
only alleged its news staff was not aware of the agreement between ABS-CBN and Reuters but
also its staff had no knowledge that it aired petitioners footage. On the 3rd of December 2004,
the Assistant City Prosecutor found probable cause to indict the staff of GMA namely, Dela Pena
and Manalastas. Consequently, the respondents appealed the Prosecutors resolution before the
DOJ, and the latter ruled in favor of the respondents and held that good faith may be raised as a
defense in the case. Meanwhile, DOJ Acting Secretary Alberto C. Agra issued a resolution which
reversed Sec. Gonzaless resolution and found probable cause to charge Dela Pena, Manalastas,
as well to indict Gozon, Duavit Jr., Flores, and Soho. Subsequently, Court of Appeals granted the
Petition for Certiorari to reverse and set aside the resolution of the Acting DOJ and granted the
issuance of temporary restraining order and/or Writ Preliminary Injuction.
ISSUE
Whether or not there is probable cause to find respondents to be held liable criminally for the
case of copyright infringement under the Intellectual Property Law (RA 8293, as amended)?
HELD
The petition of ABS-CBN was partially granted by the Supreme Court. An order was directed to
the Q.C. Branch 93 to continue the proceedings against the staff of GMA, Dela Pena and
Manalastas due to copyright infringement. The other respondents, Gozon, Duavit Jr., Flores, and
Soho were held not liable for the criminally; act of copyright infringement. It was held that their
membership in GMAS Board of Directors doesnt mean that they have knowledge, approval, or
participation in the criminal act, as there is a need for the direct or active participation in such
act. Also, there was lack of proof they actively participated or exercised moral ascendancy over
Dela Pena and Manalastas.

The Court held that the respondents mere act of rebroadcast of petitioners news footage without
the latters authority creates probable cause to find the formers personnel criminally liable for
violating the provisions of Intellectual Property Code, particularly Section 216-217 of RA 8293,
as amended, since both have not exercised diligence in their function to prevent that the footage
from being aired on television. They were aware there would be consequences in carrying ABS-
CBN footage in their broadcast.
FAELNAR -128
Taiwan Kolin vs Kolin Electronics
GR 209843 March 25, 2015

Facts:
Taiwan Kolin Corp. sought to register the trademark KOLIN in Class 9 on the following
combination of goods: television sets, cassette recorder, VCD amplifiers, camcorders and other
audio/video electronic equipment, flat iron, vacuum cleaners, cordless handsets, videophones,
facsimile machines, teleprinters, cellular phones, and automatic goods vending machine.
Kolin Electronics opposed the application on the ground that the trademark KOLIN is identical, if
not confusingly similar, with its registered trademark KOLIN which covers the following products
under Class 9 of the Nice Classification (NCL): automatic voltage regulator, converter, recharger,
stereo booster, AC-DC regulated power supply, step-down transformer, and PA amplified AC-DC.
Kolin Electronics argued that the products are not only closely-related because they fall under the
same classification, but also because they are inherently similar for being electronic products and
are plugged into electronic sockets and perform a useful function.

Issue:
W/N the products are closely related

Held:
No, the products are not related and the use of the trademark KOLIN on them would not likely
cause confusion. To confer exclusive use of a trademark, emphasis should be on the similarity or
relatedness of the goods and/or services involved and not on the arbitrary classification or general
description of their properties or characteristics.
First, products classified under Class 9 can be further classified into 5 categories. Accordingly, the
goods covered by the competing marks between Taiwan Kolin and Kolin Electronics fall under
different categories. Taiwan Kolins goods are categorized as audio visual equipments, while Kolin
Electronics goods fall under devices for controlling the distribution and use of electricity. Thus, it is
erroneous to assume that all electronic products are closely related and that the coverage of one
electronic product necessarily precludes the registration of a similar mark over another.
Second, the ordinarily intelligent buyer is not likely to be confused. The distinct visual and aural
differences between the two trademarks KOLIN, although appear to be minimal, are sufficient to
distinguish between one brand or another. The casual buyer is predisposed to be more cautious,
discriminating, and would prefer to mull over his purchase because the products involved are
various kind of electronic products which are relatively luxury items and not considered affordable.
They are not ordinarily consumable items such as soy sauce, catsup or soap which are of minimal
cost. Hence, confusion is less likely.

You might also like